阎育苏 译 北京:科学出版社,1982年
第一部分 在教室中
目的
1.帮助学生
教师最重要的任务之一是帮助学生。这个任务并不很简单,它需要时间、
实践、热忱以及健全合理的原则。
学生应当有尽可能多的独立工作经验。但是如果让他独自面对问题而得不到任何帮助或者帮助得不够。那么他很可能没有进步。但若教师对他帮助过多,
那么学生却又无事可干,教师对学生的帮助应当不多不少,恰使学生有一份合理的工作。
如果学生不太能够独立工作,那末教师也至少应当使他感觉自己是在独立工作。为了做到这一点,教师应当考虑周到地、不显眼地帮助学生。
不过,对学生的帮助最好是顺乎自然。教师对学生应当设身处地,应当了解学生情况,应当弄清学生正在想什么,并且提出一个学生自己可能会产生的问题,或者指出一个学生自己可能会想出来的步骤。
2.问题、建议、思维活动
在打算对学生进行有效、不显眼而又自然的帮助时,教师不免一而再,再而三地提出一些相同的问题,指出一些相同的步骤。这样,在大量的问题中,
我们总是问:未知数是什么?我们可以变换提法,以各种不同的方式提问同一个问题:求什么?你想找到什么?你假定求的是什么?这类问题的目的是把学生的注意力集中到未知数上。有时,我们用一条建议:看着未知数,来更为自然地达到同一效果。问题与建议都以同一效果为目的:即企图引起同样的思维活动。
从作者看来,在与学生讨论的问题中,收集一些典型的有用问题和建议,
并加以分类是有价值的。前面这张表就包含了这类经过仔细挑选与安排的问题和建议;它们对于那些能独立解题的人也同样有用。读者充分熟悉这张表并且看出在建议之后所应采取的行动之后,他会感到这张表中所间接列举的是对解题很有用的典型思维活动。这些思维活动在表中的次序是按其发生的可能性大小排列的。
3.普遍性
表中所提问题与建议的重要特点之一是普遍性,例如:未知数是什么?已知数是什么?条件是什么?这些问题都是普遍适用的,对于所有各类问题,我们提出这些问题都会取得良好效果。它们的用途不限于任何题目。我们的问题可以是代数的或几何的,数学的或非数学的,理论的或实际的,一个严肃的问题或仅仅是个谜语。这没什么差别,上述问题都是有意义的,而且有助于我们解题。
事实上,还存在一个限制,不过这与论题无关。表中某些问题与建议,只能用于“求解题”而不能用于“求证题”。如果我们的问题属于后者,则必须采用别的提问方法,见第三部分“求解题,求证题”这一段。
4.常识
我们这张表中的问题与建议是具有普遍性的,但是除去其普遍性以外,它们也是自然的、简单的、显而易见的并且来自于普通常识。例如这条建议:看着未知数!试想出一个具有相同未知数或类似未知数的熟悉的问题,这条建议不管怎样总是劝告你去做你想做的事,而对于你认真要解决的问题并未提出具体的劝告。你是不是肚子饿了?如果你希望搞点吃的,你就会想起你所熟悉的搞到食物的一些办法。你是不是有一个几何作图题?如果你想作一个三角形,你也会想起你所熟悉的一些作三角形的办法。你是否有一个任意的问题?你若希望找出某个未知数,你就会想起找出这样一个未知数或你所熟悉的类似未知数的一些办法。如果你这样做了,那你的路子也是对头的;这个建议是个好建议,它向你提出一个常能成功的程序。
我们表中的所有问题与建议都是自然的、简单的、显而易见的,而且只不过是普通常识;但是这张表把常识概括地加以叙述。这张表所提出的处理办法对于那些认真对待其问题并有某些常识的人来说是很自然的。然而按正确道路行动的人往往不注意用明确的语言来表达其行动,而且他可能根本不会这样做;
我们这张表却尝试去表达这些。
5.教师与学生,模仿与实践
当教师向学生提出表中的问题或建议时,他可能有两个目的:第一,帮助学生解决手头的问题;第二,培养学生将来能够独立解题的能力。
经验证明,适当使用我们表中的问题与建议,常能对学生有所裨益。此表有两个特点:常识性与普遍性。由于此表来源于普通常识,所以显得很自然,
学生自己也会提出这类问题。由于此表具有普遍性,所以它们对学生的帮助并非强加于人;它们只不过指出了一般的方向,而留给学生去做的还很多。
上述两个目的是密切相关的。如果学生在解决手边的问题中获得成功,他就提高了一些解题的能力。这时,我们不应该忘记我们所提问题具有普遍性而且可适用于许多情况。如果同一个问题反复地对学生有所帮助,那么他就会注意到这个问题,于是在类似的情况下,他自己就会提出这个问题。通过反复地提出这个问题,他总会有一次成功地诱导出正确的念头。通过这样一次成功,
他便发现了利用这个问题的正确途径,于是,他真正地领会了它。
学生可能对我们表中的一些问题领会得很好,以致他最终能够在恰当的时刻向自己提出正确的问题,并进行相应的自然而活跃的思维活动。这样,学生就无疑从我们的表中得到了尽可能多的收获。为了得到尽可能好的结果,教师可以做些什么事呢?
解题,譬如,就好象游泳一样,是一种实际技能。当你学习游泳时,你模仿其他人的手足动作使头部保持在水面上并最后通过实践(实地练习游泳)来学会游泳。当试图解题时,你也必须观察并模仿其它人在解题时的所作所为,并且最后通过实践来学会解题。
希望提高学生解题能力的教师,必须培养学生的兴趣,然后给他们提供大量的机会去模仿与实践。如果教师想要在他的学生中发展相应于我们表中的问题与建议的思维活动,那么他就应该尽可能地经常而自然地向学生提出这些问题和建议。此外,当教师在全班面前解题时,他应当使其思路更吸引人一些,
并且应当向自己提出那些在帮助学生时所使用的相同问题。由于这样的指导,
学生将终于找到使用表中这些问题与建议的正确方法,并且这样做以后,他将学到比任何具体数学知识更为重要的东西。
主要部分,主要问题
6.四个阶段
在求解过程中,我们很可能再三地改变我们的观点,或者改变考虑问题的途径。我们应该不断地变更我们的出发点。当我们开始着手解题时,我们对问题的概念可能很不完整;当我们有些进展以后,我们的看法就不同了;而当我们几乎已经得到解答的时候,看法就会更不相同。
为了把我们表中的问题与建议进行适当分组,我们把工作分为四个阶段。
首先,我们必须了解问题;我们必须清楚地看到要求的是什么?其次,我们必须了解各个项之间有怎样的联系?未知数和数据之间有什么关系?为了得到解题的思路,应该制定一个计划。第三,实现我们的计划。第四,我们回顾所完成的解答,对它进行检查和讨论。
上述每一阶段都有其重要性。可能会有这样的情况:一个学生想出了一个异常好的念头,于是跳过所有的预备步骤,解答就脱口而出了。如此幸运的念头当然是求之不得的,但是也可能发生很不如愿和很不走运的事:即,学生通过上述四阶段中的任何一个阶段都没有想出好念头。最糟糕的情况是:学生并没有理解问题就进行演算或作图。一般说来,在尚未看到主要联系或者尚未作出某种计划的情况下,去处理细节是毫无用处的。如果学生在实行其计划的过程中检查每一步,就可以避免许多错误。如果学生不去重新检查或重新考虑已完成的解答,则可能失去某些最好的效果。
7、弄清问题
回答一个你尚未弄清的问题是愚蠢的。去做一件你不愿干的事是可悲的。
在校内外,这种愚蠢和可悲的事情却经常发生,但教师应力求防止在他的班级里发生这样的事。学生应当弄清问题,然而他不仅应当弄清它,而且还渴望解出它。如果学生对问题没弄清或不感兴趣,这并不是他的过错,问题应当精选,
所选的题目不太难但也不要太容易,应顺乎自然而且趣味盎然,并且有时在叙述方式上也应当自然而有趣。
首先,必须了解问题的文字叙述。教师在某种程度上可以检查这一点,他可以要求学生重新叙述这题目,而学生应能流利地重新叙述这个问题。学生还应当能够指出问题的主要部分,即未知数,已知数据,条件。所以老师提问时,
不要错过这样的问题:未知数是什么?已知数据是什么?条件是什么?
学生应该仔细地、重复地并且从各个方面来考虑问题的主要部分。如果问题和某一图形有关,那末他应该画张图并在上面标出未知数与已知数据。如果对这些对象需要给以名称,他应该引入适当的符号。适当地注意选择符号,他就会被迫考虑这些必须选择符号的对象。在此预备阶段中,假定我们并不期望有一个明确的回答,而只不过想有一个临时性的回答或一个猜测,那么另外还有一个问题可能是有用的,即:满足条件是否可能呢?
(在本书第二部分中,把“弄清问题”分成两个阶段:“熟悉问题”和“深人理解问题”)。
8、例子
让我们说明上节中的某几点内容。 我们选下列简单问题:已知长方体的长、宽、高,求其对角线长度。
为了对此问题作有益的讨论,学生必须熟悉毕达哥拉斯定理及其在平面几何中的某些应用。他们对立体几何可能只有很少的系统知识。教师这时可以依赖学生对空间关系的朴素知识。
教师可以通过使问题具体化而使之有趣。如教室就是个长方体,其尺寸可以测量,也可以估计,要求学生不作测量,间接地求出教室的对角线长度。教师指出教室的长、宽、高,用手势说明什么是对角线,通过不断地和教室相联系而使他画在黑板上的图变得更加形象。
以下是老师与学生间的对话,
,未知数是什么?”
,长方体对角线的长度。”
,已知数是什么?”
,长方体的长、宽、高。”
,引入适当的符号,用哪个字母表示未知数?”
,x”
,长、宽、高应选哪些字母?”
,a,b,c”
,联系a,b,c与x的条件是什么?”
,x是长方体的对角线,长方体的长、宽、高为a,b,c”
,这是个合理的问题吗?我意思是说,条件是否充分,足以确定未知数吗?”
“是的,是充分的。如果我们知道a,b,c,我们就知道平行六面体。如果平行六面体被确定,则对角线也被确定了。”
9.拟定计划
当我们知道,或至少大体上知道,为了求解未知数,必须完成哪些计算、
要作哪些图的时候,我们就有了一个计划。从弄清问题到想出一个计划,其过程可能是漫长而曲折的。事实上,求解一个问题的主要成绩是构想出一个解题计划的思路。这个思路可能是逐渐形成的。或者,在明显失败的尝试和一度犹豫不决之后,突然闪出了一个“好念头”。老师为学生所能做的最大的好事是通过比较自然的帮助,促使他自己想出一个好念头。我们下面就要讨论的问题与建议正是要诱发这样一种好念头。
为了弄清学生的心理活动,老师应当回想他自己的经验,回顾他自己在解题时碰到的困难与取得成功的经验。
我们当然知道,如果我们对该论题知识贫乏,是不容易产生好念头的。如果我们完全没有知识,则根本不可能产生好念头。一个好念头的基础是过去的经验和已有的知识。仅仅靠记忆不足以产生好念头。但若不重新收集一些有关事实,则也不会出现好念头。只有材料还不足以盖房子,但是不收集必需的材料也盖不了房子。解决数学问题所必需的材料是我们早已获得的数学知识的某些有关内容,如以前解决的问题,以前证明过的定理。因此,以下列问题开始工作常常是合适的:你知道一个与此有关的问题吗?
困难就在于:通常有相当多的问题与我们现在手上的问题有关,即,与它有某种共同之处。我们怎样挑出其中一个或几个确实有用的问题呢?我们建议把力量放在主要的共同之处上:看着未知数!试想起一个具有相同或相似未知数的熟悉的问题来。
如果我们成功地回想起一个与当前问题密切相关的早已解决的问题,那是很幸运的。我们应当争取这样的运气;通过探索我们是可以得到它的。 这里有个问题与你的问题有关,且早已解决,你能利用它吗?
上述问题,如能很好地理解和认真地加以考虑,常常有助于激发起一连串正确的想法;但它们并不总是有用的,它们并非魔法。如果这些问题不行,我们必须寻找某些其他的适当接触点,并且探索问题的各个方面;我们不得不变化、变换、修改该问题。你能否重述这个问题?我们表中的某些问题提示了改变问题的专门方法,例如普遍化、特殊化、应用类比、舍去一部分条件等等;具体细节是重要的,但我们现在不能深入讨论。改变问题可能导致提出某种适当的辅助问题:如果你不能解决所提出的问题,则应首先尝试去解决某些与此有关的问题。
尝试去应用各种已知的问题或定理,考虑各种修改,对各种辅助问题进行试验,我们可能离开原来的问题太远,甚至最后有失掉它的危险。但是还有一个很好的问题可以把我们带回原处:你是否利用了所有的已知数据?你是否利用了整个条件?
10.例子
我们回到第8节中的例子。
,你是否知道一个与此有关的问题?”
……
,看着未知数,你是否知道一个具有相同未知数的问题?”
,好,未知数是什么?”
,平行六面体的对角线。”
,你是否知道任何具有相同未知数的问题?”
,不,我们还没有任何关于平行六面体对角线的问题”
,你是否知道任何具有相似未知数的问题?”
……
“你看,对角线是个线段,就是直线的一段。你从来没有解决过一个未知数是直线长度的问题?”
“当然,我们曾经解决过这样的问题,例如找出直角三角形的一个边。”
“好啊! 这里有一个知你的问题有关的问题,且早已解决,你能利用它吗?”
“你真走运,你想起了一个与你当前问题有关的问题,而且这个问题你以前已经解决了。你愿意利用它吗?为了能利用它,你能否引进某个辅助元素?”
图1
“看这里,你所想起的是一个关于三角形的问题。图中有三角形吗?”
我们希望这最后的提示已明白得足以诱发出解题的思路(即引入一个在图
1中用阴影画出的直角三角形)。这个引入的直角三角形的斜边就是我们所要求的对角线。但是教师应当对下述情况有所准备:即使这样明白的提示也不能使学生开窍,那么他应当动用所有越来越明显的提示。
“你是否想在图1中有个三角形?”
“在图中,你想有哪种三角形?”
“你现在还不能求出这对角线;但你说过你能求出三角形的一个边。那么现在你该怎么办呢?”
“如果对角线是三角形的一个边,你能找出它吗?”
经过或多或少的帮助后,学生终于成功地引进了决定性的辅助元素,即图中阴影三角形,在鼓励学生进入实际计算之前,教师应确信其学生对问题的理解已有足够的深度。
“我想,画出那个三角形是个好主意,你现在有了个三角形,但是你是否有未知数?”
“未知数是三角形的斜边,我们可用毕达哥拉斯定理去计算它”
“如果两边为已知,你会计算。但它们是已知的吗?”
“一个边已给定,是c。另一个边,我想也不难求出。是的,另一边是另一个直角三角形的斜边。”
“很好!现在我看出你有个计划了。”
11.实现计划
想出一个计划,产生一个求解的念头是不容易的。要成功需要有许多条件,
如已有的知识、良好的思维习惯、目标集中,还要有好运气。但实现计划则容易得多,我们所需要的主要是耐心。
计划仅给出一个一般性的大纲,我们必须充实细节并耐心地检查每一个细节,直到每一点都完全清楚了,没有任何可能隐藏错误的含糊之处为止。
如果学生真的拟定出一个计划,则教师就比较清闲了。现在的主要危险是学生可能会忘记他的计划。因为那些从外界接受计划的和根据教师的权威来采纳某个计划的学生,很容易发生这种现象;但若是学生自己搞出来的计划(即便经过某种帮助)并且学生满意地看出了最终的思路,则他就不那么容易忘记。教师必须坚持让学生检查每一步骤。
根据“直观”或“形式”上的论证,我们可以使自己相信每一步骤的正确性。我们可以集中力量在有问题的疑点上,直到完全搞清楚,毫不怀疑每一步骤都是正确的为止;或者我们可以根据形式推理的法则推导出有问题的这一点
(在许多重要的场合,直接观察与形式证明二者间的区别是足够明显的;更进一步的讨论让我们留给哲学家们去进行吧!)
主要之点是:学生应当真正地相信每一步骤的正确性。在某些情况老师可以强调“看出来”与“证明”二者之间的差别而提出:你能清楚地看出这一步骤是正确的吗?同时你也能证明这一步骤是正确的吗?
12.例子
我们继续第10节末尾留下的工作。学生最后已经得到了解题的思路。他看出未知数x是直角三角形的斜边,而给定的高度c是边长之一,另一边则是六面体的一个面的对角线。很可能这刚学生被催促引入一个适当的符号。他应当选择y表示另一边,即面上的对角线,其两边为a和b。学生现在可能看得更清楚:
解题的思路就是应该引进一个辅助未知数y0最后,陆续对这两个直角三角形进行考虑之后,他得到
x2=y2+c2
y2=a2+b2于是消去辅助未知数y,从而有
x2=a2+b2+c2
x= 222 cba ++
如果学生正确地进行上述细节运算,老师没有理由去打断他,除非必要时提醒他应当检查每一步。这样,教师可以问,
“你能清楚地看出具有三边x,y,c的三角形是直角三角形吗?”
对于这个问题,学生可能老老实实回答:“是”。但是如果老师不满足于学生的直观猜测,他应该继续提问,
“但是你能证明这个三角形是个直角三角形吗?”
除非整个班级对于立体几何已经有了良好的起点,否则教师不应当提出这个问题。即使如此,也仍然存在某些危险性,即对这个偶然提出问题的回答可能成为大多数学生的主要困难。
13.回顾
即使是相当好的学生,当他得到问题的解答,并且很干净利落地写下论证后,就会合上书本,找点别的事来干干。这样做,他们就错过了解题的一个重要而有教益的方面。通过回顾所完成的解答,通过重新考虑与重新检查这个结果和得出这一结果的路子,学生们可以巩固他们的知识和发展他们解题的能力。
一个好的教师应该懂得并且传授给学生下述看法:没有任何问题是可以解决得十全十美的。总剩下些工作要做。经过充分的探讨与钻研,我们能够改进这个解答,而且在任何情况下,我们总能提高自己对这个解答的理解水平。
现在学生已经完成了他的计划。他已经写出了答案,检查了每一步。这样,
他似乎有充分理由相信他的解答是正确的了。然而,出现错误总还是可能的,
特别当论证冗长而复杂的时候更是如此。所以要验证。特别是,如果有某种快速而直观的办法来检验结果或者检验论证,决不要忽略。你能检验这结果吗?
你能检验这个论证吗?
为了确信某个东西的存在或其质量的好坏,我们总喜欢去看看它,摸摸它。
我们总是通过两种不同的感官来感知它。同样,我们也宁可通过两种不同的证明使我们对结果确信无疑。因此要问:你能用不同方法来导出这结果吗?当然,
我们宁愿要简短而直观的论证,而不要冗长而烦琐的,所以要问:你能一下子看出它吗?
教师的首要职责之一是不要给学生以下述错觉:数学题目之间很少有联系,和任何其他事物则完全没有什么联系。当我们回顾问题解答的时候,我们自然有机会来考察一个问题与其它事物的联系。如果学生已经作出了真诚的努力并且意识到自己完成得不错,那末他们将发现对解答加以回顾确实饶有趣味。
这样,他们就热切地想知道用真诚的努力还可干些什么别的,以及下次他如何能干得同样好。教师应该鼓励学生设想一些情况,在那些情况下,他能再一次利用所使用的办法,或者应用所得到的结果。你能把这结果或这方法用于某个其它问题吗?
14.例子
在第12节,学生最后得到了解答:如果长方体自同一角引出的三个边为a,
b,c,那末对角线为
222 cba ++
你能检验这个结果吗?教师不能指望从缺乏经验的学生那里得到这个问题的良好回答。但是学生应该很早就获得下述经验:用字母表达的问题比纯粹数字题好。对于用字母表示的题,其结果很容易进行几次检验,而用数字表示的题则不然。我们的例子虽然很简单,也足以证明这点。教师可以对结果提出好几个问题,对这些问题,学生可以很容易地回答“是”;但如回答“不是”,
这将表明结果中存在严重的缺点。
“你是否使用了所有的数据?是否所有数据a,b,c都在你的对角线公式中出现?”
“长、宽、高在我们的问题中起的作用是一样的,我们的问题对a,b,c
来说是对称的。你所得的公式对a,b,c对称吗?当a,b,c互换时公式是否保持不变?”
“我们的问题是一个立体几何问题给定尺寸a,b,c,求平行六面体的对角线。我们的问题与平面几何的问题类似:给定尺寸a、b,求矩形的对角线,
这里立体几何问题的结果是否与平面几何的结果类似?”
“如果高c减小,并且最后等于零,这时平行六面体变成平行四边形。在你的公式中,令c=0,是否得到矩形对角线的正确公式?”
“如果高c增加,则对角线也增加。你的公式是否表明这点?”
“如果平行六面体的三个量度a,b,c按同一比例增加,则对角线也按同一比例增加。在你的公式中,如将a,b,c分别代以12a,12b,12c,则对角线也将乘以12,是否这样?”
“如果a,b,c的单位是尺,则你的公式给出的对角线的单位也是尺;如果将所有单位改为寸,则公式应保持正确,是否如此?”
(后两个问题基本上是等价的。参见“量纲检验”一节)
上述一些问题有几个好处。首先,公式通过这么多的检验,这一事实不能不使一个聪明的学生产生深刻的印象。学生以前就相信公式是正确的,因为公式是他仔细推导出来的。但是现在经过这么多检验,他就更深信无疑了,这种信心的增加来源于一种“实验的数据”。正是由于上述问题,公式的细节获得了新的意义,而且和不同的事实联系起来了。这样,公式就更容易记住,学生的知识得以巩固。最后,上述问题很容易转到类似的题目上。对于类似题目获得一些经验以后,一个聪明的学生就能觉察出所包含的普遍概念:即,利用所有有关数据,改变数据,对称,类比。如果他养成了把注意力集中在这些地方的习惯,他解题的能力肯定会提高。
你能检验这个论证吗?在困难而重要的场合,可能需要逐步地重新检验论证。但通常,重新检查一下令人恼火之点就够了。在本例,可以建议讨论以前提过的问题:你能证明具有三边x,y,c的三角形是直角三角形吗(见第12节末尾处)?
你能把这结果或方法用于其它问题吗?在受到一些鼓励并且经过一两个示范例子以后,学生们很容易找到应用,这些应用实质上就是把问题的抽象数学元素赋予具体的解释。当教师在进行讨论的教室里,把教室当作问题中的长方体,他自己就使用了这样一种具体的解释。一个笨拙的学生可能会提议计算食堂的对角线,而不是教室的对角线来作为一种应用。如果学生们自己提不出来更有想象力的内容,那么教师本人可以提出一个稍许不同的问题,例如:“给定长方体的长、宽、高,求中心到一角的距离”。
学生可以利用刚才解决的问题的结果,因为所求距离是对角线的一半。或者他们也可以利用引入适当的直角三角形的方法(后一种办法对于本例来说,是不那么显而易见的,并且多少有点笨拙)。
在这个应用例子之后,教师可以讨论长方体四个对角线和六个棱锥体的结构,这六个棱锥体的底是长方体的六个面、公共顶点是长方体的中心、而侧棱是长方体对角线的一半。当学生的几何想象力被充分激发以后,教师应当回到他的问题上来:你能把结果或方法用于某个其他问题吗?现在学生有机会找到更有趣的具体应用了,例如,下面就是一个:“在一个长21码、宽16码的建筑物的长方形平屋顶的中心要立一个高8码的旗杆。为了支撑这根旗杆,我们需要四根等长的拉线。规定四根拉线要离旗杆顶点为2码处的同一点开始,而另一端是建筑物顶部的四个角。问每根拉线有多长?”
学生可以采用上面已详细求解过的问题中所用方法,即在一个垂直平面上引入一个直角三角形而在水平平面上引入另一个三角形。或者他们也可以利用上面的结果:设想有一个长方体,其对角线x就是四根缆绳之一而它的边是
a=10.5,b=8,c=6
直接应用公式可求出x=14.5。
更多的例子可见“你能利用这个结果吗?”那一节。
15.不同的方法
我们对前面8、10、12、14几节所考虑的问题继续讨论一下。主要的工作,
即提出计划,已在第10节加以叙述。让我们观察教师用不同的方式来进行。 从与第10节相同之点出发,以后可以沿着稍许不同的路线提出下列各问题,
,你是否知道任何与此有关的问题?”
,你是否知道一个类比的问题?”
“你看,所提的问题是关于空间的图形,它与长方体的对角线有关。关于平面中的类比问题可能是什么?它应该与长方形的对角线有关”。
,平行四边形”。
即便非常迟钝和平凡、并且以前没有能力推测任何事物的学生,最后也会被迫对解题的思路至少作出微小的贡献。此外,如果学生确实比较迟钝,为了使学生有所准备,教师应该事先讨论平行四边形的类比问题,否则不能一下子就端出现在的这个长方体问题。然后,教师可以继续提问如下,
,这里有一个与你有关且已解决了的问题,你能利用它吗?”
“为了有可能利用它,你是否应当引入某个辅助元素?”
最后教师可以成功地向学生提出他所希望的概念。这就是把给定长方体的对角线想象为必须引入图中的一个合适的平行四边形的对角线(这个平行四边形是通过长方体和两个对边的平面的截面)。此概念本质上和前面(第10节)相同,但方法却不一样。在第10节是通过未知数来触及到学生的可用的知识的;
我们回想起一个以前已解决的问题是因为其未知数和当前提出的问题中的未知数相同。而在本节,是用类比的方法使学生触及到解题的概念。
16.教师提问的方法
在第8,10,12,14,15各节所阐述的提问方法主要是先从表中一般化的问题和建议开始,在需要时,逐步转向更特殊更具体的问题和建议,直到在学生的头脑中能引出一个回答为止。如果你必须帮助学生开拓某种思路,如果可能的话,从表中一个一般化的问题或建议重新开始提问,并在必要时再一次回到某个更特殊的问题,如此等等。
当然,这张表仅仅是这种类型的第一张表,看来对大多数简单情况是够用了。但无疑它还应该改进。重要的是,我们开始提的问题与建议应该简单、自然和一般化,同时表应当短。
建议必须简单而自然,否则就会太唐突。
如果我们想培养学生的能力而不是特殊技巧的话,那么建议必须一般化,
不仅可用于目前的问题,而且可用于各类问题。
表必须简短,使得在不同情况下,能够不矫揉造作地重复提问,从而有机会最终能为学生所掌握,并对培养思维习惯作出贡献。
为了培养学生的独立工作能力,必需逐步改为提出特殊的建议。
这种提问的方法不是一成不变的,幸好如此,因为在这类事情中,任何一成不变的、机械的、陈旧的程序必然很糟糕。我们允许有一定的灵活性,它允许采用各种办法(见第15节),它可以而且应该这样来实施,使得教师所提的问题可以由学生自已提出来。
如果有读者希望在他的班上试一试这里所提出的方法,他当然应该小心地进行,他应该仔细地研究第8节的例子和后面笫18、19、20节中的例子。他应当仔细地准备他打算讨论的例子,同时也考虑到各种不同的方法。他开始时应作少量试验,并逐渐摸索出他应如何掌握这个方法,学生如何学习这个方法并且需要多少时间。
17.好问题与坏问题
如果能很好地理解上节所提出的提问方法,则通过比较可以有助于判断某些建议的好坏,这些建议是为了帮助学生而可能提出来的。
回到原来在第10节开始时的情况,那时提问下列问题:你知道一个与此有关的问题吗?我们从帮助学生的最好意愿出发,不问这个问题,而改为提问:你能应用毕达哥拉斯定理吗?
我们的动机可能是极好的,但是这种提问却大概是最坏的。我们必须认识是在什么情况下提出这个问题的;然后我们会发现有一大堆反对意见反对这种类型的“帮助”。
(1)如果学生已接近于问题的解决,他可能理解问题的建议;但是如果他不是这样,他十分可能完全看不到问题的着眼点,因而在最需要帮助之处却得不到帮助。
(2)这建议的针对性太强了,即使学生能利用它解决当前的问题,对于将来的问题来说并没有学到什么。这种提问不是很有启发性的。
(3)即使学生理解这建议,他们也极少能理解教师怎么会想到提出这样一个问题,而学生他自己又怎样能想出这样一个问题呢?它看起来很不自然,很令人诧异,就好象变戏法耍魔术一样。它实在没有什么启发性。
对第10、15节中所描述的过程就提不出上述任何反对意见了。
更多的例子
18.一个作图题
在给定三角形中作一正方形。正方形的两个顶点在三角形的底边上,另二个顶点分别在三角形的另两边上。
,未知的是什么?”
,一个正方形”
,已如数据是什么?”
,一个给定的三角形,其它没有。”
,条件是什么?”
,正方形的四个角在三角形的边线上,两个在底上,其余两边每边上有一个。”
,是否可能满足条件?”
,我想如此,但不太有把握。”
,看起来,你解此题并不太容易。如果你不能解决所提问题,首先尝试去解决某个与此有关的问题。你能满足部分条件吗?”
,你说部分条件是什么意思?”
,你看,条件与正方形的所有顶点有关,这里有几个顶点?”
,四个。”
,所谓部分条件涉及的顶点数应当少于四个。请仅仅保持部分条件而舍去其余部分。什么样的部分条件容易满足?”
,两顶点在三角形边线上,甚至三个顶点都在三角形边线上的正方形,是容易画出来的!”
“画张图!”
学生画出图2。
图2
,你仅仅保留了部分条件,同时你舍去了其余条件。现在未知的确定到了什么程度?”
,如果正方形只有三个顶点在三角形的边线上,那么它是不确定的。”
,好!画张图。”
学生画出图3。
“正象你所说的,保持部分条件不能确定正方形、它会怎样变化呢?”
图3
“你的正方形的三个角在三角形的边线上,但第四个角还不在它应该在的地方。正象你说的,你的正方形是不确定的,它能变化;第四个角也是这样,
它怎样变化?”
……
“如果你希望的,你可以用实验的办法试试看。按照图中已有的两个正方形的相同办法,去画出更多的三个角在边线上的正方形。画出小的正方形与大的正方形。第四角的轨迹看起来象是什么?它将怎样变化?
教师已把学生带到非常接近于解答的地方。如果学生能猜到第四个角的轨迹是一条直线,他就得到这个主意了。
19.一个证明题
在不同平面上的两个角,其中一个角的每一边平行于另一角的对应边且方向相同。证明这两个角相等。
我们要证的是立体几何的一个基本定理。这个问题可以提给那些熟悉平面几何以及立体几何中下列少数事实的学生,这少数事实构成了欧几里得原理中当前这个定理的预备知识。我们不但把直接引自我们表中的问题与建议划上线,
而且把那些与它们相对应的问题与建议也划上线。例如,“求证题”是和“求解题”相对应的(在“求解题,求证题”标题下的第5,6小节中,我们再系统地讨论这种对应关系)。
,前提是什么?”
,两角在不同的平面上,其中一个的每一边平行于另一角的对应边,且方向相同。”
,结论是什么?”
,两角相等。”
“画张图,引入适当的符号。”
学生画出图4中的线,并在教师的或多或少的帮助下,标出图4中的字母。
,前提是什么?请用你的符号表达出来。”
“A,B,C和A',B',C'不在同一平面上,且AB∥A'B',AC∥A'C'。AB的方向与A'B'的方向相同,而AC的方向与 A'C'的方向相同。”
图4
,结论是什么?”
,看着结论! 尝试想起一个具有相同或相似结沦的熟悉的定理。”
,如果两个三角形全等,则对应角相等。”
,很好! 现在有一个与你的问题有关的定理,且早已证明。你能否利用它?”
,我想如此,不过我还不清楚怎么办。”
,为了可能利用它,你是否应该引入某个辅助元素?”
…… ……
,好,你提得非常好的那个定理是关于三角形的,是关于一对全等三角形的。在你的图中有没有三角形?”
,没有,但我能引进一些。让我连接B与C,B'与C',这样就有了两个三角形,ABC和A'B'C'。”
,做得好,但是这些三角形有什么用?”
,去证明结论;∠BAC=∠B’A’C’”
,好,如果你希望汪明这点,你需要两个什么样的三角形?”
“全等三角形。噢,对了,我可以选择B,C,B',C',使得
AB=A'B',AC=A'C'”
,好极了!现在你希望证明什么?”
,我希望证明两个三角形全等,
△ABC=△A'B'C'
如果我能证明这点,则立即可得结论∠BAC=∠B'A'C'。”
“妙!你有了一个新目标,这目标是一个新结论。看着这结论! 并且尝试想起一个具有相同或相似结论的熟悉的定理。”
“当且仅当一个三角形的三条边分别等于另一个三角形的三条边时,这两个三角形全等。”
图5
“做得好。本来你有可能会选出一条较差的定理的。现在这里有了一条与你的问题有关的定理,且早已证明,你能否利用它?”
,如果我知道BC=B'C',我能利用它。”
,对!那么你的目标是什么?”
,证明BC=B'C'。”
,试回忆起一个具有相同或相似结论的熟悉的定理。”
,是的,我知道一个定理,它最后结束的句子是:‘……则两线相等’,
但它并不合适。”
,为了能够利用它,你是否应该引入某个辅助元素?”
,你看,在图中BC与B'C'间并无联系,你怎么能证明 BC=B'C'?”
…… ……
,你利用了前提吗?前提是什么?”
,我们假定AB∥A'B',AC∥A'C'。是的,当然我们必须利用这点。”
,你是否利用了整个前提?你说AB∥A'B',这是你所知道的关于这些线段的全部情况吗?”
,不,根据作图,AB还等于A'B'。它们彼此平行并且相等。AC和A'C'也是这样。”
,两个等长的平行线——这是很有趣的图形。你以前见过吗?”
,当然见过!对!平行四边形!让我联结A与A',B与 B',C与C'。”
“这主意不太坏。现在你的图中有几个平行四边形?”
“两个。不,三个。不,两个。我意思是说,其中有两个,你可以立刻证明它们是平行四边形。还有第三个看来是个平行四边形。我希望我能证明它是。
这样证明就结束了!”
我们可能从这个学生前面的回答已经推测到他很聪明。但是等他作出上述最后一个回答以后,我们对此就深信不疑了。
这个学生能够猜出数学结果并且能够清楚地区分证明与猜测。他也知道猜测可以多多少少似乎是可信的。确实,他真的从数学课上得到了教益;他在解题方面有了某种实际经验,他可以看出并且摸索出一个好的解题思路。
20.一个速率问题
水以速率r流进锥形容器。容器具有正圆锥形状,底是水平的,顶点在下方,底的半径是a,高为b。当水深为y时,求水表面上升的速率。最后,假定a=4
尺,b=3尺,r=2立方尺/分,y=1尺,求未知数的数值。
图6
我们假定学生知道最简单的微分法和变化率的概念。
,已知数是什么?”
“圆锥底的半径a=4尺,圆锥的高b=3尺,水流入容器的速率r=2立方尺/分,
在某一时刻的水深y=1尺。”
“对,从问题的叙述方式看来,是建议你先忽略具体数值而用字母求解,
把未知数用a,b,r,y表示出来,而仅在最终得到未知数的字母表达式以后再代入具体数值。我愿意按照这条建议做。现在未知数是什么?”
,当水深为y时,水面升起的速率。”
,它是什么?你能用其他术语来说吗?”
,水深增加的速率。”
,它是什么?你能否再重新叙述得更不同些?”
,水深的变化率。”
,对,y的变化率。但什么是变化率?回到定义去。”
,函数的变化率是导数。”
,正确。现在y是函数吗?如前所述,我们不管y的具体数值。你能否想象y
是变化的?”
,是的,水深y随着时间而增加。”
,这样,y是什么的函数?”
,时间t的。”
,好,引入适当的记号。用数学符号,你将怎样写‘y的变化率’?”
,dy/dt”
,好,这就是你的未知数。你必须用a,b,r,y来表示它。顺便说一下,
数据中有一个是‘速率’,哪一个?”
,r是水流进容器的速率。”
,它是什么?你能用别的术语来说它吗?”
,r是容器中水的体积的变化率。”
,它是什么?你能否再重新叙述得更不同些?你将怎样用适当的记号来写它?”
,r=dV/dt”
,V是什么?”
,在时间t,容器中水的体积”
,好,这样你必须用a,b,dV/dt,y,来表示dy/dt,你将怎样做?”
…… ……
,如果你不能解决所提问题,首先尝试去解决某个与此有关的问题。如果你到现在还看不出dy/dt与数据间的联系;尝试去引入某种能作为中间过渡踏脚石的更简单的联系。”
,你看不出还有别的联系吗?例如y与V是否彼此独立?”
“不,当y增加,V一定也增加。”
“那么说,是有联系了,这联系是什么?”
,好,V是锥体的体积,y是锥体的高。但我现在还不知道底的半径。”
,不过,你可以考虑它。叫它什么,譬如设它为x吧!”
,V= 3
2 yxp
。”
,正确,关于x又知道些什么?它是否与y独立?”
,不,当水深y增加,自由表面的半径x也增加。”
,这么说,它们之间是有联系的。但这联系是什么?”
,当然是相似三角形。x:y=a:b”
,你看,又多了个联系,我不愿错过从它那儿得到的好处。别忘了,你希望知道的是y与V之间的联系。”
,现在我有x=ay/b
V= 2
32
3b
yap,
,很好,这看来像个踏脚点。难道不是吗?但你别忠了你的目标。未知数是什么?”
,噢,是dy/dt”。
,你必须找出dy/dt,dV/dt与其他数量间的联系。但这里有的却是y,V和其他数量间的联系。你该怎么办?”
,当然是微分!
tyb yatV= 2
22p
就是它。”
,妙!那么从已经给出的数值能得出什么结果呢?”
,若a=4,b=3,dV/dt=r=2,y=l,则
ty××= 9 1162 p 。”也即
dy/dt=0.358尺/分。
第二部分怎样解题——一段对话
1.熟悉问题
我应该从哪儿开始?从问题的叙述开始。
我能做什么?观察揣摩整个问题,尽量使其清晰而鲜明。暂时先抛开细节。
这样做,我能得到什么好处?你会明白问题,使自己熟悉问题,并把问题的目标牢记在脑海中。这样全神贯注地对待问题也会调动起你的记忆力,做好准备去重新联想与问题有关的各点。
2.深入理解问题
我应该从哪儿开始?还是从问题的叙述开始。当你对问题的叙述已如此清楚并已深深地印入脑海,以致你即使暂时不去看它,你也不怕把它完全忘掉时,
你就可以开始下面的工作了。
我能做什么?先把问题的主要部分剖析出来。因为前提与结论是“求证题”
的主要部分。未知、已知与条件是“求解题”的主要部分。再把问题中的主要部分都弄一遍,并且要逐个地考虑,轮流地考虑,而且在各种组合中来考虑,
同时把每个细节与其它细节联系起来,把每个细节与整个问题联系起来。
这么做,我能得到什么好处?你会准备好并弄清楚以后可能起作用的细节。
3.探索有益的念头
应该从哪儿开始?从考虑问题的主要部分开始。当主要部分能很清楚地排列出来,想得明明白白(这应归功于你前面的工作)并且也记得住时,这时开始做下一步。
怎样进行?从各个方面来考虑你的问题,找出与你现有知识有关之处。
从各个方面考虑你的问题。分别突出各个部分,考察各个细节,用不同方法反复审查同一细节。把细节用不同方式组合起来,从不同角度考虑它。试着在每一细节中发现某些新意义,尝试在整个问题中得出某些新解释。
从你现有知识中找出与问题有关之处。试想过去在类似的情况下有什么曾帮过你的忙。在你所考察的内容中,设法找出熟悉的东西来,在你所熟悉的东西中,努力找出有用的东西来。
能找出什么?一个有用的念头,也许是个决定性的念头,它能使你一限看出解决问题的途径。
念头有什么用?它会给你指出整个或部分解题途径,它或多或少地清楚地向你建议该怎么做。念头多多少少还是完整的。如果你有一个念头,你就够幸运的了。
碰上一个不完整的念头怎么办?应该加以考虑。如果它看来有好处,就应该多考虑一会儿。如果它看来是可靠的,你应当确定它能引导你走多远,并重新考虑一下形势。由于这个有益的念头,情况已经变化了。你要从各个方面来考虑新形势并找出它与你现有知识之间的联系。
再次这样做,还能得到什么好处?如果你走运的话,你或许能找到另一个念头。也许下一个念头会引导你去解决问题。也许在下一个念头以后,你还需要几个有益的念头。也许有些念头会把你引入歧途。无论如何,你应当感谢所有的新念头,感谢那些次要的念头,感谢那些模糊的念头,也感谢那些使模糊念头得以纠正的补充性念头。即使你暂时还没有发现什么有价值的新念头,但如果你对问题的概念更完全了,或者更连贯、更和谐或者更平衡了,那你也应当表示感谢。
4.实现计划
应该从哪儿开始?从引导到解决问题的思路开始。当你感到你已抓住主要的联系,并且自信能提供可能需要的次要细节时,就开始。
怎幺做?你对问题应抓得很有把握。详细地进行你以前认为可行的全部代数或几何运算。用形式推理或直接观察检查每一步骤的正确性,或者,如果你能够的话,两种方法都用。如果你的问题很复杂,你可以分成“大”步骤和“小”
步骤,每一大步骤又由几个小步骤组成。首先检查大步骤,以后再检验小步骤。
这样做,我能有什么好处?这样提出的解,每个步骤无疑都是正确的。
5.回顾
应该从哪儿开始?从解答开始,它的每一个细节都应该是完整而正确的,
怎么做?从各个方面考虑这个解,找出与你已有知识之间的联系。
考虑解的细节,并尝试使它们尽可能地简单;研究解答中较冗长的部分,
使它们更短些;试着一眼就看出整个解。试着去改进解的各部分,尝试去改进整个解,使它直观,使它尽量自然地适合于你已有的知识。总结你解题的方法,
尝试看出它的要点,并且尝试把它用于其他问题。总结所得结果并试着把它用于其他的问题。
这样做,我能有什么好处?你可能找出一个新的更好的解,你可能发现新的有趣的事实。无论如何,如果你用这方式养成研究与总结你的解的习惯,你将获得某些井然有序的,便于应用的知识,并且你将会提高你解题的能力。
第三部分 探索法小词典
1.类比
类比就是一种相似。相似的对象在某个方面彼此一致,类比的对象则其相应部分在臬些关系上相似,
(1)长方形可与长方体类比。事实上,长方形各边之间的关系与长方体各面之间的关系相似,
长方形的每一边恰与另一边平行,而与其余的边垂直。
长方体的每一面恰与另一面平行,而与其余的面垂直。
让我们把边称为长方形的边界元素,而面称为长方体的边界元素,则前述两个命题可合而为一并可同等地应用于这两个图形,
每一边界元素恰与另一边界元素平行,而与其余的边界元素垂直。
这样,我们就将所比较的两个系统的对象(即长方形的边与长方体的面)的某些共同关系表达出来了。这两个系统的类比存在于关系的共性之中。
(2)在我们的思维、日常谈话、一般结论以及艺术表演方法和最高科学成就中无不充满了类比。类比可在不同的水平使用。人们常常使用含糊不清的,
夸大的,不完全的或没有完全弄清楚的类比,但类比也可以达到数学精确性的水平。所有各种类比在发现解答方面都可能起作用,所以我们不应当忽略任何一种。
(3)在求解一个问题时,如果能成功地发现一个此较简单的类比问题,我们会认为自己运气不错。在第十五节,我们原来的问题是长方体的对角线,它的较简单的类比问题就是长方形的对角线,这个类比问题引导我们到达原问题的解答。我们将讨论这种类型的另一个例子。我们需要求解下列问题,
求均匀四面体的重心。
若不具备积分与物理知识,这问题是很困难的。在阿基米德与伽里略的时代,它是一个严肃的科学问题。因此,如果我们希望用尽可能少的预备知识来解决它,我们就应该寻求一个较为简单的类比问题。在平面上的对应问题很自然地就是下面的问题,
求一均匀三角形的重心。
现在,我们有了两个问题而不是一个问题。但两个问题比起一个问题来可能还更容易回答——假定这两个问题能巧妙地联系起来的话。
(4)现在我们暂时把原来四面体的问题放在一边,而把注意力集中在有关三角形这一比较简单的类比问题上。为了求解这个问题,我们必须了解一些关于重心的知识。下列原理似乎是可信的而且提出它来也很自然,
若一物质系统S由几部分组成,每一部分的重心都位于同一平面上,则该平面也必包含此整个系统S的重心。
对于三角形情况来说,这一原理给出我们所需要的一切。首先,它指出三角形的重心位于三角形的平面上。于是,我们可以把三角形看成由平行于三角形某边(图7中边AB)的许多个小条条(薄条条无限窄的平行四边形)所组成。每一个小条条(平行四边形)的重心显然是它的中心,而所有这些中心位于连线CM上,
C为与AB边相对的顶点,M为AB的中点(见图7)。
图7
通过三角形中线CM的任何平面包含有三角形中所有平行小条条的重心。由此得出结论:整个三角形的重心就在这一中线上。但是,根据同一理由它也必须在其他二条中线上,所以它必须是所有三根中线的公共交点。
我们现在希望用纯几何方法(与任何力学上的假设无关)来证明三根中线交于同一点。
(5)在弄懂了三角形的例子之后,四面体的情况就相当容易了。因为我们现在已经解决了一个和我们所提问题有类比关系的问题,所以一旦解决这个类比问题,我们就有了一个可以照着办的模型。
在解决我们现在用作模型的类比问题中,我们设想三角形是由平行于其一边AB的平行小条条所组成的。现在我们设想四面体ABCD也由平行于其一棱AB的小条条所组成。
组成三角形的小条条之中点全部位于同一直线上,即位于连接边AB的中点
M与相对顶点C的那根三角形中线上。组成四面体的小条条的中点全部位于连接棱AB的中点M与对棱CD(见图8)的同一平面上;我们不妨将此平面MCD称为四面体的中面。
图8
在三角形情况下,我们有象MC那样的三根中线,其中每一根都必须包含三角形的重心。因此,这三根中线必须交于一点,这一点就是重心。在四面体情况下,我们有象MCD那样的六个中面(连接一条棱中点与其对棱的平面),其中每个中面都必定包含四面体的重心。因此,这六个中面必交于一点,这一点就是重心。
(6)这样,我们就解决了均匀四面体的重心问题。为了完成这个求解过程,
现在我们需要用纯几何(与力学上的考虑无关)来证明六个中面通过同一点。
当我们解决了均匀三角形的重心问题以后,我们发现,为了完成求解过程,
需要证明三角形的三条中线通过同一点。这个问题可类比于上述问题,但显然较为简单。
在解决四面体这一问题时,我们又可利用较简单的三角形类比问题(这里,
我们假定它已经解决了)。事实上,我们考虑通过从D点出发的三条棱DA,DB,
DC的三个中面;每一中面同时也通过对棱的中点(通过DC边的中面经过中点M,
见图8)。现在,这三个中面和△ABC所在平面交于该三角形的三个中线。这三条中线交于一点(这是前面较简单的类比问题的结果),而这点和D点一样,也是三中面的公共点。连结这二个公共点的直线是所有三个中面的公共线。
我们证明了六个中面中通过顶点D的三个中面有一条公共直线。对于通过顶点A的三个中面,同样也成立;对于经过顶点B的三中面以及经过顶点C的三中面也是如此。把这些事实适当地联系起来,我们就可证明这六个中面有一个公共点(通过△ABC三边的三中面确定一公共点和交于此点的三交线。于是,根据我们刚才证明的,通过每一交线,一定还有一个中面)。
(7)在上述(5)和(6)中,我们都利用了一个三角形的较为简单的类比问题去解决四面体问题。但从一个重要方面来看,(5)和(6)两种情况是不相同的。
在(5)中,我们是利用较简单的类比问题这一方法,逐点模仿它来求解。但在(6)
中,我们则是利用了较简单的类比问题所得的结果,我们并不关心这结果是怎样得到的。有时,我们可能同时利用较简单的类比问题的方法及其结果。如果我们把上述(5)和(6)看成是同一个问题求解的两个不同部分,则上述例子就是同时利用类比问题的方法及结果的。
我们这个例子是典型的。在求解所提出的问题的过程中,我们经常可以利用一个较简单的类比问题的解答;我们可能利用它的方法或者可能利用它的结果,或者可能三者同时利用。当然,在更困难的问题中,可能会出现我们这个例子中尚未出现过的复杂情况。特别是,可能发生下述情况:类比问题的解不能直接用于我们原来的问题上。那时,可能需要我们去重新考虑解答,去改变它,修改它,直到我们在试过解答的各种形式以后,终于找到一个可拓广到我们原来的问题为止。
(8)我们希望能预测结果,或者,至少在某种似乎可信的程度上预测到结果的某些特征。这种似乎可信的预测通常是以类比为基础的。
这样,我们可能知道,均匀三角形的重心及其三个顶点的重心重合(即,
三个质量相同的质点放在三角形的三个顶点上)。了解这点以后,我们可以猜测均匀四面体的重心与其四个顶点的重心相重合。
这种猜测是一种“类比推论”。已知三角形和四面体在许多方面相似,我们就猜测它们在其他某一个方面也是相似的。如果把这种猜测的似真性当作肯定性,那将是愚蠢的。但是忽视这种似真的猜测将是同样愚蠢甚至更为愚蠢的。
类比推论看来是最普通的一种推论,并且可能是最主要的一种。它产生了多少似乎可信的推测,这种推测可能被经验和更严格的论证加以证实或推翻。
为了预测药物对人类的影响,化学家在动物身上进行试验,再由类比得出结论。
甚至我认识的一个小男孩也这么做。他的小狗需要到兽医那儿去医疗,于是他问,
,谁是兽医”
,动物的医生。”
“哪种动物是动物的医生?”
(9)得自许多类似情况的类比结论比得自较少情况的类比结论要强。但是这里质量仍然比数量更为重要。清晰的类比较模糊的相似更有价值,安排有序的例子比随意收集的情况更能说明问题。
前面【上述(8)】我们提出了一个关于四面体重心的猜测。这猜测就是根据类比而提出的;四面体的情况类比于三角形的情况。通过考察另一个类比的例子,均匀棒(即均匀密度的直线段)的例子,我们可以增加对猜测的认识。
存在于
线段 三角形 四面体
之间的类比有许多方面。线段包含在直线上,三角形在平面上,四面体在空间中。直线段是最简单的一维有界图形,三角形是最简单的多边形,四面体是最简单的多面体。
线段有两个零维边界元素(2端点)而其内部是一维的。
三角形有三个零维及三个一维边界元素(三顶点,三边),而其内部是二维的。
四面体有四个零维,六个一维,四个二维边界元素(四顶点,六边,四面),
而其内部是三维的。
这些数字可以列成一个表如下,其中各列分别表示零维,一维,二维与三维元素的数目,各行分别表示线段,三角形与四面体的数目,
零维 一维 二维 三维
线段 2 1
三角形 3 3 1
四面体 4 6 4 1
只须对二项式展开有稍许的了解,便可认出这些数字是巴斯卡三角形中的一部分。我们在线段、三角形和四面体中找出了一个值得注意的规则性。
(10)如果我们已经体会到我们所比较的对象是有密切联系的,则下列“类比推论”对于我们可能有某些价值。
均匀捧的重心与其两端点的重心相重合。均匀三角形的重心与其三顶点的重心相重合。为什么我们不应该设想均匀四面体的重心与其四顶点的重心相重合呢?
还有,均匀捧的重心按比例1:1来划分其端点间的距离。均匀三角形的重心按比例2:1来划分任何顶点与其对边中点间的距离。为什么我们不应该猜测均匀四面体的重心是按比例3:1来划分任何顶点与其对面的重心间的距离呢?
说上述问题所提出的猜测是错误的,说这样美妙的一种规律性竟遭破坏,
这点总叫人觉得极不可能。认为和谐的简单秩序不会骗人这样一种感觉,在数学及其他科学领域中指引着作出发现的人们,并表达为拉丁格言,
简单是真理的标志
[从上面讲的会想到,所讨论的结果可推广到n维。如果对前三维(n=1、2、
3)成立而对维数高的n就不再成立,这看来不大可能。这种猜测是一种“归纳推论”;它表明归纳很自然地以类比为基础。参见“归纳与数学归纳法”一节。
[(11)在结束本节以前,我们简单地考虑一类最重要的情况:在这类情况下,类比这一数学概念变得更精确了]。
(I)两个数学对象系统设为S和S',是这样相互联系的,S的对象之间的某些关系和S'的对象之间的某些关系遵循同一法则。
在S和S'间的这种类比可以用上述(1)中所讨论的内容为例说明之;把长方形的边作为S,把长方体的面作为S'。
(II)在两个系统S与S'的对象之间存在一一对应,即保持某种关系。也就是说,如果一个系统的对象之间保持这样一种关系,则在另一系统的对应对象之间也保持同一关系。在两个系统中的这种联系是一种非常精确的类比;它称为同构。
(III)在两个系统S与S’的对象之间存在一对多的对应而保持某种关系(这在高等数学研究的各分支中,特别在群论中很重要,这里不多赘言)。这种情况称为同态。同态也可看成另一种非常精确的类比。
2.辅助元素
我们对问题的概念在我们工作结束时远比我们开始工作时丰富得多(见
“进展与成就”一节)。随着工作的进展,我们在原有考虑之外,增加一些新元素。旨在促进求解而引入的这种元素称为辅助元素。
(1)有各种辅助元素。解决几何问题,我们可能在图中引入新的线,即辅助线。解决代数问题,我们可能引人辅助未知数【见“辅助问题”,(1)】。辅助定理是这样一种定理,我们证明它是希望促进我们对原来问题的求解。
(2)引入辅助元素有各种理由。当我们想到一个与我们现在的问题有关、
且早已解决的问题时,我们很高兴。很可能我们能利用这样一个问题,但目前还不知道怎么利用它。例如,我们试图求解的是一个几何问题,而我们想到的早已解决的有关问题是三角形问题。但在我们的图中并没有三角形;为了有可能利用所想的问题,我们必须找到一个三角形;所以我们不得不在我们图中用添加适当的辅助线的办法引入一个三角形。一般说来,当我们想到一个早已解决的有关问题后,我们必须经常问:为了可能利用它,我们是否应该引入某个辅助元素?(第10节中的例子是典型的)。
回到定义去,我们将有另一个引进辅助元素的机会。例如,说明圆的定义时,我们不仅应该提到其圆心和半径,而且还应该把这些几何元素在图中表示出来。如果不把它们表示出来,我们就不能对定义有任何具体的应用;叙述定义而不作图只不过是空口说白话罢了。
力图利用已知结果和回到定义去,是引入辅助元素的一些最好的理由;但它们不是仅有的理由。为了使问题的概念更完整,更富于启发性,更为人所熟悉,我们可以引入辅助元素,虽然目前我们几乎不知道我们怎样才能利用这些所添加的元素。我们可能仅仅感觉到加上这样那样的元素用那种方式看问题是个“好念头”。
引入辅助元素可以出自这种理由,也可以来自别的理由,但总得有些理由。
我们不能随随便便地引入辅助元素。
(3)例子。已知三角形一角和由此角顶点向对边所作的高和三角形的周长,
作这个三角形。
我们引入适当的记号。令已知角为a,从角a的顶点A向对边所作的高为h,
已知周长为P。我们画张图,在上面很容易标上a与h。我们是否已利用了所有的数据?没有!我们的图并未包括等于三角形周长的已知长度P。因此,我们必须引入P。但是怎样做呢?
我们可以用各种方式来试图引入P。图9,10所表示的方式看起来很笨拙。
如果我们自己琢磨一下为什么这两张图看来如此令人不称心,我们就可能看出是由于缺乏对称性的缘故。
事实上,这个三角形有三条未知边:a,b,c。我们象通常所做的那样,
把A的对边叫做a,其余两边则相应地称为b与c。我们知道
a+b+c=P
这里,边b与边c的作用相同;它们是可交换的;我们的问题对于b和c是对称的。
但在图9和图10中,b和c的作用却不相同;放上长度P,我们对待b和c就不同了;
图9和图10破坏了问题对b和c的自然对称性。我们应该这样来放置P,使得它和b
和c的关系是对称的。
图9
图10
上述考虑可能有助于建议象图11那样放置长度P。我们在三角形的边a的一侧,加上线段CE,其长为b,在三角形的另一侧加上线段BD,其长为C,使得在图11中线段 ED的长度恰好是P,即,
b+a+c=P
如果我们对怍图题有些经验,我们就不会忘记和ED一起引入辅助线AD与AE。它们都是等腰三角形的底边。事实上,在问题中引入象等腰三角形这样简单而又为人熟悉的元素是合理的。
图11
迄今,我们在引入辅助线方面一直是十分幸运的。我们观察新图,就可以发现∠EAD和已知角a有一简单关系。事实上,利用等腰三角形△ABD与△ACE,
可知∠DAE=α/2+90°知道这个特点以后,我们很自然地会去作出△DAE。作此三角形时,我们就要引入一个辅助问题,它远比原来的问题简单。
(4)教师与教科书的作者不应当忘记:聪明的学生和聪明的读者不会满足于验证推理过程的每一步是正确的,他们还要求知道进行各一步的动机和目的。
引入辅助元素是引入注目的一步。如果一条微妙的辅助线在图中出现得很突然,
看不出任何动机,并且令人惊讶地解决了问题,那末聪明的读者和学生将会失望,他们感到上当受骗。因为只有在我们的论证及发明会创造的能力中充分发挥了数学的作用后,数学才是有趣味的。如果最引人注目的步骤的动机和目的不可理解,那么我们在论证和发明创造方面就学不到什么东西。为使这样的步骤可以理解,需要加以适当的说明(如前面(3)中所做的那样),或者精选问题和建议(象第lO、18、19、20节中所做的那样),这需要大量的时间和精力,但却是值得一做的。
3.辅助问题
辅助问题是这样一个问题,我们考虑它并非为了它本身,而是因为我们希望通过它帮助我们去解决另一个问题,即我们原来的问题。原来的问题是我们要达到的目的,而辅助问题只是我们试图达到目的的手段。
一只飞虫企图穿过窗户玻璃逃出去,它在同一扇窗户上试了又试,而不去试试附近打开的窗户,而那扇窗户就是它进来的那扇。人能够或者至少能够行动得更聪明些。人的高明之处就在于当他碰到一个不能直接克服的障碍时,他会绕过去;当原来的问题看起来似乎不好解时,就想出一个合适的辅助问题。
构想一个辅助问题是一项重要的思维活动。举出一个有助于另一问题的清晰的新问题,能够清楚地把达到另一目标的手段设想成一个新目标,这都是运用智慧的卓越成就。学会(或教会)怎样聪明地处理辅助问题是一项重大任务。
(1)例子。求满足方程的x值,
x4-13x2+36=0
如果我们看到x4=(x2)2。我们就会发现引入
y=x2
的好处。我们现在得到一个新问题:求满足方程的y值,
y2-13*y+36=O。
这个新问题是一个辅助问题;我们打算把它用作解决原问题的手段。辅助问题的未知数y可恰如其份地称为辅助未知数。
(2)例子。在一长方体中已知由一顶点引出的三个棱的长度,求该长方体的对角线。
在试图求解这一问题(第8节)时,我们可由类比(第15节)引导到另一问题:
在一长方形中,已知由同一顶点引出的两个边的长度,求长方形的对角线。
这个新问题是个辅助问题:我们之所以考虑它是因为我们希望从对它的考虑中引出对原问题有用的东西。
(3)好处。考虑辅助问题的好处可以是多种多样的。我们可以利用辅助问题的结果。譬如在例1中,通过求解y的二次方程,我们已经求得y等于4或等于9,
然后我们推得 x2=4或x2=9,从而求出x的所有可取的值。在其它情况下,我们可以利用辅助问题的方法。如例2中,辅助问题是平面几何问题;它类比于原来的立体几何问题,但更为简单。引入这一类辅助问题是合理的,因为我们希望它是有启发性的,它能给我们机会去熟悉以后可用于原问题的某些方法、操作或工具。在例2中,辅助问题的选择更为幸运,因为仔细地考察它一番之后,我们发现其方法与结果均可加以利用(见第15节和“你是否利用了所有的数据?”
那一节)。
(4)风险。我们不去考虑原问题,而花费时间与精力去注意辅助问题。如果我们对辅助问题的研究失败了,那末我们在它上面所花的时间与精力就白白损失了。所以在选择辅助问题时,我们应当加以判断。对于我们的选择,我们可能有各种正当理由。辅助问题可以比原来的问题更容易理解;或者它看来更富启发性;或者它有某种美的号召力。有时,辅助问题的唯一优点是它很新颖,
提供了尚未被探索过的可能性;我们选择它是因为我们对原问题厌倦了,并且看来似乎所有的方法部已用尽了。
(5)怎样找出它。发现所提问题的解,常常有赖于发现一个合适的辅助问题。令人不愉快的是,没有万灵的方法来发现合适的辅助问题,正如没有万灵的方法求解一样。但无论如何,确实有一些问题和建议,它们常常是有所裨益的。例如,看着未知数。通过问题的变化常常会使我们想到有用的辅助问题。
(6)等价问题。如果两个问题中每一问题的解都蕴含另一问题的解,就说这两个问题是等价的。因此,在例1中,原问题与辅助问题等价。
考虑下列定理,
A.在任何等边三角形中,每一角均等于60°。
B.在任何等角三角形中,每一角均等于60°。
此二定理不能看作是同一条定理。它们包含不同的概念:一个与边的相等有关,另一个与三角形的角相等有关。但每一定理都可由另一定理得出。因此求证题A与求证题B等价。
如果我们需要求证A,则引入求证题B作为一个辅助问题是有某些好处的。
定理B的证明要比证明A容易些,而且更重要的是,我们可以预见到B比A容易;
我们可以这样判断,我们可能从一开始就发现B很可能比A容易。事实上,定理B
仅与角有关,它比定理A更“单一”,定理A与角和边都有关。
如果原问题和辅助问题是等价的,则从原问题过渡到辅助问题称为可逆化归,或双向化归,或等价化归。例如,A化归为B(见上文)是可逆的,例1中的化归也如此。从某个方面说来,可逆化归比其它引入辅助问题的方法更重要,更令人想往,但是那些和原问题不等价的辅助问题可能也很有用;见例2。
(7)等价辅助问题链。等价辅助问题链在数学论证中是屡见不鲜的。我们需要解决问题A;我们看不出解答,但我们可能发现A与另一问题等价。考虑B
时,我们又可能涉及与B等价的第三个问题C。照这样下去,我们又可将C化为 D,
如此等等,直到最后得到问题L,其解答为已知或明显可知。既然每一个问题都和前一个问题等价,则最后一个问题也必定和原问题A等价。于是我们能够从问题L推出原问题 A的解答,而L是辅助问题链的最后一个环节。
这种问题链,正如我们从帕扑斯的重要章节中所见,早已为希腊数学家们所注意。我们重新考虑例1作为说明。让我们称(A)为未知数x的条件,
(A) x4-13x2+36=O
解决这个问题的一种方法是将所提出条件变换成另一个条件,称为(B),
(B) (2x2)2-2(2x2)·13+144=O
我们观察到条件(A)与条件(B)不同。如果你愿意,你可以说它们仅仅稍许有些不同。你会很容易相信它们一定等价,但它们肯定不是同一个方程。从(A)过渡到(B)不仅正确,而且有清楚的目的,这对任何熟悉求解二次方程的人来说都是显而易见的。沿此一方向继续做下去,我们可将条件(B)再变换成另一条件(C),
(C) (2x2)2-2(2x2)·13+169=25
照此方法继续下去,我们有
(D) (2x2-13)2=25
(E) 2x2-13=±5
(F) x2=(13±5)/2
(G) 2 513 ±±=x
(H) x=3或-3,或2,或-2我们所做的每次化归都是可逆的。于是最后一个条件(H)与第一个条件(A)等价,所以,3、-3、2、-2是我们原问题所有可能的解。
上面我们从原条件(A)导出一系列条件(B),(C),(D),……,每一个都等价于前一个。这一点值得我们给予最大的注意。等价条件是由同一对象满足的。
因此,如果我们从所提条件过渡到等价于它的新条件,我们就有相同的解。但是如果我们从所提条件过渡到较窄的条件,我们就失去解;如果我们从所提条件过渡到较宽的条件,我们则得到非正常的外来解,它与所提问题无关。如果在一串连续的化归中,我们过渡到较窄的,接着又过渡到较宽的条件,我们可能完全偏离原来的问题。为了避免这种危险,我们必须小心地检查每次新引入的条件的性质:它与原条件等价吗? 当我们所处理的对象不是像这里的单个方程而是一组方程时,或者当条件不是用方程来表达(例如,象几何作图问题)时,
上述问题尤为重要。
[请与“帕扑斯”一节,特别是评注(2),(3),(4),(8)相比较。那里的描述受到了不必要的限制。它描述一个求解问题的链,其中每个问题都有一个不同的未知数。这里所讲的例子则相反,链中所有各个未知数相同,仅仅是条件的形式不同。当然,并不需要这种限制。]
(8)单向化归。我们有两个都未曾求解的问题A与B。如果我们能解A,则我们能导出B的完全解。反之则不然;即,如果我们能解B,我们可能会得到A的某些信息,但我们却不知道怎样从B导出A的完全解。在这样一种情况下,解 A要比解B收获大。让我们称A为这两个问题中的期望大的而B为期望小的。
如果从所提问题过渡到期望大的或期望小的辅助问题,我们称这一步骤为单向化归。有两类单向化归,二者在某些方面都比双向或可逆化归更冒风险。
例2说明的是化归为期望小的问题的一个单向化归。事实上,如果我们能够解决属于长、宽、高分别为a,b,c的长方体的原问题,令c=O,得到长为a,
宽为b的长方形,则我们就转到辅助问题。化沟期望小的问题的单向化归的另一例子是“特殊化”这一节的(3),(4),(5)。这些例子表明,有时凑巧,我们可能利用期望小的问题作为踏脚石,将辅助问题的解加上适当的补充说明,可以得到原问题的解。
化为期望大的问题的单向化归也可能会成功(见“普遍化”这一节(2)及“归纳与数学归纳法”这一节(1),(2)中所述第一问题化为第二问题的例子)。事实上,期望人的问题可能更容易着手;这就是“发明者的矛盾”。
4.波尔查诺(Bolzano)
他是逻辑学家与数学家。在他逻辑学的综合性著作:《科学沦》中,有相当大一部分是关于探索法这一题目的(第三卷,293—575页)。在他著作的这一部分,他写道:“我根本不认为我在这里能够提出任何早先未曾为所有具有才华的人所察看出的研究过程;并且我也根本不想允诺你们可以从我这里发现这方面的很新颖的任何内容。但是,我将煞费苦心地用清晰的词句来说明所有有才能的人所遵循的研究规则与方法,这些有才华的人在大多数情况下,甚至不知道他们自己是遵循这些规则与方法的。虽然,即使正在做这件事的时候,我也不敢幻想我将会完全成功,但我仍然希望这里所提出的一孔之见会博得某些知音并在以后有所应用。”
5.好念头
这是对解答突然有进展的一种口语描述[参见“进展与成就”(6)]。好念头的出现,每个人都体验过,但只能心领神会而难于言传,所以提一提像亚里士多德这样古老的权威曾经偶然给过一个很有启发性的描述,可能会使人感到兴趣。
大多数人会同意:想出一个好念头是一种“灵感活动”。亚里士多德对“灵感”所作的定义如下:“灵感就是在微不足道的时间里,通过猜测而抓住事物本质的联系。”例如说:“如果你看见一个人以某种方式和一个富翁谈话,你可能立刻猜想此人正在设法借钱。又如,观察到月亮发光的一边总是朝着太阳,
你可能会突然想到为什么会这样:这是因为月亮是由太阳光照亮的。”
第一个例子并不坏,但太庸俗了;关于富翁和钱这类事情不需要多少灵感来加以推测,并且那个念头也并不怎么高明。但第二个例子却给人以深刻的印象,如果我们发挥一点想象力把它联系其适当的背景来看的话。
我们应当认识到,在亚里士多德的时代,因为没有钟表,如果他想知道时间,他必须观察太阳和星星;因为没有路灯,如果他计划在夜间旅行,他必须观察月亮的月相。他比现代的城市居民对天空熟悉得多,并且他的天生的智慧未被天文学理论的报刊出版物的生搬硬套的片言只语所蒙蔽。他把整个月亮看成是一只平盘,和太阳这只平盘相类似,但光辉暗淡多了。他一定曾经奇怪过月亮的形状和位置为什么老是在变化。他偶然也在白天观察过太阳,大约是在日落或日出的时候,并且终于发现“月亮发亮的一边总是朝着太阳”,这本身就是一个令人可敬可佩的成就。于是,现在他看出月亮变化着的外表好象一个从一侧照亮的球,所以它一半亮,另一半黑。他不再想象太阳和月亮是平盘了,
而把它们想象或球状,一个发光,另一个受光。他理解本质的联系,他“在微不足道的时间里”突然改变了他以前的概念:想象力有了一个突然的跳跃,产生了一个好念头,这是天才的一次闪烁。
6.你能检验这结果吗?
你能检验这结果吗?你能检验这论证吗?对这些问题若能给出很好的回答,
将加强我们对答案的信任并巩固我们的知识。
(1)数学问题的数字结果可以这样来检验,把它们与观测值或可观测数字在常识上的估计值相比较。由于产生于实际需要或天生好奇心的问题几乎总是以事实为基础的,所以可以预期这种与可观测事实作比较的步骤一般不能省略。
但是每一个教师都知道学生在这方面能做出不可思议的事来。有些学生求出船的长度为16,130英尺,船长的年龄为8岁零二个月,顺便说一下,这位船长已经是一位祖父,对于这样一件事,他们也会坚信不疑,泰然自若。如此不顾明显的谬误并不一定说明他们愚蠢,而只不过对人为编造的问题漠不关心罢了。
(2)“字母题”比“数字题”更容易接受有趣的检验(见第14节)。作为另外一个例子,我们考虑底为正方形的棱台。设下底边长为a,上底边长为b,高为h,则其体积为
(a2+ab+b2)h/3
我们可用“特殊化”一节所讲的方法检验这结果。事实上,若a=b,则棱台成为棱柱,公式成为a2h;若b=O,则棱台成为角锥体,公式成为a2h/3。我们还可用“量纲检验法”。事实上,公式的量纲是长度的立方。还有,我们可用数据的变化来检验公式;事实上,若正数值a,b或h中的任一个增大,则公式的数值也增大。
这类检验不仅可用于最后结果,也可用于中间结果。它们是如此有用,值得讨论,参见“问题的变化”这一节第(4)点。为了能利用这种检验,我们可能发现把“数字题”加以普遍化并变为“字母题”是有好处的,参见“普遍化”
这一节第(3)点。
(3)你能检验这论证吗?在逐步检验论证时,我们应当避免单纯的重复。首先,单纯的重复容易令人厌烦,缺乏启发性,使人注意力涣散。其次,在我们曾经跌过一次跤的地方,如果环境与从前一样,我们可能再次跌跤。如果我们感到需要把整个论证重新逐步检查一遍,我们至少应当改变各步的次序,或者改变它们的分组,以引入某些变化。
(4)排出论证中最薄弱的环节并首先加以审查,这只需要较少的劳力,而且更有兴趣。在挑出论证中值得审查之点时,一个很有用的问题是:你曾否利用了所有的数据?
(5)很清楚,我们非数学方面的知识不能完全奠基在形式逻辑的证明上。
我们日常知识的较可靠的部分是不断被我们每天的经验所检验,所加强的。在自然科学中,这种检验采取了细心试验与测量的形式,并与数学论证结合在一起。我们的数学知识能否只以形式逻辑的证明为基础呢?
这是个哲学问题。我们不能在这里辩论。但肯定的是,你的数学知识,我的数学知识,或者你的学生的数学知识,都不是仅仅以形式逻辑证明为基础的。
任何可靠的知识,必有深厚的实验基础,而且通过每个已成功地检验其结果的问题使这种基础更加坚实。
7.你能用不同方式导出这一结果吗?
当最终所得结果冗长而复杂时,我们自然揣测存在着某个更清楚而且少迂迥的解:你能用不同方式导出这一结果吗?你能一下子看出它吗?即使我们成功地找出一个令人满意的解,我们可能仍然对找出另一个解感兴趣。就象我们期望通过两种不同的知觉去感觉到一个物体一样,我们也期望用不同的推导方法去取得对理论结果的有效性的信心。就象我们看到一个物体后还想摸摸它一样,
在有了一个证明后,我们希望找到另一个证明。
两个证明比一个好。“抛两个锚更安全”。
(1)例子。求正圆台的侧面积S,已知它的下底的半径 R,上底半径r和高h。
这个问题可用各种方法求解。例如,我们可能知道整个圆锥的侧面积的公式。
由于圆台是从圆锥切去一个较小的圆锥而得到的,所以它的侧面积是两个圆锥侧面积之差;于是剩下要做就是把它用R,r,h来表示。把这个思路付诸实现,
我们最后就得到公式
S=π(R+r) 22)( hrR +?
在用这种或那种方法求得这结果以后,经过较长的演算后,我们可能希望有一个更为清楚并且较少迂迥的论证。你能用不同方式导出这结果吗?你能一下子看出它吗?
为了能直观地看出整个结果,我们可以从尝试看出其各个部分的几何意义开始。这样,我们可能看出
22)( hrR +?
是斜高的长度(圆锥可看作是由一个等腰梯形绕平行两边中点连线旋转而成的,
斜高是该等腰梯形的腰;见图12)。此外,我们还可能发现
π(R+r)=(2πR+2πr)/2
是圆台两底周长的算术平均值。注意公式的这同一部分也可改写为
π(R+r)=2π(R+r)/2
这就是圆台的中截面之周长(这里,我们称平行于圆台上底和下底并等分其高的平面与圆台的交为中截面)。
在找到各部分的新解释以后,我们现在可以从不同角度来看整个公式。于是,我们可以这样读它,
侧面积=中截面周长×斜高
这里,我们可能回忆坦梯形面积的公式
面积=中线×高
图12
(此中线平行于梯形的两个平行边并等分其高)。只要直观地看到圆台侧面积和梯形面积这两种陈述间的类比关系,我们就可以“几乎一下子”看出圆台的整个结果。这就是说,对于以上经过冗长计算所得到的结果,我们现在感到非常接近于它的一个简短而直接的证明了。
(2)上面的例子是典型的。我们不完全满足于我们所导出的结果,而希望去改进它,改变它。因此,我们研究这个结果,尝试去更好地理解它,尝试看出它的某个新侧面。我们可能对于结果的某一小部分首先成功地观察出一个新的解释。然后,我们可能相当幸运地发现观察其他部分的新方式。
一个接一个地,审查各个部分,尝试用各种方式去考虑它们,我们可能终于能从不同角度看出整个结果,而我们关于结果的新概念可能给出一个新证明。
人们可能认为,这种现象对于处理某个高级问题的有经验的数学家要比那些解决某个初等问题的初学者更有可能发生。可是,具有大量数学知识的数学家比初学者更容易冒滥用知识而使论证不必要地复杂起来的危险。但作为补偿的是,有经验的数学家比初学者更能重视结果中细微部分的重新解释,并且能把它们积聚起来,最终重新写出整个结果。
不过,即使在低年级,学生也可能提出一个不必要那么复杂的解。于是,
教师至少一次或者两次指出下列各点,他不但应该指出如何更简捷地解题,而且也应该指出如何找出存在于结果本身的更简短解答的线索。
参见“归谬法与间接证明“一节。
8.你能利用这个结果吗?
借助于自己的方法来找出问题的解是发明创造。如果问题不太难,这发明创造也不大,但无论如何它毕竟还是发明创造。有了某个发明创造,尽管不大,
我们也应该探索它后面是否有更多的东西。我们不应该错过由这新结果所开创的可能性,我们应该再尝试使用一次我们已经使用过的方法。要利用你的成功!
对某个别的问题,你能利用这个结果或方法吗?
(1)如果我们对变化一个问题的主要方法,如“普遍化”,“特殊化”,“类比”,“分解和再组合”比较熟悉的话,我们就很容易想出一个新问题。我们从所提出的问题出发,用刚才提到的那些方法由它导出其他问题,从这些问题再导出别的问题,如此等等。从理论上说,这一过程是无限的,但在实际中,
我们很少进行得很长,因为这样所得到的问题容易成为棘手的问题。
另一方面,我们可以构造出新问题,这些新问题我们很容易利用以前所解决的问题加以解决,但这些易解的新问题又容易显得索然无味。
找出一个既有趣又好下手的新问题并不那么容易,这需要经验、鉴别能力和好运气。但是,当我们成功地解决了一个好问题以后,我们应当去寻拔更多的好问题。好问题同某种蘑菇有些相像,它们都成堆地生长。找到一个以后,
你应当在周围找找;很可能在附近就有几个。
(2)我们打算用第8,1O,12,14,15节中讨论过的同一个例子来阐明上述论点。所以,我们从下列问题开始,
已知长方体的长、宽、高,求外接圆的直径。
棱锥体的底面是一长方形,其中心为棱锥体的高的足。已知棱锥体的高及其底面的各边,求各侧棱。
已知空间中两点的直角坐标(x1,y1,z1),(x2,y2,z2),求此两点的距离。
我们容易解决这些问题,因为它们和已知其解的原问题相差不多。在每一情况中,我们都对原问题加些新概念,如外接圆,棱锥体,直角坐标。这些概念容易加进去,也容易去掉,并且当去掉它们之后,我们又回到了我们原来的问题。
由于我们引入原问题的概念是有趣的,所以上述问题也有一定的趣味。最后一个问题,即由两点的坐标确定其距离尤其重要,这是由于直角坐标很重要的缘故。
(3)如果我们已知原问题的解,这里还有另外一个我们很容易解决的问题:
已知长方体的长、宽和对角线,求其高。
事实上,我们原问题的解主要在于:为四个量(即长方体的长、宽,高和对角线)建立一个关系式。如果这四个量中的任意三个为已知,则我们可以由这关系式求出第四个。于是新问题可解。
对于从已有解的问题导出易解的新问题,这里有个模式;我们设原来未知数为已知,并将原来的已知数之一作为未知数。在这新、老两个问题中,联系已知数与未知数的关系式相同。在一个问题中找出关系式,我们即可把它用于求解另一个问题。
这个通过变换数据的地位以导出新问题的模式和第(2)点中的模式迥然不同。
(4)我们现在用其他的办法导出某些新问题。
对我们的原问题很自然地使之普遍化,就得到下列新问题:已知一个平行六面体从对角线一个端点出发的三条棱以及三棱间的三个夹角,求平行六面体的对角线。
用特殊化的办法,我们得到下列问题:已知正方体的棱长,求它的对角线。
用“类比”的办法,我们可得无数多的各种各样变型的问题。下面几个是从第(2)点中所考虑的问题导出来的,
已知正八面体的棱,求它的对角线。
已知正四面体的棱,求外接球的半径。
已知地球(假定为球体)表面上两点的几何坐标(经度和纬度),求两点间的球面距离。
所有上述问题都很有趣,但是只有用“特殊化”办法所得到的那个问题,
才能直接在原问题的解的基础上求出它的解。
(5)我们可以把原问题的某些元素看成变量,用这个办法从原问题导出新问题。
第(2)点所述问题的一个特例是:已知正方体的棱,求它的外接球的半径。
让我们把正方体和正方体与球的公共中心看成是固定不变的,但是可以改变球的半径。如果球的半径很小,则球在正方体内,随着半径的增大,此球胀大(就像一个橡皮的气球在充气的过程中)。在某一时刻,此球碰到这正方体的表面;
再过些时候,碰到它的棱;再晚些,此球通过其顶点。在这三个关键时刻,球的半径应取何值?
(6)如果一个学生从来就没有机会去解决一个他自己所发明创造的问题,
那么他的经验是不完整的。教师可以向学生示范如何从一个刚刚解决的问题引出新问题,这样做可以引起学生的好奇心。教师也可以留一部分创造发明给学生。例如,他可以谈到刚才所讨论的那个膨胀的球,他可以问学生:“你打算计算什么?半径的哪些值特别有趣?”
9.实现
想出一个计划与贯彻一个计划是两码事。在某种意义上,这点对数学问题也成立;因为在实现求解的计划和想出这个计划二者之间,其工作特点是不同的。
(1)就像建桥时搭架来支持桥身一样,当我们考虑数学上最后的和严格的论证时,我们也可以利用临时的与仅仅似乎有理的论证。不过,当工作充分进展之后,我们拆去棚架,桥自己应能站得住。同理,当求解过程充分进展之后,
我们将除去所有临时的和仅仅似乎有理的论证,而结论只能由严格的论证所支持。
在制定求解的计划时,我们不应该过于害怕那些仅仅是似乎有理的,探索性的论证。这时任何能导致正确概念的东西都是对的。但当我们开始实现求解计划时,我们必须改变这种观点,这时我们应该仅仅承认确凿的、严格的论证。
实现你的求解计划,同时检验每一步。你能清楚地看出这个步骤是正确的吗?
在实现计划时,我们检验每一步花的力气越多,那么在制定计划时,我们就可以更自由地应用探索性论证。
(2)我们应当适当考虑实现计划细节的工作程序,特别当问题是复杂的情况下更应如此。我们不应该略去任何细节,应该了解我们面前的细节对于整个问题的关系。我们不应该忽视主要步骤间的相互联系。因此我们应该按适当的次序进行。
特别是,在我们有充分理由相信论证的主要步骤是正确的以前,我们去检验次要的细节是不合理的。如果论证的主要思路有破绽,那么检查这个或那个次要细节将无济于事。
实现论证细节的次序和制定论证细节的次序可能是迥然不同的;而当我们把这些论证的细节最后写出来时,其次序可能更不同。欧几里得几何原理把论证的细节用一种刻板的、系统的次序提出来,这一点常为人效仿,但也常受人指责。
(3)在欧几里得的著作中,所有论证都按同一方向进行:在“求解题”中,
是从已知数据走向未知数,而在“求证题”中,是从前提走向结论。任何新元素如点、线等等,都必须正确地由已知数据推出或者从以前各步已正确导出的元素所推出。任何新推断都必须正确地由前提或者从以前各步已正确证出的推断所证明。每一步新元素,每一个新推断,当首次碰到时,都必须加以审查,
因此它只须审查一次;我们可以把全部注意力只集中于当前这一步,我们既不必瞻前,也不必顾后。这最后的新元素(我们必须加以检验它的推导过程)就是未知数。这最后的新推断(我们必须审查它的证明过程)就是结论。如果每步都正确,则最后一步也正确,从而整个论证是正确的。
如果目的是洋细审查论证,则欧几里得的论证展开方式很值得毫无保留地加以推荐。特别是,当论证是我们自己弄出来的,如果它冗长而复杂,同时我们不仅已经搞出来了,而且也已经从大的方面研究过它,所以除了审查其本身每个细节以外,再没有什么其他的事好做了,这时最好就是用欧几里得方式把整个论证写出来。
如果目的是把论证传授给读者或一个从来没有听到过它的听众,那么就不能无保留地推荐欧几里得的论证展开方式。对于说明每一特定点而言,欧几里得论证展开方式是优越的,但在阐述论证的主要思路方面,则不那么好。“聪明的读者”很容易看到每步是正确的,但要看出其来龙去脉、目的以及整个论汪的联系,则具有很大的困难。造成困难的原因是:欧几里得论证展开方式所遵循的顺序,相当经常地与创造它时的自然顺序刚好相反[欧几里得论证展开方式严格地服从“综合”的程序;参见“帕扑斯”一节,特别是其中的评论(3),
(4)(5)]。
(4)现在我们小结一下。欧几里得论证展开方式是严格地从已知到未知,
从前提到结论,这对于详细检验论证是完美的,但对于了解整个论证的主要线索却远非完美无缺。
我们非常期望学生能应用欧几里得方式来检验他自己的论证,但是这种工作也不应当搞得过分死板。我们并不期望教师用纯粹欧几里得方式给出许多证明。但是,在进行了本书所推荐的讨论,即学生在教师的指点下尽可能独立地发现求解的主要思路以后,再用欧几里得方式展开论证则是非常有用的。有些教科书采用的方式是:首先给出主要思路的直观提示,然后再用欧几里得的论证顺序说明各个细节,这看来也是合理的。
(5)为了想知道自己的命题是否为真,认真的数学家尝试去直观地看出它,
并给以形式逻辑的证明。你能清楚地看出它是正确的吗?你能证明它是正确的吗?
在这方面,数学家很像一个到商店买布的妇女,为了想知道布的质量如何,她总是希望看看它,摸摸它。直观的洞察和形式逻辑的证明是感知真理的两种不同方式,这堪与通过视觉与触觉两种不同的感官来感知物体相比拟。
直观的洞察可能远远超前于形式逻辑的证明。任何一个聪明的学生,无须任何系统的立体几何知识,当他弄清楚名词术语之后,他立刻能看出平行于同一直线的两直线彼此平行(此三条直线可以在同一平面上,也可不在同一平面上)。但证明这个命题则需要冗长、细致和创造性的准备工作,如欧几里得几何原理第11分册命题9所给出的那样。
逻辑规划与代数公式的形式演算比直观要深入得多。几乎每个人都可立刻看出:任意取三根直线可划分平面为7部分(请看着它唯一确定的部分,由三根直线所围成的三角形)。但几乎没有人能看出(即使他是全神贯注、聚精会神地来看);任意取的5个平面将空间分成26个部分。但是可以严格证明其准确数字确实是26,而且证明过程既不长也不难。
实现我们的计划,同时检验每一步。检验每一步时,我们可以依赖直观洞察或形式法则。有时直观在前,有时形式推理在前。同时用两种方式来做则是个有趣又有用的练习。你能清楚地看出这一步是正确的吗?是的,我能很清楚地看出它来。这是直观走在前面了;但形式推理能否紧紧跟上呢?于是,我们问:
你是否也能证明它是正确的?
尽可能形式地证明我们所直观看到的,以及尽可能直观地看出我们所形式证明过的,这是一种增进智力的练习。不幸,在教学中,并不总有足够的时间来这样做。第12,14节所讨论的例子在这方面是典型的。
10.条件
条件是“求解题”的一个主要部分。参见“求解题,求证题”一节第(3)
点,还可参见“新旧术语”一节第(2)点。
如果一个条件包含了过多的部分,则此条件称为多余的。如果一个条件的各部分互相矛盾并且互不相容以致无对象可满足此条件,则此条件称为矛盾的。
这样,如果条件用比未知数个数多的线性方程来表达,则它不是多余就是矛盾的;如果条件用比未知数个数少的方程来表达,则它是不充分的,不足以确定未知数;如果条件用与未知数个数一样多的方程来表达,那么在通常情况下,它对于确定未知数正好是充分的,但在例外情况下,也可能是矛盾的或不充分的。
11.矛盾
参见“条件”一节。
12.推论
推论是一个定理,它是在研究另外一个刚求得的定理时很容易推出来的定理。推论(Corollary)这个字来源于拉丁文,一个更口语化的泽法是“小费”或
“酒钱”。
13.你能从已知数据导出某些有用的东西吗?
在我们面前有个未解决的问题,一个随便用什么方法处理的问题。我们必须找出已知与未知间的联系。我们可以把我们待解的问题表示成已知与未知之间的广阔空间,当作已知与未知之间的一道鸿沟,在其上需要架桥。我们架桥可以从任何一边(从未知或者从已知)开始。
看着未知数!试想出一个具有相同或相似未知数的熟悉问题。这是建议你从未知数开始进行工作。
看着已知数!你能从已知数导出什么有用的东西吗?这是建议你从已知数开始进行工作。
通常,从未知数开始论证似乎更为可取(参见“帕扑斯”和“倒着干”这两节)。但另一种起步方法是从已知数开始,也有成功的可能,也必须常常试试,
并值得加以说明。
例子:给定三点A,B,C。过A作一线与B,C等距离。
已知是什么?给定三点A,B,C的位置。我们画一张图,表示已知的事项(图
13)。
图13
未知是什么?一条直线。
条件是什么?所求直线过A,且与B,C等距离。我们把未知事项与已知事项放在一张图中,表示出所需要的关系(图14)。根据点到直线距离的定义,图14
上表示出了定义中所包含的直角。
这样画出的图未免太“空旷”。把未知的直线与已知的A,B,C相连结,
看来不能令人满意。图14需要某条辅助线,还要加点什么——但是要加什么呢?
一个高水平的学生对此也可能一筹莫展。当然,有各种办法可试,但帮助学生精神重新振作起来的最好问题是:你能从已知事项导出什么有用的东西?
图14
实际上,已知是什么?就是图13中表示的三个点,再没有别的了。我们迄今尚未充分利用过点B和C;我们必须从它们导出什么有用的东西。但是只有两个点,你能做什么呢?把它们用一条直线连起来。这样,我们就画出图15。
图15
如果我们把图14和图15叠起来,问题的解答可能突然闪现出来。这里有两个直角三角形,彼此全等,还有一个十分重要的新交点。
14.你能重新叙述这个问题吗?
你能重新叙述这个问题吗?你能否重述得更不同些?提出这些问题的目的是找出合适的“变型的问题。”回到定义去。参见“定义”一节。
15.分解与重新组合
分解和重新组合是重要的智力活动。
研究一个引起你兴趣或者挑起你好奇心的对象:如,一间你打算租用的房子,一封重要而又神秘的电报,或者任何一个来龙去脉及其目的部使你困惑不解的事物,或者任何一个你打算解决的问题均可做为研究的对象。你对于这个对象有一个整体的印象,但这印象很可能还不够明确。你想起了一个细节,你便集中注意力于它。然后,你又注意到另一个细节;以后又注意到其他一个。
细节的各种组合都可能呈现出来,过了一会儿,你又把对象当作整体来加以考虑了,但现在你是从不同的角度来看它了。你把整体分解成许多部分,然后你又把各个部分重新组合成或多或少有些不同的整体。
(1)如果你钻到细节中去,你可能会在细节中迷途。过多或过细的具体情节是脑力的一种负担。它们如一叶障目会阻碍你充分注意主要之点,甚至使你完全看不到主要之点,只见树木而不见森林。
我们当然不希望为不必要的细节去浪费我们的时间,我们应该把我们的精力用到主要内容上。困难就在于我们事先说不出哪些细节最后会变成主要的,
而哪些又不会。
因此,我们首先应当把问题作为一个整体来了解。在弄清了问题之后,我们将处于较有利的地位来判断哪些具体细节可能是最主要的内容。在研究过一两个主要之点以后,我们将会处于更有利的地位来判断还有哪些细节可能值得较细致的研究。让我们进入细节并将问题逐步地加以分解,但不要超过我们所需要的程度。
当然,教师不能期望所有的学生都在这方面干得很出色。正相反,有的学生有一种非常愚蠢、非常坏的习惯,他们在把问题当作整体来了解之前就贸然从细节开始工作。
(2)我们将考虑教学问题,“求解题”。
在把问题作为一个整体加以理解,并了解过其目的,其主要点之后,我们希望开始了解细节。我们应该从哪儿开始?几乎在所有的情况下,从考虑问题的主要部分(即,未知数,已知数与条件)开始是合理的。我们要在几乎所有的情况下奉劝读者从下列问题开始详细研究:未知数是什么?已知数是什么?条件是什么?
如果我们希望研究进一步的细节,我们应当怎么办?通常适当的办法是去研究每个数据本身,并且把条件的各个部分分开,再研究每一部分本身。
特别是当我们的问题比较困难时,我们可能感到很有必要进一步把问题再分解成几部分,并研究其更细微的末节。这样,我们就可能有必要回到某个术语的定义去,需要引入此定义所涉及的新元素,并加以研究。
(3)分解问题以后,我们试用某个新方式重新组合其元素。尤其是,我们可能尝试把问题的元素重新组合成某个新的、更好下手的问题,这个问题我们可能当作一个辅助问题。
当然,重新组合的可能性是无限的。困难的问题需要有一种神奇的、不寻常的、崭新的组合。而解题者的才能就在于组合的独创性。但也存在着某些普通的、相对简单的组合,它们对于较简单的问题而言已经够用。对于这样的组合我们应当彻底加以了解并且首先试用,即使我们最后不得不求助于不太显而易见的方法。
以下是个形式上的分类,其中简洁地列出了最普通和最有用的组合。从所提问题构造出一个新问题,我们可以,
(I)保持未知数不变而改变其他(已知数与条件);或者
(II)保持已知数不变而改变其他(未知数与条件);或者
(III)同时改变未知数和已知数。
我们将研究上述几种情况。
[情况(I)与情况(II)是交叠的。事实上,有可能同时保持未知数和已知数不变而只改变条件的形式来变换问题。例如下列两个问题,虽然看起来等价,
但并非完全相同,
给定一边,作一个等边三角形。
给定一边,作一个等角三角形。
在本例中,两个命题的差别很小,但这点差别在别的情况下则可能至关重要。后面这类情况虽然在某些方面很重要,但我们在这里却不能占用过多的篇幅来讨论它们。请与“辅助问题”那节的第(7)点最后一点说明相比较]。
(4)为了变换所提的问题,保持未知数而改变已知数与条件是经常用到的。
“看着未知数”的建议是针对具有相同未知数的问题的。我们可以回忆一个早已解决的同类问题:尝试想起一个具有相同或相似未知数的熟悉问题。如果想不起来,就努力创造一个:你能想起便于确定未知数的其他已知数吗?
一个新问题与所提问题关系越密切,则其有用的可能性越大。所以,我们要保持未知数不变,同时也试图保持某些已知数和某一部分条件,并在允许的范围内使改变尽可能地少,只改变一两个已知数和一小部分条件。一个好办法是:我们只略去某些东西而不增添任何东西;我们保持未知数,保持仅仅一部分条件,舍去其他部分,但不引人任何新的条件或巳知数。关于这种情况的例子和说明见第7,8点。
(5)保持已知数不变,同时我们可以尝试引入某些有用的和更好着手的新未知数。这样一个未知数必须是由原数据得到的。于是,当我们问:“你能不能从已知数导出某些有用的东西?”时,我们脑海中就有了这样一个未知数。
我们注意到最好能满足下面两点:第一,新未知数应当更好着手,即,它比原来的未知数更容易从已知数求得。第二,新未知数应当是有用的,即求出它以后,它应该在求解原未知数方面能起到某些肯定的作用。简而言之,新未知数应该是一块踏脚石。它是小河中间的石块,它和我们的距离比我们所欲到达的彼岸更近,当我们踏上这石块时,它会帮助我们向彼岸迈进。
新未知数应当是既好着手又有用,但在实际中,我们往往不得不只满足于其中的一点。如果提不出什么更好的东西,那么从已知数导出某些可能有用的东西也是值得的;同样,引入一个和原未知数密切相关的新未知数,即使这个新未知数乍一看来并不特别好下手,这样做也是合理的。
例如,如果我们的问题是找出平行六面体的对角钱(如第8节),我们可以引入一个平面上的对角线作为新未知数。我们这样做或者是因为我们知道从平面上的对角线可以求出空间的对角线(如第10节);或者是由于我们看出平面上的对角线容易求,而我们猜测它在找出空间的对角线方面可能有用(请与“你是否利用了所有的已知数?”一节相比较)。
如果我们的问题是作一个圆,我们必须找出两样东西:圆心和半径;我们可以说,这问题有两部分。在某些情况下,一部分较另一部分更好着手,因此,
在任何情况下,我们都可以合理地对下述可能性考虑片刻,你能解决问题的一部分吗?在提这个问题时,我们应当衡量衡量各种可能性:是集中注意力于圆心呢,还是集中注意力于半径?选这个作新未知数,还是选那个作新未知数?哪个上算?这类问题常常是非常有用的。在更复杂或更高级的问题中,决定性的念头经常出现在把某个更好着手,同时又是主要的部分从问题中剖析出来的过程之中。
(6)同时变换未知数和已知数。这样做,我们离开原问题比上面更远。我们当然并不喜欢这样;我们意识到这样会有完全失去原问题的危险。但如果其他变动较少的办法不能得出又好着手又有用的东西,那我们就迫不得已只好使用这种变动较大的办法了;或者,如果这样得出的新问题有较大成功的希望,
我们就可以被吸引到离开原问题如此远的地方来。如果需要的话,你能不能改变未知数已知数,或者两者都改变,以使新未知数和新已知数彼此更接近?
同时改变未知数和已知数的一个有趣的方式是把未知数与数据之一进行互换(参见“你能利用这结果吗?”一节第3点)。
(7)例子。已知三角形一边a、垂直于a的高h和a的对角a,作一个三角形。
未知是什么?一个三角形。
已知是什么?两条线段a,h和角a。
现在,如果我们对几何作图问题还比较熟悉的话,我们可尝试把这样一个问题化简为点的作图问题。
我们画线段BC等于边长a,所有余下要找的就是三角形
图16
中a边所对的顶点A(见图16)。这样我们就有了个新问题。
未知是什么?点A。
已知是什幺?直线h,角a,已给定位置的两点B,C。
条件是什么?点A到直线BC的垂直距离应为h,∠BAC=α。
事实上,我们已经变换了我们的问题,改变了未知和已知。新的未知是一个点,老的未知是一个三角形。在这新老两个问题中,某些已知事项是共同的,
即线段h和角α;但在老问题中给定的是长度a,而现在则代之以给定两点B与C。
新问题不难。下列建议使我们十分接近于解。
把条件的各个部分分开。本题的条件有两部分,一个与已知的h有关,另一个与已知的α有关。对这个未知点要求,
(I)对直线BC的距离为h;
(II)成为一角的顶点,此角大小为α,其两边通过给定点 B,C
如果我们只保留一部分条件,而舍去其余部分,则此未知点不能完全确定。
有许多点都可满足条件中的(I)这一部分,这就是那些位于平行于BC且相距为h
的平行线上的所有点*。这根平行线是满足部分条件(I)的点的轨迹。满足部分条件(II)的点的轨迹是一圆弧,其端点为B与C。我们可画出这两个轨迹;它们的交点就是我们所求作的点。
——————
*:过B与C的直线把平面一分为二,我们选择其中之一在它上面作出A。所以这里我们可以只考虑BC的一根平行线,否侧我们应当考虑两根这样的平行线。
方才所用的程序颇有趣味;解决几何作图问题时,我们常常可以成功地按照其模式,即:把问题化简成作一个点,并将此点当作两轨迹的交点来作图。
但这个程序的某一步具有更普遍的意义,在解决任何类型的“求解题”时我们可遵循它的模式,即:仅仅保留一部分条件,合去其余部分。这样做,我们所提问题的条件减弱了,我们对未知数的约束减少了。这样未知数能确定到什么程度?它能怎样改变?通过提问这点,我们实际上是建立了一个新问题。如果未知的是平面上的一个点(如我们例中的情况),那么确定此点所作出的轨迹就蕴含了该新问题的解。如果未知的是其他类型的数学对象(在第18节中是一个正方形),则我们必须恰当地描述并精确地规定对象的某个集合的特征。 即使这未知事项不是个数学对象(如下述8中的例子),我们去考虑并找出其特征,去描述或列举那些满足原问题中关于未知事项的部分条件的对象也可能是有用的。
(8)例子。在一个纵横填字字谜中,允许使用同音异义的双关语和倒字*,
我们发现该字谜有下列暗示,
————————
*译注:倒字指颠倒一字的字母顺序而成的字,如live与evil
“一个机器中的向前与向后的部分(Forward and backward part of a
machine)(有五个字母)”
未知数是什么?一个字。
条件是什么?这个字有五个字母。它与某种机器的某部分有关。当然,它应该是个英文字,并且我们希望它不是很生僻的字。
这条件对于确定此未知数是否充分?不,或者不如说,这条件可能是充分的,但现在已 经清楚的那部分条件肯定是不充分的。满足它的字太多,如,lever”
(操纵杆),或“screw”(螺钉)等等。
此条件表达得含混不清——当然是故意的。如果找不到什么东西可以似乎令人可信地描述为机器的“向前部分”,如果对其“向后部分”也是这样,我们可以猜测它可能指的是向前和向后阅读。研究字谜暗示的这种解释也许是个好念头。
把条件的各个部分分开。此条件有两个部分,一个与字的意义有关,另一个与拼法有关。对此未知字的要求是,
(I)是一个短字,它的含义是某种机器的某个部分;
(II)是一个有五个字母的字,它倒过来拼时还成为一个字,它的意思是某个机器的某个部分。
如果我们仅保留条件的一部分而舍去其余部分,未知字就无法完全确定。
有许多字满足条件的部分(I),这也是一种轨迹。我们可以“描述”这个轨迹(I),
并“跟踪”它直至得到它和轨迹(II)的交点。比较自然的程序是集中力量于条件的部分(I),去收集具有规定含义的字,在收集到若干个这样的字以后,我们检查它们的字长是否合乎规定,是否能倒过来念。这样,在得到正确的字以前,
我们可能收集到下述几个字:lever(操纵杆),screw(螺钉),wheel(轮),
shaft(轴),hinge(铰链),rotor(发电机的转子)。
当然是“rotor"”(发电机的转子)。
(9)在上述(3)中,我们对提出的“求解题”的元素重新组合得到一个新“求解题”的种种可能性进行了分类归纳。如果我们所引进的并不正好是一个新问题,而是两个或多个新问题,那么就有更多的可能性。这点我们必须提到,但不打算分类归纳了。
还有别的可能性。特别是,“求解题”的解可能依赖于“求证题”的解决。
我们只是提一提这种重要的可能性。由于篇幅所限,不再予以讨论。
(10)对于“求证题”,我们只需少量的附加说明。这些说明和前面“求解题”较为广泛详尽的说明【(2)至(9)】相类似。
对求证题的整体有了了解以后,我们一般应当研究它的主要部分,即我们欲证或欲推翻的定理的前提和结论。我们必须彻底了解这些部分。前提是什么?
结论是什么?如果需要停下来注意更多的特定点,我们可以把前提的各部分分开,并且考虑每个部分本身。然后我们可以考虑其他细节,一步步深入地把问题分解成许多部分。
在分解问题以后,我们尝试用某种新方式将其元素重新组合。特别是,我们可以把元素重新组合成另一定理。在这方面有三种可能性。
(I)我们保持结论不变而改变前提。我们首先试图找出这样一个定理:看着结论!并试想起一个具有相同或相似结论的熟悉的定理。如果找不到,我们就创造一个:你能否想起另外一个很容易导出结论的前提?我们可以用省略某些东西而不是添加某些东西的办法来改变所提问题的前提:只保留前提的一部分,而舍去其余部分,这结论是否仍有效?
(II)我们保持前提不变而改变结论:你能从前提导出某些有用的东西吗?
(III)我们同时改变前提和结论。如果只改变一个不行,我们可能倾向于同时改变两个。如果需要的话,你能否改变前提,或结论,或二者都改变,以使新前提和新结论彼此更接近?
我们这里不打算对下述情况所引起的各种可能性加以分类归纳。这些情况是:为了解决所提出的“求证题”,我们引人两个或更多的新“求证题”,或者我们把“求证题”和适当的“求解题”联系起来考虑。
16.定义
一个术语的定义就是用其他假定已为人所周知的术语来阐明该术语的意义的一个语句。
(1)数学中的专业术语有两类。有些作为原始术语不加定义而被接受。其余则当作派生的(导出的)术语而用适当的形式加以定义;也就是说,其意义用原始术语和先前已经定义过的派生术语来加以陈述。这样,对于点,直线,平面这样的原始术语,我们并不给出形式的定义*。但对于“分角线”,“圆”,“抛物线”这类概念,我们要给出形式定义。
————————————
*:在这方面,自欧几里得及其希腊追随者的时代以来,观念已经有所变化。他们曾定义过点、直线与平面,然而他们的定义与其说是形式定义,还不如说是一种直观的说明。当然,直观说明是允许的,在教学中,甚至是非常希望的。
上述最后一个带引号的术语可定义如下:抛物线是下述点的轨迹,这个点距离一定点和一定直线的距离相等。该定点称为抛物线的焦点,而该定直线称为抛物线的准线。不言而喻,所有上面所考虑的元素是在一确定平面中并且上述定点(焦点)不在定直线(准线)上。
我们假定读者并不知道所定义的术语:抛物线,抛物线的焦点,抛物线的准线。但我们假定读者知道所有其他各个术语的意义,如点,直线,平面,点与另一点间的距离,确定的轨迹等等。
(2)字典中的定义从表面上看来,和数学定义并无多大区别,但他们是以不同的旨意写的。
字典的作者关心各个字当前流行的含义。当然,他接受这流行的意义并以定义的形式尽可能简洁地加以陈述。
可是数学家却不关心他的专业术语有什么流行的意义,至少他主要不关心这点。“圆”或“抛物线”或其他这类专业术语在日常说话中可能表示什么,
也可能不表示什么,这与他无关。数学定义产生数学上的意义。
(3)例子。已知抛物线的焦点和准线和一直线,求作此抛物线与已知直线的交点。
我们解决任何问题的方法必定由我们知识的状况所决定。所以,目前这个问题的解决方法主要取决于我们对抛物线性质的熟悉程度。如果我们对抛物线知道得不少,那么我们可以尝试去利用我们的知识,从这些知识中汲取有益的东西:你知道一个能被利用的定理吗?你知道一个与此有关的问题吗?如果我们对抛物线、焦点、准线所知甚少,那么这些术语不免令人望而生畏,我们自然想摆脱它们。怎样摆脱它们呢?请听下面教师与学生关于这个问题的一段对话。
他们选取了适当的记号:P为任何未知交点,F为焦点,d为准线,c为与抛物线相交的直线。
,那么,未知是什么?”
,点P”
,已知是什么?”
,直线c与d,与点F。”
,条件是什么?”
,P是直线c和抛物线的交点,抛物线的准线为d,焦点为F。”
,对。我知道你对抛物线了解不多。但我想你能够说出抛物线是什么?”
,抛物线是与焦点和准线距离相等的点的轨迹。”
,对。这定义你记得很对。那很好,但我们还必须利用它;回到定义去。
根据抛物线的定义,关于点P你有什么可说的吗?”
,P在抛物线上。因此,P与d和F的距离相等。”
“好,画张图。”
学生在图17上引入线PF与PQ,后者是P点向d所作垂线。
图17
“现在,你能重新叙述这个问题吗?”
,利用刚才引入的线,你能重新叙述这个问题的条件吗?”
,P是在线c上使PF=PQ的点。”
,好,请用文字说明:PQ是什么?”
,P到d的垂直距离。”
,好,现在你能重新叙述这个问题吗?请用明确而又简洁的句子加以叙述。”
“在已知直线c上作点p,使它和已知点F及已知直线d等距离。”
“看看从原来的说法到你现在的说法之间有多大进步吧!对这问题,原来的说法充满了不熟悉的专业术语,抛物线呀,焦点呀,准线呀;听起来未免有点装腔作势和趾高气扬,而现在一点儿也没有那些不熟悉的专业术语了;你已经使这个问题简练了。干得好!”
(4)消去专业术语。上面例子的工作结果就是消去了专业术语。我们从一个包含某些专业术语(抛物线、焦点、准线)的问题的陈述开始,而最终找到了没有这些术语的陈述。
为了消去一个专业术语,我们必须知道这个专业术语的定义;但仅知其定义还不够,我们还必须利用定义。在上例中,只记得抛物线的定义是不够的。
决定性的一步是在图中引入直线PF和PQ,根据抛物线的定义这二者是相等的。
这过程是典型的。我们在问题的概念中引入适当的元素。我们在定义的基础上建立所引入的元素之间的关系。如果这些关系完全表达了术语的含义,则我们就已经利用了定义。利用了定义,我们同时也就消去了专业术语。
刚才所叙述的过程可称为:回到定义去。
用回到一个专业术语定义的办法,我们除去了这个术语,而代之以新元素和新关系。这在我们的问题的概念中所产生的变化可能很重要。无论如何,对问题的某种重新叙述,“问题的某种变化”是与结果密切相关的。
(5)定义与已知定理。如果我们知道“抛物线”这个名词并且关于其曲线形状具有某些模糊概念,但此外别无所知,那么我们的知识显然不足以解决作为例子的这个问题或其他有关抛物线的重要几何问题。为此目的,我们需有哪类知识呢?
我们可以认为几何这门学科是由公理、定义与定理所组成。在公理中,并未提及抛物线,公理只处理点、直线等这类原始术语。有关抛物线的几何论证和任何涉及抛物线的问题的求解都必须利用关于抛物线的定义或定理。为了解决这样的问题,我们至少必须了解其定义,当然如果还知道某些定理则更好。
显然,我们关于抛物线所说的一番话,对于任何派生概念都同样成立。当我们开始解决一个涉及这种派生概念的问题时,我们还不知道在定义或某些有关它的定理中,我们选用哪一个比较好一些;但有一点是肯定的,我们必须选其中一个。
然而在有些情况下,我们并没有选择的余地。如果我们只知道概念的定义,
别无其他,我们就只好被迫采用这定义。如果我们所知并不比定义为多,我们最好的机会可能是:回到定义去。但是,如果我们知道有关概念的许多定理,
并且已有许多使用这些定理的经验,那么我们就有机会找到一个涉及上述概念合适的定理。
(6)几个定义。球通常定义为:与一定点保持定距离的点的轨迹(现在所说的是空间的点,不限于平面)。但球也可定义为:一圆绕直径旋转所描画的表面。
我们还知道另外一些关于球的定义,可能还有许多其他的定义。
当我们必须求解含有派生概念(如“球”,“抛物线”)的问题,并且当我们希望回到它的定义去的时候,我们可能需要就各种定义进行选择。在这种情况下,选择适合情况的定义至关紧要。
在阿基米德时代,求出球面积是个伟大而艰巨的任务。阿基米德曾经就我们刚才所引述的两种球的定义进行选择。他宁愿把球看成由一个圆环绕一个固定直径旋转而成。他在圆内内接一正多边形,边数为偶数,多边形的固定直径连接相对的顶点。此多边形近似于圆,且与圆一起旋转,产生了一个凸面,其两端为两个锥体,顶点在固定直径的两端,中间是若干个圆台。此复合表面近似于球,阿基米德就用它来计算球面积。但如果我们把球看成距离中心距离相等的点的轨迹,那么对于球面就提不出如此简单的近似。
(7)在考虑论证时,回到定义去固然重要,但在检验论证时,这点也很重要。
有的人提出了自称为阿基米德球面问题的新解答。如果他对球仅有模糊的概念,则他的解不会有任何高明之处。如果他对球的概念是清楚的,但却不会在论证中使用它,那么我就无法知道他究竟有没有什么概念,从而他的论证也不会是高明的。所以,我在倾听别人论证的时候,我总期待在某一时刻他将说出某些关于球的实质性内容,用到球的定义或者某个关于球的定理。如果等来等去总等不到这样一个时刻,那么他的解答一定不行。
我们不仅应当检验别人的论证,而且当然也应当用同样的方式检验我们自己的论证。你是否已经考虑了问题中所包含的所有基本概念?你是怎样利用这个概念的?你是否利用过它的意义,它的定义?你是否利用了它的基本事实,有关它的已知定理?
巴斯卡也曾强调过“回到定义去”在检查论证的有效性方面的重要性。他叙述过这样一条规则:“在心里用定义的事实代替被定义的术语。”哈达马得曾强调:“回到定义去”在想出论证方面也是重要的。
(8)回到定义去是一项重要的智力活动。如果我们希望了解为什么字的定义如此重要,那么我们应当首先认识到,字是重要的。如果不用字,不用符号或某种记号,我们几乎不能思维。所以,字和符号是有威力的。原始民族信仰字和符号具有魔力。我们可以理解这种信仰,但却不可苟同。我们应当知道:
字的威力不在于它的声音,也不在于说话者送出的一阵气息,即“热气”,而在于字给我们提示的概念以及这些概念最终所依据的事实。
因此,寻求字面背后的意义和事实是一种健全的倾向。对于回到定义去,
数学家寻求的是:掌握那些在专业术语后面数学对象间的实际关系;物理学家寻求的是:专业术语后面的明确实验;而具有某种常识的普通人则希望找出铁的事实而不仅仅为字面所愚弄。
17.笛卡尔
笛卡尔是大数学家与大哲学家。他计划给出一个解题的普通方法,但是他仅留下他的未完成遗作:“思维指导法则”。在他死后从他的手稿中所发现并印刷出版的这篇论文的片断比他那篇更为人所知的文章:“方法论”包含有更多的也更有趣的有关解题的材料,虽然“方法论”比“法则”的写作日期还要晚。看来,笛卡尔下面这段话叙述了写“法则”的动机:“当我年青的时候,
我听见有关天才的发明,甚至在我还没有读过作者的著作以前我就尝试自己去发明它们。这么做,我发觉我在某种程度上利用了某些规则。
18.决心,希望,成功
认为解题纯粹是一种智能活动是错误的;决心与情绪所起的作用很重要。
半心半意和懒洋洋地同意做一点事情,对于在教室中做代公式题可能是够了。
但是,去求解一个严肃的科学问题需要坚强的意志才能成年累月地含辛茹苦和百折不挠。
(1)决心随着希望与失望,称心与挫折而波动摇摆。当我们认为解答就在眼前时,决心很容易维持;当我们陷入困境,无计可施时,决心很难保持下去。
当我们的推测成为现实时,我们欢欣鼓舞。当我们以某种信心所遵循的道路突然受阻时,我们又不免垂头丧气,我们的决心也随之动摇了。
“你能无望而受命,百折而不挠。”这才称得上意志坚强,受人尊敬和信受职责,才是一个具有崇高目标的高尚的人,然而这类决心对于科学家则大可不必,科学家应当抱有某些希望才开始工作,并且应当有某种成功才继续干下去。在科学工作中,决心的大小必须灵活地根据前景而定。除非你对一个问题有某些兴趣,你才去着手解答它;如果这问题看来有指导意义,那么你就定下心来认真地去作;如果它很有搞头,你就全力以赴。一旦你目标已定,你就要锲而不舍,但你的日标对你自己来说不可过高。你不要轻视微小的成功,相反,
你要追求它们:如果你不能解决所提问题,首先尝试解决某个有关的问题。
(2)当一个学生的错误实在很大或者迟钝得令人恼火时,原因几乎总是相同的:他根本不想解题,甚至不愿正确理解这个问题,所以他对问题并未理解。
因此,凡是真心希望帮助学生的教师首先应当挑起学生的好奇心,给他某种解题的愿望。同时教师也应当给学生一一些时间,使他下定决心,定下心来做他的功课。
教学生解题是意志的教育。当学生求解那些对他来说并不太容易的题目时,他学会了败而不馁,学会了赞赏微小的进展,学会了等待主要的念头,学会了当主要念头出现后全力以赴。如果学生在学校里没有机会尝尽为求解而奋斗的喜怒哀乐,那么他的数学教育就在最重要的地方失败了。
19.诊断
这里,诊断用作一个教育方面的专业术语,其含义是:“对学生的工作进行更细致的鉴定。”分数虽然对学生也是一种鉴定,但多少有些粗糙。希望改进学生工作的教师需要对好的地方和坏的地方作更细致的鉴定,就象一个希望改进病人健康状况的医生需要诊断一样。
这里我们特别关心学生解题的效率。我们如何鉴定它呢?我们可以利用解题四阶段间的差异。事实上,学生在不同阶段的行为是很有特点的。
由于思想不集中而造成的对问题了解不完整大概是解题中最为常见的毛病。至于在制定一个计划并得到求解的一个总的概念这一阶段中,常见的是两种截然不同的毛病。有的学生没有任何计划或总的概念,就急急忙忙地选人具体计算和作图;另外一些学生则笨头呆脑地干等着某个念头的降临,而不会做任何事情去加速其来到。在实现计划阶段,最常见的毛病是粗枝大叶,不耐心检查每一步。根本不检查结果是屡见不鲜的;学生乐意得到一个答案,丢下笔结束,对于最靠不住的答案他们也满不在乎。
对这类毛病作出细心的诊断以后,教师有某种可能来医治它,办法是向学生不断提出表中所列的某些问题。
20.你是否利用了所有的已知数?
由于我们的知识是逐步增加的,我们对问题的概念在结束时要比开始时丰富得多(参见“进展与成就”一节第1点)。但现在它怎么样了?我们已经得到所需要的了吗?我们的概念足够吗?你是否利用了所有的已知数?你是否利用了整个条件?对于求证题,相应的问题是:你是否利用了全部前提?
(1)作为说明,让我们回到第8节(接下来是第10,12,14,15节)中所讲的
“长方形问题”。有可能出现下述情况:学生想到了怎样计算一个平面的对角线,22 ba +,然后他停滞不前了。教师可以帮助他而提问:你利用了所有的已知数吗?这时学生差不多总会看出:表达式 22 ba + 。中未包含第三个数据c。
因此,他应当试试让c出现。这样一来,他很有可能发现那个关键的直角三角形,
其二边为 22 ba +,和 c而其斜边就是所求的长方体的对角线(见“辅助元素”
一节第(3)点,那里有另一个说明)。
我们这里所讨论的问题非常重要。它们在寻找一个解答时的用处在前例中已很清楚。它们可能帮助我们找出对问题的概念中的弱点。它们可能指出一个被遗漏的元素。当我们知道有某个元素仍被遗漏时,我们自然会设法让它出现。
这样,我们就有了线索,就有了明确的探索途径,就有了好机会去找到关键的思路。
(2)所讨论的问题不仅对构造一个论证有帮助,而且对检验它也有帮助。
为了更具体些,我们需要检验一个定理的证明,其前提包含三部分,所有这三部分对于定理的成立都是必要的。这就是说,如果舍去前提的任何部分,定理将不成立。因此,如果证明时,忽视了利用前提的任一部分,则证明必有错误。
此证明利用了全部前提吗?它利用了前提的第一部分吗?哪儿用了前提的第一部分?哪儿用了第二部分?哪儿用了第三部分?回答所有这三个问题,我们就检验了证明。
这类检验是有效的,富于启发的,并且当论证冗长而复杂时,为了彻底理解它几乎是必不可少的(见“聪明的读者”一节)。
(3)我们所讨论的问题以审查我们对问题的概念的完整性为目的。如果我们没有把任何主要的数据,或条件,或前提考虑进去,那么我们的概念肯定不会完整。但如果我们不体会某个主要术语的意义,则我们的概念也不完整。因此,为了检查我们的概念,也应该提问:你已考虑了问题中所包含的所有必要的概念吗?(见“定义”一节第(7)点)。
(4)不过,对上述说明需加小心并加以某些限制。实际上,它们只能直接用于“陈述完善的”而且“合理的”问题。
一个陈述完善的而且合理的“求解题”必须具有所有必需的已知数,而没有任何多余的已知数;并且它的条件必须恰好充分,既不矛盾,也不多余。在解决这样一个问题时,我们当然必须利用所有的已知数和整个条件。
求证题的对象是数学定理。如果问题是陈述完善的并且是合理的,那么定理前提中的每一句对于结论都是必要的。证明这样一个定理时,我们当然必须利用前提中的每一句话。
在传统的教科书中所提的数学问题被假定是陈述完善的,合理的。但我们不应该过分信赖这点;即使有最微小的怀疑,我们也应该问:“满足这条件可能吗?”试图回答这个问题,或者其他类似问题,我们可以至少在某种程度上相信我们的问题提得正确,正如所假设的那样。
只有当我们知道我们面前的问题的陈述是完善的,合理的,或者我们至少没有理由作相反的猜测时,我们才能原原本本地提问本节所提到的问题以及与之有关的问题。
(5)有些非数学问题在某种意义上也是“陈述完善的”,例如,精彩的棋局假定只有一个解并且在棋盘上没有多余的棋子,等等。
但“实际问题”通常远非“陈述完善的”,从而需要对本节所提的问题认真地重新予以考虑。
21.你知道一个与此有关的问题吗?
我们几乎不能想象有一个问题是绝对的新颖,和我们以前解决过的任何问题都不相似,都无关系;但若居然有这样一个问题存在,它将是不可解的。事实上,当解决问题时,我们总利用以前解决的问题,用其结果或用其方法,或利用解决它们时所得到的经验。当然我们所利用的这些问题必须在某一方面与我们当前的问题有关。所以,我们提这个问题:你知道一个与此有关的问题吗?
想出一个与我们当前问题多少有关的已解决的问题,通常并不困难。相反,
我们可能发现这样的问题太多了,从中选出一个有用的问题反倒可能有困难。
我们必须找出联系密切的问题:我们看着未知数,或者我们寻找一个早已解决的问题,这个问题与我们当前的问题通过“普遍化”,“特殊化”或“类比”
而发生联系。
我们这里所讨论的问题的目的是把以前所获得的知识调动起来(见“进展与成就”一节第1点)。我们数学知识的一个主要部分是以过去证明过的定理形式存储在我们大脑中的。所以有下列问题:你知道一个可能用得上的定理吗?
当我们的问题是一个“求证题”(即必须证明或推翻一个提出的定理)时,上述问题可能特别合适。
22.画张图
参见图形一节。引入适当的符号:见“符号”一节。
23.检验你的猜测
你的猜测可能是对的,但是把一个逼真的猜测当作已被证明的真理来看待却是愚蠢的(原始人常这样)。你的猜测也可能是错的,但是把一个逼真的猜测完全弃之不顾也同样是愚蠢的(书呆子有时如此)。某类猜测是值得我们加以研究并认真对待的,例如下列猜测:即那些在我们仔细考虑并真正理解了我们所确实感兴趣的问题之后而提出的猜测。虽然这样的猜测往往很少表明全部真理,
但却至少包含一部分真理。如果我们适当地研究这样一种猜测,我们就有可能提炼出整个真理。
在许多情况下,猜测最后被证明是错误的,但它们在诱导出一个更好的猜测方面仍然是有用的。
除非我们没有鉴别能力,否则任何念头都不会真正地坏。如果我们根本就没有念头,那才真正糟糕。
(1)别这样。这里是一个有关约翰·琼斯先生的典型故事。琼斯先生在一家公司里工作。他曾经希望能长点工资。但他的希望(就象希望常常那样地)落空了。他的某些同事的工资增加了,而他的工资依然照旧。琼斯先生对此不能无动于衷。他烦恼不已,并且最后他猜疑布鲁恩董事应对他的不提升一事负责。
我们不能责备琼斯先生产生这样一个怀疑。确实有某些迹象是与布鲁恩董事有关。真正的错误是琼斯先生在有了那个怀疑以后,对所有相反的迹象视而不见了。他因固执地相信布鲁恩董事就是他的敌人而庸人自扰,他的所作所为愚不可及,以致他几乎使该董事成为他真正的敌人。
约翰·琼斯先生的毛病在于他的行为和我们大多数人一样。他永不改变他的主要意见。他改变他的次要意见倒不少见,并且十分突然;但他对他自己的意见,无论大小,从不怀疑,并且坚持到底。他从不怀疑他自己的意见,不问自己这些意见对不对,也从不严格检验这些意见——如果他了解严格检验意味着什么的话,他将会格外憎恨严格检验。
让我们承认约翰·琼斯先生在一定程度上无可非议。他很忙,他在公司里和在家里都有任务,他很少有空来怀疑或检验。充其量,他只能检验他信念中的一小部分,如果他没有时间检验上述那个怀疑,他为什么应当检验呢?
然而我们仍然要奉劝各位,莫学约翰·琼斯先生。莫要让你的怀疑,你的猜测,或者你的推想不经检验而继续增长以致根深蒂固。从理论上说,在任何情况下,最好的念头也会因不批判地加以接受而受损,因严格检验而兴盛。
(2)一个数学例子。在周长一定的所有四边形中找出面积最大的一个。
未知是什么?四边形。
已知是什么?四边形周长一定。
条件是什么?所得四边形的面积应大于任何周长相同的其他四边形的面积。
这个问题和通常的初等几何问题很不相同,所以我们自然要从猜测开始。
哪种四边形可能具有最大面积?最简单的猜测是什么?我们可能听见过在所有具有相同周长的图形中,圆的面积最大;对于这种说法似乎可信,我们甚至可以猜出某些理由。现在,哪种四边形最接近于圆?从对称性来看,哪一种最接近于圆?
正方形是显而易见的一种猜测。如果我们认真对待这个猜测,我们应当弄清楚它意味着什么?我们应有勇气写出:“在所有周长一定的四边形中,正方形的面积最大。”当我们决定检验这个命题时,情况就变化了。本来我们的问题是个“求解题”,在正式提出我们的猜测以后,我们的问题变成“求证题”了。
我们必须证明或推翻已形成的“定理”。
如果我们不知道任何早已解决的、和我们当前问题相类似的问题,我们可能会感到我们的任务很棘手。如果你不能解决所提出的问题,首先尝试去解决某个与此有关的问题。你能解决问题的一部分吗?我们可能想起,如果在四边形中,正方形与众不同,那么根据同一事实,它必定在长方形中也与众不同。如果我们能证明下述命题“在所有周长一定的长方形中,正方形的面积最大”,
那么我们将解决我们问题的一部分。
这个定理看起来比前一定理更好着手;当然,它较弱。无论如何,我们应当弄清楚它们是什么意思;我们应当有勇气更详细地去重新说明它。用代数语言去重新表述它一遍是有好处的。
邻边为a与b的长方形面积是ab。
与上述长方形具有相同周长的正方形的一边是 2 ba + 。这样,正方形的面积是( 2 ba + )2。它应大于长方形的面积,于是我们有
( 2 ba + )2 > ab
这成立吗?这一推断可写成其等价形式
a2+2ab+b2>4ab
这是成立的,因为它等价于
a2-2ab+b2>0或
(a-b)2>0
此不等式肯定成立,除非a=b,即被审核的长方形是个正方形。
我们尚未解决我们的问题,但由于我们敢于面对我们颇为明显的猜测,我们得到某些进步。
(3)一个非数学的例子。在某个组字的字谜中,要求我们找出有七个字母的字,关于这个字的线索是:“Do the walls again,back and forth(重新砌墙,前与后)。”
未知是什么?一个字。
已如是什么?字长一定,七个字母。
条件是什么?在这个字的线索中说明字与砌墙的关系,但仍很含糊。
因此,我们必须重新研究这个字的线索。当我们这样做时,可能会注意到线索的最后一部分:“…again,back and forth。”你能解决问题的一部分吗?
这里有个机会去猜测字首。由于如此强调其重复性,这个字很可能是以“re”
开头的。这是个显而易见的猜测。如果我们想相信它,我们应当琢磨它的含义。
要找的字可能是下述模样,
RE……。
你能检验这些结果吗?如果字谜的另外一个字和我们刚才所考虑的字在第一个字母上交叉,那么该字以R开头。我们转到该字并检验R,这是个好念头。如果我们检验R获得成功,或者至少没有发现任何反对它的理由,我们就回到我们原来的字上。我们再次问:条件是什么?当我们重新研究线索时,我们可能注意其最后面一部分:“…back and forth”。它的含义是否为:“我们所要找的这个字不仅可以由前向后读,而且也可以由后向前读?”这是一种不那么明显的猜测[但却有这样的情况,见“分解与重新组合”一节,(8)]。
无论如何,让我们正视这种猜测;我们来体会其含义。字的模样可能如下,
RE…ER,
除此以外,第三个字母应和第五个字母相同;它很可能是个辅音,而第四个字母(即中间的字母)是个元音。
现在读者可以很容易猜出这个字了。如果还不行,他可以试试所有的元音,
一个接一个地放在字的中心字母位置上。
24.图形
图形不仅是几何问题的对象,而且对于解答所有各类问题都有很大的帮助,即使初看起来问题与几何无关。这样,在解决问题方面,我们有两种原因要考虑图形。
(1)如果问题是个几何问题,我们必须考虑图形。此图形可以存在于我们的想象之中或是画在纸上。在某些情况下,可能希望对这图形只加以想象而不必画出它来。但如果我们需要逐个地审查各个细节,则希望画张图。如果细节很多,我们不能把所有的细节都同时想象出来,但它们却可以同时呈现在图上。
想象的细节图形可能会忘记;但画在纸上的可以保留,当我们再一次看图时,
它可以使我们想起以前的见解,这可省去不少我们重新回忆以前的各种考虑的麻烦。
(2)现在我们专门讨论图形在几何作图问题中的应用。在开始研究几何作图问题时,可以画张含有未知数和已知数的草图,所有这些元素都按照问题条件的规定来配置。为了清楚地理解问题,我们必须分别考虑每一个已知数和每一部分条件;然后我们把所有各部分重新统一起来,把条件当作一个整体来考虑,力图同时看到问题所需要的各种联系。没有一张画在纸上的图,我们几乎不能处理、分开与重新组合所有上述细节。
另一方面,当我们已最终解出问题以前,能不能画出这样一张图仍然是个疑问。问题所提出的全部条件是否可能满足?在我们得到确定的解答以前,我们没有资格回答:“是”,虽然我们开始时假定过一张图,其中未知数和已知数的关系满足条件的规定。在画图时,我们作出了似乎没有保证的假定。
不,不是这样。并非一定如此。只要我们没有把可能性与必然性混为一谈,
那么当我们在审核我们的问题时考虑下述可能性没有什么不对,这种可能性就是:存在某个满足未知数条件的对象并且它和全部已知数都具有所需要的关系。
当法官询问被告时,他考虑被告犯了所谈到的罪行,这种假定并没有什么不对,
只要他并不完全陷入这种假定就行。如果数学家和法官都等到其审核取得某种肯定结果以后才作出自己的判断。那么他们就可以毫无偏见地对可能性进行审核。
在研究作图问题时,假定条件均满足,画张草图,再开始研究问题这一方法可追溯于希腊几何学家。帕扑斯有一句简短而有点莫测高深的话说:“设欲为之者如已为之(Assume what is required to be done as already done)”
以下的建议不那么简练但更清楚:“画张假想的图,图中假定问题的所有各个部分都满足条件。”
这是针对几何作图问题所作的建议,但实际上并不需要局限于任何如此特殊的一类问题。我们可以把它拓广到“求解题”,并采用下列一般形式的陈述方式:“假定问题的条件完全满足,然后对此假设情况加以审核。”
请与“帕扑斯”一节的(6)作比较。
(3)我们对实际画图中的几点进行讨论。
(I)我们画图应当用仪器画得很精确,还是随手近似画画?
两种图形都各有用途。原则上,精确的图形在几何中所起的作用就象精确测量在物理中所起作用一样;但在实际中,精确图形不如精确测量重要,因为几何定理比物理定律得到更广泛的证实。初学者作图时则应尽可能精确,以便能得良好的实验基础;而精确图形对于更高明的人来说,也可能启发出几何定理。但是对于论证来说,随手细心画的图形通常已足够了,而且这样做要快得多。当然,这样的图不应当看起来荒诞不经;假定线是直的就不应画成弯弯曲曲的,圆不能画成个土豆。
一个不精确的图形有时可能给出错误的结论,但这危险不大,而且可以用各种方法加以避免,特别是用变化图形的方法。如果我们集中注意力于逻辑联系,并且认为图形是一种辅助工具,决非我们结论的基础,而真正的基础是逻辑联系,那就不会有什么危险【这一点可由某些众所周知的悖论富有启发性地加以说明,这些悖论巧妙地利用了图形上故意画得不准确之处】。
(II)重要的是元素必须按所需要的关系来配置,至于按什么顺序来作图倒并不重要。所以可选最方便的顺序。例如,在说明三等分时,我们需要画两个角α及β,使α=3β。如从任意一个α开始,我们不能用直尺和圆规作出β。
因此,你选择一个颇小的但却是任意的β,从任意的β开始,你作α是容易的。
(III)你的图形不应给出任何不恰当的特殊情况。图形的各部分不应当呈现出问题所未要求的明确关系。除非已有规定,线段不应看起来相等,或者垂直。如果三角形不是等腰的或直角的,就不能画得象等腰三角形或直角三角形。
三个角为45°,60°,75°的三角形,在文字的精确意义方面而言,是和等腰三角形与直角三角形的形状差别最大的*。如果你想考虑一个“一般的”三角形,
你可以画这样的三角形,或跟它差不多的三角形。
——————————
*:如三角形的三角为α,β,γ,且90>α>β>γ,则在90。-α,α-β,β-
γ三个差之中,至少有一个小于15°,除非“α=75°,β=60°,γ=45°。事实上,易证恒等式
156 )()(2)90(3 =?+?+? gbbaa
成立。由此式显然可推得上述结论。
(IV)为了强调不同线段的不同地位,你可以使用粗线或细线,实线或虚线,
或者用不同颜色的线。如果你尚未完全决定采用某一根线作辅助线的话,你就轻一点画它。你可以用红笔画已知元素,而用其他的颜色来强调重要的部分,
例如一对相似的三角形,等等。
(V)为了说明立体几何,我们应当用三维立体模型,还是画在纸上或黑板上的图形?
立体模型好,但不易制作而且售价昂贵。所以,通常我们只能满足于画图,
虽然它们不容易使人印象深刻。使用自制厚纸板模型的某种试验是非常合乎初学者的需要的。把我们日常周围的事物用来表达几何观念是有帮助的。这样,
一个匣子,一块瓷砖,或者一个教室,都可以用来说明长方体,一根铅笔可以用来说明圆柱,一个灯罩可以用来说明正圆台,等等。
(4)在纸上画图比较容易,也容易识别,容易记忆。我们特别熟悉平面图,
关于平面图形的问题也特别容易着手。我们可以利用这点优越性:当我们设法找到那些非几何对象的合适的几何表示时,我们就可以在处理非几何对象时利用我们处理图形的能力。
实际上,几何表示、各种图形和表格都用于所有的科学中,不仅用于物理、
化学以及自然科学,而且也用于经济学,甚至用于心理学。我们应当使用某种适当的几何表达方式,力求用图形的语言来表达一切事物,把所有各类问题简化为几何问题。
这样,即使你的问题不是几何问题,你也可试试去画张图。对于非几何问题,去找出一个清晰的几何表达方式,可能是走向解答的重要一步。
25.普遍化
普遍化就是从考虑一个对象过渡到考虑包含该对象的一个集合;或者从考虑一个较小的集合过渡到考虑一个包含该较小集合的更大的集合。
(1)如果我们碰巧遇到下列和数
1+8+27+64=100
我们可以看出,它可表示成这样的奇特的形式
13+23+33+43=102
现在,我们自然要问:是否连续自然数的立方和
l3+23+……+n3
都是一个自然数的平方?我们提出这个问题,就是把问题普遍化了,这次普遍化是很凑巧的,它从一次观测到的事实引出显然普遍的规律。在数学、物理及自然科学中,许多结果是由这种凑巧的普遍化而发现的。参见“归纳与数学归纳法”一节。
(2)在求解题中,普遍化可能很有用。我们考虑下列立体几何问题:“给定一直线与一正八面体的位置。求过已知直线并二等分已知八面体体积的平面。”这个问题可能看起来很难,但事实上,稍稍熟悉正八面体的形状就足以提出下列更普遍的问题:“给定一直线与一个具有对称中心的立体的位置。求过已知直线并二等分已知立体体积的平面。”所求的平面当然经过立体的对称中心,所以这个平面应由这点与已知直线所硫定。由于八面体有对称中心,所以我们原来的问题也就迎刃而解了。
读者不会看不到第二个问题比第一个更普遍化,但却比第一个容易得多了。事实上,我们解第一个问题的主要成就是创遗了第二个问题。创造第二个问题时,我们承认了对称中心的作用;我们把八面体的性质(它对目前这个问题很重要):即它有这样一个中心这个性质剖析出来了。
更普遍的问题可能更易求解,这看来矛盾,但前例已说明确实有这样的情况。在解决特殊问题方面的主要成就是创造出普遍化的问题。在找出普遍化的问题后,剩下的工怍只是一小部分了。因此,在我们的例中,普遍化的问题的求解只是求解特殊问题工作量中的一小部分。
见“发明家的矛盾”一节。
(3)一个棱台的底为正方形,下底边长10英寸,上底边长5英寸,高为6英寸,求它的体积。“如果将字母代替数字10,5,6,例如用a,b,h,这就是把问题普遍化了”。我们得到一个比原来更普遍化的问题如下:“一个棱台的底为正方形,下底边长为a,上底边长为b,高为h,求它的体积。”这样的普遍化可能非常有用。从“数字”题过渡到“字母”题可以得到一个新程序;我们可以变化已知数据,而这样做,我们可以用各种方法检验我们的结果。
参见“你能检验这结果吗?”一节第(2)点和“问题的变型”一节第(4)点。
26.你以前见过它吗?
很可能,我们以前曾经解决过我们现在必须解决的相同问题:或者我们以前听到过它,或者我们有个很类似的问题。我们不应该放弃探索这些可能性。
我们试图回忆发生过什么。你以前见过它吗?或者你是否见过相同的问题而形式稍有不同?即使回答是否定的,这类问题也可能起到开始调动有用知识的作用。
作为本节标题的问题经常用于更普遍的意义。为了得到解答,我们必须从我们的记忆中汲取有关的知识,我们必须调动起我们记忆中处于休眠状态的知识的有关部分(“进展与成就”)。当然我们事先不知道哪部分知识有用,但是存在可能性,我们不应放弃探索。这样,在求解某一个别的问题时,曾经起过作用的特点在解决当前问题中还可能起作用。所以,当前问题的任何特点,如果它给我们的印象表明它可能是重要的,那么我们就应当尝试去认识它。它是什么?你熟悉它吗?你以前见过它吗?
27.这里有个与你的问题有关且早已解决的问题
这是个好消息;有个问题的解答是已知的,且与我们当前的问题有联系,
这当然值得欢迎。如果这联系是紧密的并且解答很简单,那就更受欢迎了。很有可能,这样一个问题可用来解决我们当前的问题。
这里所讨论的情况是典型而重要的。为了看得更清楚,我们把它与利用辅助问题从而解决问题的情况相比较。在这两种情况下,我们的目标都是解决某个问题A,但我们却引人并考虑另一个问题B,希望它对解题A有利。差别在于对
B的关系不同。这里,我们是想到一个老问题B,我们知道它的解但不知道如何利用它。那里,我们是创造了一个新问题B,我们知道(至少是强烈地猜测到)
如何利用它,但目前还不知道如何解它。我们关于B的困难形成了两种情况问的所有差别。当我们克服了这个困难,在这两种情况下我们可以以同样的方式利用B;可以利用它的结果或方法[“辅助问题”一节第(3)点中有说明].而且如果我们运气好,可以同时利用它的结果和方法。在这里所考虑的情况下,我们对B的求解很清楚,但目前还不知道如何加以利用。因此,我们问:你能利用它吗?你能利用它的结果吗?你能利用它的方法吗?
利用某个早已解决的问题的意图会影响我们对当前问题的理解。试图把新老两个问题联系起来,我们就在新问题中引入一些元素,它们对应于老问题中某些重要元素。例如,我们的问题是确定外接于已知四面体的球。这是个立体几何问题。我们可能回忆起,我们解过平面几何的类似问题,即作已知三角形的外接圆。于是,我们回想起,在平面几何这一老问题巾,我们曾使用过三角形的边的中垂线。在我们当前这个问题中,应当试图引入某些类比的东西。这样就可能使我们想到在当前问题中引入四面体的中垂面作辅助元素。有了这个念头,仿照平面几何中的类比问题,我们就很容易得到立体几何问题的解。
上例是典型的。考虑一个早已解决的有关问题,这导致我们引入辅助元素,
而引入适当的辅助元素使我们有可能在解决当前问题中充分利用有关问题。为此,当我们思考如何利用一个早已解决的有关问题对,我们问:“为了可能利用它,你是否应该引入某个辅助元素?”
这里有一个定理与你的问题有关且早已证明过。这是上述讨论的变型,在第19节将举例说明。
28.探索法
它曾是某个学科分支的名称,尚未清楚地给予定义,属于逻辑学、哲学或心理学,常常很简略,很少详细说明,而且如今实际上已被忘怀。它的目的是研究发现和发明的方法和规律。在欧几里得几何评论家那里,我们可以找到这类研究的片言只语;在这方面帕扑斯的一段话尤其有趣。在试图建立探索法体系方面最著名的是笛长尔和莱布尼兹,这两人都是伟大的数学家和哲学家。伯纳德·波尔查诺曾提出关于探索法的著名详细说明。本书企图用朴素而现代化的形式来复兴探索法。见“现代探索法”一节。
作为形容词,heuristic的意思是“有助于发现的。”
29.探索式论证
探索式论证不是最终的和严格的论证,仅是临时的和似乎为真的,其目的是去发现当前问题的解。我们经常不得不使用探索式论证。当我们得到完整的解以后,我们得到完全的肯定性,但在得到这种肯定性以前,我们经常只能满足于多少有些似乎为真的猜测。在得到最终论证以前,我们可能需要临时论证。
当我们构造严格的论证时,我们需要探索式论证就象盖房子需要脚手架一样。
参见“进展的标志”一节。探索法论证常以归纳或类比为基础;参见“归纳与数学归纳法”和“类比”,(8),(9),(10)*。
————————
*:也可参见作者在《美国数学月刊》第48卷450 — 465页上的一篇文章。
探索法论证本身是好的,但把探索法论证和严格论证混为一谈却很不好。
而更坏的是把应该严格证明之处却用了探索法论证。
有蝗题目的教学中,特别是对工程师和物理学家教授微积分时,如果对探索浊论证的性质更好地加以理解,对它的局限性和长处坦率地加以承认,并且如果教科书公开提出探索法论证,那么教学可以显著改进。得体而直接了当地提出探索法论证可能是有用的;它通常含有严格论证的某些萌芽,从而可能为严格论证作准备。但是,如果这种论证提得模模糊糊不那么理直气壮,那它多半是有害无益的。参见“为什么要证明?”一节。
30.如果你不能解决所提出的问题
如果你不能解决所提的问题,这时别让失败太影响你,应在某个较容易获得的成功中去尝试寻求安慰,尝试去首先解决采个与此有关的问题;从这里获得勇气再去研究原来的问题。当原问题看来不可解,你不要忘记人类的高明之处就在于会迂迥绕过不能直接克服的障碍,就在于能想出某个适当的辅助问题。
你能不能想出一个更好着手的有关问题?现在你应该发明一个有关的问题,不仅仅是回忆起一个。对于后者,当我们问:“你如道一个有关的问题吗?”
的时候,我们已经尝试过了。
在我们的表中,以本节标题开始的那一段的其余问题都有一个共同的目的:变化问题。变化问题有各种办法,如“普遍化”、“特殊化”、“类比”
以及其他属于“分解与重新组合”的各种办法。
3I.归纳与数学归纳法
归纳法是通过对特例进行观察与综合以发现一般规律的过程。它用于所有科学甚至数学。数学归纳法则仅在数学中用以证明菜类定理。从名称上看,二者有联系,这点毋宁是不幸的,因为二者在逻辑方面的联系极少。不过二者之间还有某种实际联系;我们常把两种方法一起使用。我们将用同一个例子来说明这两种方法。
(1)我们可能碰巧看到
1+8+27+64=102
于是,把各个数写成平方和立方,我们可以把上述事实写成更有趣的形式,
13+23+33+43=102
怎么会有这样的事发生?是否经常有这样的事,即连续自然数的立方和是一个自然数的平方?
我们提的这个问题很象博物学家所提的。博物学家看到一个奇特的植物或者稀有的地质层,会设想一个普遍的问题。我们的普遍问题是关于下列连续自然数的立方和的,
13+23+33+…+n3
我们是由特例n=4引出这个问题的。
对于这个问题,我们能作什么呢?博物学家将怎么办?我们可以研究其他特例。特例n=2,3更简单,下一个特例是n=5。为了一致与完整,我们加上n=1的情况。将所有情况排列整齐,就象一位地质学家把他某种矿的所有标本排列起来一样,我们可得下表,
很难令人相信,所有连续自然数的立方和是某个自然数的平方,这一事实仅仅是巧合而已。在类似的情况下,博物学家对于这个由观察特例所得到的一般规律的正确性将很少怀疑;一般规律几乎总由归纳所证明。虽然数学家基小上用同一方式思考,但他在表达时却有更多的保留。他会说,下列定理是由归纳有力地提出的,
前n个自然数的立方和是一个自然数的平方。
(2)我们已被引导到猜测一个值得注意的、有些神秘的定律。为什么n个连续自然数的立方和应该是一个自然数的平方?不过,显然,它们是某个自然数的平方。
博物学家在这种情况会怎么办?他将对他的猜测进行检验。在这样做时,
他可以遵循各种研究路线。博物学家可能积累更多的实验证据;如果我们也希望这样做,我们必须检验下列情况:n=6,7,…。博物学家也可能重新检验引起他猜测的那些事实,他把这些事实小心地加以比较,尝试去剖析出更深刻的规律性,某种更进一步的类比。让我们遵循这条研究路线。
我们重新研究表中所列出的n=1,2,3,4,5等情况。为什么所有这些和都是某个自然数的平方?关于这些平方,我们能说些什么?它们的底是1,3,6,10,
15。关于这些底数有什么情况呢?有没有某个更深刻的规律,某种更进一步的类比?不管怎样,它们不象是增加得很不规则。它们是怎样增加的?这个系列中相邻两项之差本身也是增加的,
3-1=2,6-3=3,10-6=4,15-10=5
这些差数显然很有规律。这里,我们在耶些平方的底数之间,可以看出惊人的类比,在数字1,3,6,10,15中,我们可以看出明显的规律性,
1=1
3=1+2
6=1+2+3
10=1+2+3+4
15=1+2+3+4+5
如果这种规律性是普遍的(而相反的情况是难以置信的),则我们所猜测的定理将取一个更精确的形式:即,
对于n=1,2,3,…
13+23+33+…+n3=(1+2+3+…+n)2。
(3)刚才所述定律是由归纳法所发现的,其发现方式告诉我们关于归纳的概念,这种概念必然是单方面的,不完全的,但并不是歪曲的。归纳法企图去找出所观察事物背后的规律性与统一性。它最明显的手段是普遍化、特殊化和类比。试验性的普遍化从努力了解所观测到的事实开始;它以类比为基础,并由更进一步的一些特例加以检验。
关于归纳这个论题,哲学家们有很多分歧,我们就此止步,不再评论。但应该补充一点:许多数学结果是首先由归纳所发现,以后再加以证明的。已严格地提出来的数学是一门系统的演绎科学,而正在形成过程中的数学是一门实验性的归纳科学。
(4)我们在数学中可以象物理科学中那样利用观察与归纳去发现普遍规律。但有区别。在物理科学中,比观察与归纳更高的权威是没有的,但在数学中却有这样一个权威:严格的证明。
我们用实验观点工作片刻之后,改变一下我们的观点可能是有益的。让我们严格起来。我们已发现一个有趣的结果,但其论证仅仅是似乎有理的,实验的,假设的,探索法的;让我们尝试用严格的证明来把它肯定地建立起来。
现在我们面临一个“求证题”:证明或推翻上述结果(见上述2)。
这里有个小简化。我们也许知道
1+2+3+…+n= 2 )1( +nn
不管怎么样,这是容易证明的。取一长方形,其边为n与n+1,用波折线把它分成两半,如图18a所表示的是n=4情况。每一半都是“阶梯形”的,而其面积的表达式为1+2+…+n;当n=4,它是1+2+3+4,见图18b。现在整个面积是 n(n+1),
而阶梯形面积仅是上述面积之一半;于是上述公式得证。
图18
我们可以将归纳所得之结果变换为
13+23+33+…+n3=( 2 )1( +nn )2
(5)如果我们想不出如何证明这结果,我们至少可以检验它。让我们检验我们尚未检验过的第一个情况,n=6。这时有
l+8+27+64+125+216=(6X7/2)2
经过计算,这是正确的,两边都等于441。
我们可以更有效地检验上述公式。这公式很可能普遍成立,即对n的所有数值都成立。当我们从任何n值过渡到下一个值n+1时,此公式是否仍成立?按照前面所写的公式,我们也应当有
13+23+33+…+n3+(n+1)3 =[ 2 )2)(1( ++ nn ]2
现在作简单的检验。从上式减去前页的公式,得
(n+1)3=[ 2 )2)(1( ++ nn ]2—[ 2 )1( +nn ]2
这公式很容易检验。右侧可写成
( 2 1+n )2[(n+2)2-n2]=( 2 1+n )2[n2+4n+4-n2]=( 2 1+n )2(4n+4)
=(n+1)2(n+1)=(n+1)3=左侧
于是我们由实验所发现的公式通过了极为重要的检验。
让我们弄明白这个检验意味着什么。我们已经毫无疑问地证明
(n+1)3=[ 2 )2)(1( ++ nn ]2-[ 2 )1( +nn ]2
但现在我们还不知道
13+23+33+…+n3=( 2 )1( +nn )2
是否成立。如果我们知道它成立,则加上方才我们毫无疑问地证明的那个等式,
我们就可推断
13+23+33+…+n3+(n+1)3 =[ 2 )2)(1( ++ nn ]2
也成立。它就是对下一个整数n+1的情况的推断。现在我们确实知道,对于n=1,
2,3,4,5,6,我们的猜测是成立的。根据我们方才所说的,这个猜测在n=6
时既然成立,则当n=7时也必成立。在n=7时成立,则在n=8时也成立;在n=8时成立,则在n=9时也成立;如此等等。它对所有n都成立。这就证明了它普遍成立。
(6)上述证明在许多类似情况下可作为一个模式。这个模式的主要思路是什么?
我们需要证明的推论必须预先以精确的形式给出。
此推论必须与整数n有关。
此推论必须相当“明显”的,使得我们有某种可能检验它从n过渡到下一个整数n+1时是否仍然成立。
如果我们在有效地检验这点方面获得得成功,我们有可能利用检验过程中所得到的经验来得出结论:如果此推论对 n成立,则它对n+1也必成立。只要上述结论成立,我们知道当n=1,此推论成立就足够了,于是n=2成立,于是n=3
成立,等等;从任一整数过渡到下一个整数,我们就普遍化地证明了此推论。
上述过程是常用的,它应该有个名字。我们可以叫它“从 n到n+1的证明”
或更简单些,“过渡到下一个整数”。遗憾,我们所必需证明的精确推断可来自任何来源,从逻辑观点看来,来源如何,无关紧要。现在,在许多情况下,
例如方才所详细讨论的情况,其来源是归纳,推论是用实验方式找到的,冈此证明象是归纳的一种数学补充;这是对上述名称的一点解释。
(7)这里还有一点,虽然微小,但对任何期望自己去证明的人来说却很重要。在前面,我们通过观察与归纳,找到两个不同的推论,一个在第1点一个在第2点;后者比前者更精确。在讨论第二个推论时,我们发现有检验从n过渡到
n+1的可能性,于是我们能够用“数学归纳法”找到一个证明。如果讨论第一个推论而不考虑由第二推论所添加的精确性,我们就几乎不可能找到这样一个证明。事实上,第一个推论不及第二个精确、“明显”、“明确”、易试验和易检查。从第一推论过渡到第二推论,从比较不精确的陈述过渡到精确的陈述是为最后证明所作的一种重要准备。
上述情况有矛盾的一面。第二个推论更强些;它直接蕴含着第一个,而多少有点“含糊”的第一个推论却几乎不蕴含更“清晰”的第二个推论。这样,
更强的定理却比较弱的更易掌握;这是“发明家的矛盾”。
32.发明家的矛盾
雄心大的计划,成功的希望也较大。
这看起来矛盾。但当从一个问题过渡到另一个,我们常常看到,新的雄心大的问题比原问题更容易掌握。较多的问题可能比恰好只有一个问题更容易回答。较复杂的定理可能更容易证明,较普遍的问题可能更容易解决。
如果我们更仔细地看几个例子(“普遍化”一节,第(2)点;“归纳与数学归纳法”一节,第(7)点),那么这矛盾就消失了。雄心大的计划可能有更多的成功机会,如果它不是仅仅根据浮夸,而是立足于看到某些表面以外的东西。
33.能满足条件吗?
是否可能满足条件?条件是否充分,足以确定未知数?或者它不充分吗?或是多余吗?或是矛盾吗?
这些问题在早期阶段经常用到,那时我们并不需要最后的答案,只需要一个预想的答案,一个猜测,例如,见第8,18节。
能预见到我们所追求的结果的任何特点都是有益的。当我们对于我们能预测的事物有了某种概念以后,我们对于我们应当按什么方向前进就知道得更清楚了。现在,问题的一个重要特点是它所允许的解的数目。最令人感兴趣的问题是那些恰有一解的问题;我们倾向于把具有唯一确定解的问题当作仅有的“合理”问题。在这种意义上,我们的问题是否“合理”?如果我们能回答这个问题,
即使是一个似乎有理的猜测,我们对这问题的兴趣就有所增长而我们就能工作得更好。
我们的问题“合理”吗?如果我们能轻而易举地加以回答,那么它在我们工作的早期阶段是有用的。如果不好回答,那么在寻求回答的过程中产生的烦恼可能会超过所得到的兴趣。“可能满足条件吗?”这一问题以及我们的表中有关这方面的其他问题也是如此。当回答这些问题可能容易而且似乎有理时,我们应当提这些问题。但当回答看来很难而且含糊不清时,就应当放弃这些问题。
对于“求证题”,相应的问题是:命题是否可能成立?或者很可能命题不成立?提问题的方式清楚地表明我们只期望有一个猜测,一个似乎可信的临时性回答。
34.莱布尼兹
他是大数学家与哲学家。他计划写一篇“发明的艺术”,但并没有实现。
散见在他的著作中的许多片断表明,他对于他常常强调其重要性的题目怀有令人感兴趣的想法。这样,他写道:“没有什么比看到发明的源泉更重要的了。
就我看来,它比发明本身更有趣。”
35.引理
引列的含意是辅助定理。它来源于希腊语,可直译为“假设的什么”。
我们打算证明一个定理,比如说,是A。我们被引导去猜测另一个定理,
比如说,是B;答B为真,我们也许能用它来证明A。我们暂时假定B成立,把它的证明推迟一下,而先去证明A。这样一个假设的定理B就是原来所提定理的辅助定理。我们这段简短的叙述相当典型,它阐明了“引理”一字现在的含意。
36.看着未知数
这是句老生常谈式的忠告;其相应的拉丁格言是:“respice finem”。
即,看着终点。记住你的目的。勿忘你的目标。想着你希望得到的东西。不要看不见你所需要的。记住你为什么而工作。看着未知数。看着结论。这是“看着终点”的两种不同的说法,它们特别适合于数学问题,分别适用于“求解题”
和“求证题”。
集中注意力于我们的目标,集中意志于我们的目的,我们就会想出达到它的方式和方法。达到目的的方法是什么?你怎样达到你的目的?你怎样才能得到这类结果?什么原因会产生这样一个结果?你在哪里看见过这样一个结果?为了得到这样一个结果,人们通常怎么办?于是尝试想起一个具有相同或相似未知数的熟悉的问题。尝试想起一个具有相同或相类似结论的熟悉的定理。和前面一样,上述两种说法分别适合于“求解题”和“求证题”。
(1)我们将考虑数学问题,一个“求解题”,并且建议:尝试想起一个具有相同未知数的熟悉问题。让我们将此建议与下一问题中所包含的建议相比较:
“你知道一个与此有关的问题吗?”
后一个建议比前一个更具有一般性。如果一个问题与男一个有关,则这两个问题有某些共性;它们可能包含着若干共同的对象或概念,或有某些共同的已知数据,或某一部分条件是共同的,等等。第一个建议要求具有这样一个特殊的共同点:两个问题应具有相同的未知数。就是说,两种情况下未知数是同一类对象,例如,都是直线的长度。
这个特殊化建议与一般化建议相比,有某些经济节约效果。
首先,在表达问题方面,我们可能节约些精力;我们不必立划看着整个问题,而只须注视未知数。问题可以看成如下形式
“已知……,求线的长度。”
其次,在选择方面也有某些节约。与所提问题有关或有共性的问题可能多如牛毛。但注视未知数,我们的选择就有限了;我们只考虑具有相同未知数的一类问题。此外,在具有相同未知数的问题中,我们当然首先考虑对我们来说是最初等的与最熟悉的问题。
(2)我们面前的问题具有下列形式,
“已知……,求线的长度”。
现在,这类最简单与最熟悉的问题是关于三角形的:已知三角形的三个元素,求某一边的长度。记住这点以后,我们已经找到了某些。可能有关的问题:
这里有个与你的问题有关且早已解决的问题。你能利用它吗?你能利用它的结果吗?为了利用我们所熟悉的有关三角形的结果,我们必须在图中作一个三角形。
是否有个三角形?或者我们应该引入一个,以便利用我们所熟悉的结果? 为了能利用我们所熟悉的结果,你是否应该引入某个辅助元素?
有好几个简单问题的未知数是三角形的边(它们的已知数据彼此不同;已知两角一边或两边一角,并且角相对于已知边的位置可以不同。所有这些问题在直角三角形时尤为简单)。集中注意力于我们面前的问题,我们尝试找出应该引入哪类问题,哪个早已解决的问题(具有相同未知数的)最适合我们当前的目的。
在引入合适的辅助三角形时,可能会发生下述情况:我们尚不知道该三角形的三个元素。不过这并不绝对需要;如果我们预见到用什么办法能得到所遗漏的部分,我们就已经有了长足进步,我们有了一个求解的计划。
(3)上面(1)和(2)所提出的过程主要已由第10节的例子加以说明(该例中,
由于学生的迟钝,说明得不那么清楚)。我们不难增加许多类似的例子。事实上,
在不太高的班级中,几乎对于所有的“求解题”都可由下列建议开始解题:尝试想起一个具有相同或相似未知数的熟悉的问题。
我们必须按一定格式提出下列问题,并且首先看着未知数,
(I)已知……,求线段长。
(II)已知……,求角。
(III)巳知……,求四面体的体积。
(IV)已知……,在图中作此点。
如果我们在处理初等数学问题方面有些经验,我们将很快地想起某些简单而熟悉的问题,或者具有相同未知数的问题。如果所提问题不是简单而熟悉的问题,我们自然企图利用我们所熟知的东西,并从那些简单问题的结果得到好处。我们试图把某个有用而又众所周知的东西引入到问题中,而这样做可以使我们有个良好的开端。
在下面指出的四种情况下,每一种情况都有一个显而易见的计划,一个关于以后求解过程的似乎可信的猜测:(I)未知数应该作为某个三角形的一边而求得。需要引入一个适当的三角形,它具有三个已知的或容易求得的元素。(II)
未知数直该作为某个三角形的一角而求得。需要引入一个合适的三角形。(III)
如果四面体的底面积丢口高的长度为已知,则未知数可解。需要找出侧面积与相应的高。(IV)求作的点应该作为两轨迹的交点而求得,轨迹或是一圆,或是直线。需要从所提条件中找出这样的轨迹。
在所有上述情况,计划都是通过具有相同未知数的简单问题并要求利用其结果或方法而提出的。当我们实行这样一个计划时,当然可能会碰到困难,但我们有某种念头来开始着手工作,这是很大的优点。
(4)如果没有一个与所提问题有相同未知数且早已解决的问题,就不存在上述优点。在这种情况下,求解原问题将大为困难。
“已知球的半径,找出球面积”。这个问题是由阿基米德解决的。可以说,
几乎没有更简单的具有相同未知数的问题,而且那时肯定没有阿基米德可加以利用的这样一个更简单的问题。事实上,阿基米德的解答可以认为是最著名的数学成就之一。
“已知四面体的六条棱,求内切于四面体的球面积”。如果我们知道阿基米德的结果,则解决这个问题并不需要阿基米德那样的天才;留给我们去解决的只是用四面体的六条棱去表示内切球的半径。这虽不很容易,但其困难并不能与阿基米德的问题相比拟。
是否知道一个有相同未知数而且早已解决的问题,可能是造成难题和容易题之间全部差别的原因。
(5)我们刚才提过,当阿基米德求球面积时,他并不知道任何有相同未知数而且早已解决的问题。但他却知道各种有相似未知数而早已解决的问题。有些曲面的面积比球面积容易求,它们在阿基米德时代已为人所共知,如正圆柱体的侧面积,正圆锥体的侧面积,圆台的侧面积等等。我们可以肯定,阿基米德曾经仔细地考虑过这些较简单的相似情况。事实上,在其解答中,他利用了一个由两个锥体与若于个圆台所组成的复合体来作为球体的近似(见“定义”,
第(6)点)。
如果我们不能找到一个和我们当前问题的未知数相同且早已解决的问题,
那么,我们就去尝试找一个具有相似未知数的问题。对于我们当前的问题来说,
后者的联系要比前者为弱,因此,一般说来更不易为我们的目的服务,但无论如何,它们可能成为有价值的行动指南。
(6)我们补充一些有关“求证题”的说明,它们与前述“求解题”的大量说明相类比。
我们必须证明(或推翻)一个清晰陈述的定理。任何过去已证明并且在某方面与我们当前的定理有联系的定理都有可能对我们有用。但是,我们可以期望那些与我们当前定理有相同结沦的定理能提供最直接的帮助。知道这点,我们就看着结论,也就是说,我们考虑我们的定理时要强调其结论。我们注视定理的方式可以写成下列程式,
,如果……,则角相等”。
我们集中注意力于我们当前的结论,并尝试去想起一个具有相同或相似结论的熟悉的定理。特别是,我们尝试去想起一个极其简单而又熟悉的这类定理。
在这种情况下,有各种各样的这类定理,我们可能想起下面这个:“如果两三角形全等,则其对应角相等”。这里有一个与你的定理有关,且早已证明的定理。你能利用它吗?为了利用它,你是否应当引入某个辅助元素”?
遵照上述建议,并且考虑我们所想起的定理对我们能提供什么帮助,我们就可能设想出一个计划:根据全等三角形,去证明问题中的角的相等。我们看到我们必须引入一对包含该角的三角形并且证明它们全等。根据这样一个计划开始工作肯定不坏,并且它最终可能引导我们达到所需要的目的,如第19节所述。
(7)小结。我们回忆具有相司或相似未知数且早已解决的问题(具有相同或相似结论且早已证明的定理),就会有一个好机会沿正确方向开始我们的工作,
并且可能设想出一个解题计划。在简单情况下(这在低年级最为常见),具有相同未知数的最初等的问题(具有相同结论的定理)通常很多。尝试去重新回忆具有相同未知数的问题是个显而易见的常识性的手段(请与第4节中这方面的论述作比较)。令人惊讶的是:如此简单有用的手段并非普遍为人所知。作者倾向于认为:以前甚至还没有人对它作过一般性的、充分的论述。无论如何,不论是教师还是学生都不能不重视正确利用下列建议:看着未知数!并尝试想起一个具有相同或相似未知数的熟悉问题。
37。现代探索法
现代探索法力求了解解题过程,特别是解题过程中典型有用的智力活动。
它有各种信息来源,哪一个都不应忽略。认真研究探索法,应当同时考虑逻辑学与心理学的背景,不应忽视象帕扑斯、留卡尔、莱布尼兹和波尔查诺这些占老的作家关于这个论题所说的话,而且极不应忽视无偏见的经验。在解题方面的经验以及注意别人解题的经验,必须成为建立探索法的基础。在这种研究中,
我们不应忽视任何一类问题,并且应当找出关于处理各类问题所共有的特征来;
我们的目的应当是找出一般特征而与问题的主题无关。研究探索法有其“实际”
目的:对解题中典型有用的智力活动有更好的了解,这对教学,尤其是数学教学,会产生某些良好的影响。
本书是实现这种计划的首次尝试。我们将讨论此字典中的各个标题怎样配合我们的计划。
(1)我们的表实际上是一个在解题中典型有用的智力活动表;表中所列的问题与建议提示这类活动。有些活动在第二部分又重新加以叙述,其中有些在第一部分讨论和说明得更为详尽。
有关表中特殊问题与特殊建议的补充知识,读者应参见字典部分的那15条条文,其标题就是表中15个段落的第一句:“未知教是什么”,“可能满足条件吗?”,“画张图”,……,“你能利用这结果吗?”希望了解有关表中特定项资料的读者,应看一看表中包含该项的那一段段首的几个字,然后查找字典中以那几个字作为标题的条文。例如,“回到定义去”这条建议包含在表中的一段,它的第一句为:“你能重新叙述这问题吗?”在此标题下,读者可发现需要同时并行参考“定义”一节,在该节中,问题中的建议得到解释与举例说明。
(2)解题过程是个复杂的过程,它有好几个不同的方面。本字典中,有12
条主要条文较为详细地研究了其中某些方面,我们将在下面提到这些条文的标题。
当我们专心一致地工作时,我们强烈地感觉出工作的进展;当我们的进展迅速时,我们欢欣鼓舞;当进展迟缓时,我们抑郁寡欢。在解题过程中对于“进展与成就”来说重要的是什么?讨论这个专题的段落常常被字典的其他部分所引用,因此应当及早阅读。
当我们尝试求解一个问题时,我们轮流地考虑其各个方面;在心中不断地琢磨它;“变化问题”对我们的工作是必要的。我们变化问题可以通过“分解与重新组合”其元素,或通过回到某些术语的“定义”去,我们也可以利用有力的办法:“普遍化,特殊化和类比”。变化问题可能会把我们引导到采用“辅助元素”,或发现一个更好着手的“辅助问题”。
我们必须小心地区分两类问题:求解题和求证题。我们的表特别适合“求解题”。为了使此表也能用于“求证题”,我们必须加以修改,改变某些问题与建议。
在所有各类问题,特别是不太简单的数学问题中,应用适当的“符号”与几何“图形”大有裨益而又是经常不可缺少的。
(3)解题过程有许多方面,但其中有些在本书中完全未予考虑,有些则很简略。我们认为在一本小册子中这样做是合理的,因为那些内容有的看来过细,
有的显得技术性过强,有的则尚有争议。
在寻求解答的过程中,临时性的、仅仅是似乎可信的探索性论证是重要的,
但你不应该把它当成一个证明;你必须猜测.但也要“检验你的猜测”,探索性论证的性质放在“进展的标志”一节中讨论,但讨论还可进一步。
在我们的论题中考虑某些逻辑模式是重要的,但不引入任何技术性条文似乎是可取的。只有两条主要致力于心理学方面,即“决心,希望,成功”和“潜意识的工作”。还有一段关于动物心理学的附记,见“倒着干”。
我们强调这点,即各种问题,特别是“实际问题”,甚至“谜语”,都属于探索法范畴。我们还要强调指出:确实无误的“发明创造规律”并不属于认真研究的范围。探索法讨论人类面对问题时的行为。自从人类社会开始以来,
这种讨论大概一直是流行的,而这种古老讨论的精华看来都保存在“谚语的智慧”之中。
(4)由于教师或学生可能有特殊的兴趣,有几个特殊问题的条文包括进来了,有些更一般化的专题的条文则予以扩充。
有几条讨论方法论问题的条文在初等数学很重要,如“帕扑斯”,“倒着干”(在第三段已引述),“归谬法与间接证明”,“归纳与数学归纳法”,“建立方程”,“量纲检验”以及“为什么要证明?”。有些条文和教师尤其有关,
如“代公式问题”,“诊断”,还有其他一些则与较一般学生更富有雄心壮志的学生有关,如“聪明的解题者”,“聪明的读者”和“未来的数学家”。
这里不妨指出,在第8、10、18、19、20节以及字典的各条条文中所给出的教帅与学生的对话,不汉仅可作为范例供教师上课时参考,而且也可指导自学的解题者。把思想描述为“心灵会话”,描述为当事人和他自己的一种会话是恰当的。问答式的对话显示出求解过程中的进展情况;解题者自问自答时也可以沿着类似的路线前进。
(5)其余标题不打算一一列举了;只提几组如下,
有些条文包括了有关我们论题历史的一些说明,如“笛卡尔”,“菜布尼兹”,“波尔查诺”,“探索法”,“新老术语”,“帕扑斯”(这最后一条已在第4段小引过)。
少量几条条文解释了专业术语,如“条件”,“推论”,“引理”。
有些条文只给出并行参考的标题(在目录中记有【+】的条文)。
(6)探索法的目的在于其普遍性,在于研究与其课题无关而适用于所有各类问题的程序。然而本书的例子却几乎毫不例外地取自初等数学。不容忽视,
这是一种局限性。但我们希望这种局限性不致于严重损害我们研究的趋势。事实上,初等数学问题可提供全部我们所需要的多样性,而研究这些问题的求解特别容易着手,而且有趣。此外,我们虽然很少引用非数学问题作为例子,但并非完全置之脑后。我们从未直接引用较高深的数学问题,可是这些问题却构成了本书的真正的背景。对这类研究感兴趣的熟练数学工作者不难根据自己的经验补充例子,进一步阐明由初等数学例题所说明的各点。
(7)本书作者对下述几位现代作者谨致以谢意(他们的名字未列入“探索法”条文)。他们是物理学家兼哲学家马赫(Ernst Mach),数学家哈达马(Jacques
Hadamard),心理学家杰姆斯(William James)和克勒(Wolfgang K6hler)。作者还希望提及心理学家邓克(K.Duncker)和数学家克劳斯(F.Krauss),这两位的著作(有的著作是他们的研究工作取得相当进展以后发表的,有的是部分发表的著作)中有与我们相平行的一些内容。
38.符号
如果你想体会一个选择适当而众所周知的符号有什么好处,你可以试试把几个不太小的数相加,条件是不准用大家熟悉的阿拉伯数字,但如果你想写的话,你可以用罗马数字。例如取数字为MMMXC,MDXCVI,MDCXLVI,MDCCLXXXI,
MDCCCLXXXVII。
对数学符号的重要性我们几乎总是不会估计过高的。现在使用十进制符号的计算工作者比古代不能以如此方便的形式记数的计算工作者要沾光得多。在求解曾经考验过阿基米德天才的面积和体积问题上,一个熟悉代数、解析几何和微积分常用符号的普通现代学生较之古希腊数学家不知要沾多少光。
(1)说活与思维有密切联系,使用文字有助于思维,某些哲学家和语言学家还进一步断言:使用文字对于运用推理是必不可少的。
但后一说法未免有点言过其实。凡对严肃的数学工作稍具经验的人都知道:不用文字而只注视几何图形或仅演算代数符号也可以进行一些相当艰巨的思维。图及符号和数学思维有密切的联系,它们的使用有助于思维。我们可以改进哲学家和语言学家上面那段多少有点狭隘的断言,办法是把文字和其他各类符号并列而说成:使用符号对于运用推理看来是必不可少的。
无论如何,数学符号的使用类似于文字的使用。数学符号看来象一种语言,
一种构造良好的语言,一种非常适合其目的、简练而准确的语言,其规则与通常的语法不同:它毫无例外。
如果我们接受这个观点,“建立方程”就象是一种翻译一样,把普通语言翻译成数学符号的语言。
(2)某些数学符号,如+、-、=以及若干其他符号,具有固定的传统意义,
但其余符号,如大写小写的罗马字母与希腊字母,则在不同的问题中具有不同的意义。当我们面对一个新问题时,我们必须选择某些符号,我们必须引入适当的记号。这与使用普通语言有些类似。许多词在不同的上下文中有不同的意义;但在精确性很重要的场合下,我们必须小心选择我们的用词。
在解题中,选择符号是重要的一步。应谨慎从事。我们现在花费在选择符号上的时间,以后可由避免了狐疑不定和混乱而节省下来的时间所弥补。此外,
在小心选择符号时,我们必须把问题中需加符号的元素仔细想个明白。这样,
选择一个合适的符号可能大大促进了对于问题的了解。
(3)一个好符号应该是不含糊的、富有意义的、便于记忆的;它应该避免有害的第二重意义而利用有用的第二重要意义;符号的次序与联系应提示事物的次序与联系。
(4)符号首先必须不含糊。在同一研究中,不允许一个符号代表两种不同的对象。如果在解一道题时你把某个量叫做a,那么你应避免把与这同一问题相联系的任何其他量也叫做a。当然,在不同的问题中,a可以有不同的意义。
虽然禁止对不同的对象使用相同的符号,但并不禁止对同一个对象使用不同的符号。所以a与b的乘积可以写成
a×b a·b ab
在某些情况下,对同一对象使用两个或更多的不同符号是有好处的,但这时要特别小心。通常,一个对象只用一个符号较好,在任何情况下都不应随便使用好几个符号。
(5)一个好符号应便于记忆,并便于确认;符号应立即使我们想起对象是什么,而对象则使我们想到符号是什么。
使符号便于确认的一个简单办法是使用对象的第一个字母作符号。例如第
20节中,用r表示速率,用t表示时间,用V表示体积。 然而我们不能在所有情况下都用第一个字母。这佯,在第20节中,我们必须考虑半径但我们不能把它叫做r,因为这个字母已经表示速率了。还有其他的原因限制符号的选择,还有其他的手段使其便于确认,这些我后面将讨论到。
(6)当符号的次序与联系可向我们提示对象的次序与联系时,符号对于形成哉们的概念特别有用。现举例说明。
(I)为了表示在问题的概念中彼此接近的对象,我们使用字母表中彼此接近的字母。
这样,我们对于给定的量或常数,一般用英文字母表中开头的几个字母,
如a,b,c来表示。而对未知量或变量,则常用末尾的几个字母,如x,y,z来表示。
在第8节,我们用a,b,c表示长方体的已知的长、宽、高。在这种情况下,
符号a,b,c比用长、宽、高的第一个字母 l,w,h更可取,因为这三个长度在问题中起的作用相同,对比我们利用接连的三个字母加以强调。此外,正如我们刚才所说的,a,b,c是字母表中头三个字母,所以它们是表示已知量的最常用的字母。在另外一些情况下,如果三个长度所起的作用不同,并且知道哪些长度是水平的,哪个长度是垂直的,对于解题很重要,那么这时符号l,w,h
可能更可取。
(II)为了表示属于同一类的对象,我们常常对同类对象选用属于同一字母表中的字母,而对不同类对象选用不同字母表中的字母。因此,在平面几何中,
罗马字母如A,B,C,…常用于点,
小写罗马字母如a,b,c,…常用于线,
小写希腊字母如α,β,γ,…常用于角。
如有两个对象分属于不同的类,但彼此有特殊的联系(这一点对我们的问题很重要),我们可选不同字母表中相对应的字母来表示它们,如A,a,B,b
等等。一个大家熟悉的例子就是三角形的常见记法,
A,B,C代表顶点,
a,b,c代表边,
α,β,γ代表角。不言而喻,这里a是顶点A的对边,而A点的角叫做α。
(III)在第20节,字母a,b,x,y是特别精选的,以表明所指出的元素的性质与联系。字母a,b暗示所代表的量是常数;x,y表示变量;a在b前就如x
在y之前,而这暗示 a对b的关系与x对y的关系相同。事实上,a与x是水平的,b
与y是垂直的,而a:b=x:y。
(7)符号
△ABC∽△EFG表示问题中的两个三角形相似。在现代书籍中,此公式意味着两个三角形相似,其顶点按照符号的书写次序相对应,即A对E,B对F,C
对G。但在较古老的书籍中,并没有引入这种关于次序的附加规它;为了断定哪个顶点和哪个顶点相对应.读者必须看图或记住其推导情况。
现代记法大大优越于古老记法。使用现代记法,我们不必看斟就可由公式得出结果。这样,我们可推出
∠A=∠E
AB:BC=EF:FG
以及其他这类关系。而占老记法所表达的内容甚少,同时不能给出这样确定的结果。
如果一个符号比其他符号表达的内容更多,则可称为更“含义丰富的”。
相似三角形的现代记号就比古老符号含义丰富,它比古老符号更充分地反映了事物的次序和联系,所以它可能比古老符号为提哄更多成果奠定基础。
(8)字(或词)有第二重含义。通常包含一个字的某些上下文对这个字会有影响,并在其原义之外增加了一些东西,某些细微的差别,或第二重含义或“内涵”。如果我们写作时很小心,就会在许多几乎是同义字中选择一个字,其第二重含义最为贴切。
在数学符号中有类似的情况。即使是数学符号也可能从通常包含它们的上下文中得到某种第二重含义。如果我们小心地选择符号,我们必须考虑这点。
现在我们加以说明。
有些字母已获得根深蒂固的传统意义,如e通常表示自然对数的底,i表示
1?,即虚数单位,π表示圆周率。我们最好按传统的含义来使用这些字,如果我们把这样一个符号用于别的含义,则其传统的含义会随时干扰我们自定的含义,从而会引起麻烦,甚至会产生误解。诚然,尚未研究过多少课题的初学者比起对处理这类麻烦很有经验的数学家较少受这类有害的第二重含义的影响。
如果用得巧妙,符号的第二重含义也会有所帮助,甚至很有帮助。以前多次用过的符号可能帮助我们回想起某个有用的程序;当然,我们应当十分小心翼翼地把符号当前的(基本的)含义同其以前的(辅助的)含义清清楚楚地分开。
固定符号[如前面(6)(II)中所提到的三角形各部分用的传统符号]有很大的优越性,既然以前多次用过,它可以帮助我们回想起各种以前用过的程序。我们记忆公式时也用的是某个固定符号。当我们由于特殊情况不得不以一种不同往常的含义来使用固定符号时,我们理所当然地应给予充分的注意。
(9)当我们必须在两个符号中选出一个而又各有道理,莫衷一是时,我们需要凭经验与爱好来选出较合适的一个,正如我们写作时凭经验和爱好来选词汇一样。但是我们知道上面所讨论过的各种利弊仍然是有益的。无论如何,我们应该小心地选择符号,并且选择要有充分的道理。
(10)不但班级中最不可造就的孩子可能讨厌代数,甚至聪明过人的孩子有时也会对它反感。符号总不免有些武断和不自然;学习一种新符号对记忆是一种负担。如果聪明的学生不理解这种负担有什么好处,他就会加以拒绝。如果他没有充分的机会亲身体验到“数学符号语言有助于思维”,那么他讨厌代数是无可非议的。帮助学生获得这方面的经验体会是教师的重要职责,是最重要的职责之一。
我说,这是一个重要的职责但并非易如反掌。上面的说明可能对此有些帮助。还可参见“建立方程”一节。
我们推荐用广泛讨论公式性质的办法来检验一个公式,这足种特别有启发性的练习;见“第14节”和“你能检验这结果吗?”节第2点。
39.帕扑斯
帕扑斯是生活在公元300年左右的一位希腊数学家。在他的文集第七册中,
他提到了他称之为“analyomenos'’的一个学科分支。我们可把这一名词译成
“分析宝库”或者“解题艺术”,甚或“探索法”;最后一词在这里似更可取。
很容易找到帕扑斯报告的优秀英译本;下面是他原文的意译,
“所谓探索法,简言之,是一种学说的特殊部分,供那些学过普通几何原理、渴望获得求解数学问题能力的人之用,而且它也仅仅对此有用。它是下述三个人的工作:几何原理的作者即欧几里得,玻尔加的阿波罗纽斯和阿里斯陶斯长老。探索法教的是分析与综合的程序。”
“在分析中,我们从需要求解或求证的内容开始,我们假定它成立,由此得出结果,从这结果又得出结果,直到我们达到可怍为综合的起点的这一点为止。因为在分析中,我们假定把需要去做的当怍已经做好的(把要求寻找的当作已经找到的:把必须求证的当作已经成立的)。我们问根据什么前提可以导出所需要的结果;然后我们又问这个前提的前提可能是什么,如此等等,这样从前提过渡到前提,直到最终我们遇到某个事物是已知的,或被认为是成立的。此过程称为分析,或倒退求解,或回归论证。”
“但在综合中,此过程相反,我们从分析中最后达到的一点开始,从已知的或被认为成立的事物开始。我们从它导出在分析中位于它之前的一项,同时继续推导,沿各步骤逆向而行,直到最终成功地达到我们所需求解或求证的内容为止。上述过程称为综合,或构造性求解或前进论证。”
“分析有两类:一类足‘求证题’的分析,其目的是建立为真正的定理;
另一类是‘求解题’的分析,其目的是求解未知数。”
“如果我们有个‘求证题’,我们需要去证明或推翻一个清晰陈述的定理
A。我们至今还不知道A成立还是不成立;但我们从A导出另一定理B,从B导出另一C,等等,直到我们遇到最后一个定理L,关于它我们有确切的了解为止。假定我们所有的推导可逆,则若,L为真,A也将为真。从L开始我们证明在分析过程中位于L之前的定理K,然后,用同样方式前进,我们追溯各个步骤;从C证明
B,从B证明A,这样我们就达到了我们的目的,但如果L不成立,则我们也已证明A不成立。”
“如果我们有一个‘求解题’,我们需要找出某个未知数x,它满足清晰表达的条件。我们至今尚不知是否有什么东西可能满足此条件;但我们假定有一个x满足所提条件,我们从它导出另一个未知数y,y必须满足有关的条件;然后我们再把 y与另一个未知数相联系,如此等等,直到我们遇到最后一个未知数z;我们可以用某个已知方法求得它为止。如果这里确实存在一个z,满足加于它的条件,则这里也存在一个z满足原来的条件(假定我们所有的推导均可逆)。我们首先求出 z;接着,知道z以后,我们求出在分析中处于z之前的未知数:按同一方式,我们按各个步骤反向前进,最后,知道y,我们求得x,于是达到我们的目的。但是,如果没有什么东西可满足加z的条件,则关于x的问题无解。”
我们不应该忘记:以上并非直译而仅仅是信手译来,是一种意译。在原文和意译之间存在着干差万别,由于帕扑斯的文章在许多方面都很重要,所以值得评注如下,
(1)我们的意译比原文采用了更明确的名词,引入符号 A,B,…,L,X,Y,…,
E。而这些原文都没有。
(2)意译有“数学问题”字样,而原文意指“几何问题”。这是强调了帕扑斯所描述的程序决不限于几何问题;它们,实际上,甚至不限于数学问题。
我们需要举例说明这点,因为在这些情况下,随题目性质而来的普遍性与独立性很重要(见第3节)。
(3)代数说明。求满足方程
8(4x+4-x)一54(2x+2-x)+101=0
的x。这是个“求解题”,对初学者来说并不太容易。他必须熟悉分析的概念;
当然不是指分析这个词而是指通过反复简化以达到目的的概念。还有,他必须熟悉最简单的几类方程。即使有某些知识,还需要有好念头、一点好运气和一点创造能力,这样才能看出:由于4x=(2x)2,4-x=(2x)-2,所以引入
y=2x
可能是有益的。现在,这种代换确实有益,代入后,得到关于y的方程
8( 22 1yy + )一54( yy 1+ )+101=0
这看起来比原来简单。但我们的工作尚未结束。还需要有点创造,即引入另一个代换,
z= yy 1+
它将条件变换成
8z2—54z+85=0
如果解题者熟悉二次方程的求解,则分析到此结束。
什么是综合?这就是一步一步地做完这些由分析所预见到的可能的计算。
解题者完成他的问题并不需要什么新念头,计算各个未知数时只需要耐心与注意。计算的顺序恰与创造时的顺序相反:首先求z(z=5/2,17/4),接着求y(y=2,
1/2,4,1/4),最后是原来所要求的x(x=1,-1,2,-2),综合是沿着分析的步骤逆向而行。在本例,很容易看出为什么要这样做。
(4)非数学的说明。一个原始人希望渡过一条小河;但他不能用通常的办法渡河,因为昨晚已经涨水了。于是,渡河成为一个问题的对象;“渡河”即这个原始问题中的x。这个人可能回想起他曾沿着一棵倒下的树渡过其它几条河。于是他到处寻找一棵合适的倒下的树,这就成为他的新的未知数y。他找不到合适的树,但有大量的树立在河边;他希望其中有一棵能倒下来。他能使一棵树倒下来横跨这条小河吗?这是个了不起的念头,并且这里有一个新未知数:
用什么办法能弄倒这挺使之磺跨小河。
如果我们接受帕扑斯的术语,这一串念头应称之为“分析”。如果这原始人成功地完成了他的分析,他可能就成为桥与斧头的发明人了。什么是综合?
就是把念头化为行动。综合的最后一个行动是沿着一棵树走过小河。
在分析与综合中充满了相同的对象;它们锻炼了人在分析中的脑力和在综合中的体力;分析存在于思想之中,而综合则存在于行动之中。还育一个区别:
次序一正一反。走过河去是第一个愿望,分析由此开始,但它又是最后一个行动,综合到此结束。
(5)意译比原文稍为更清楚地提示出分析与综合之间的天然联系。经过前面一些例子,这种联系更明显了。分析自然先仃,综合后继;分析是创造,综合是执行;分析是制定一个计划,综合是执行这个计划。
(6)意译保留了甚至强调了原文中的某些占怪的句子:“把需要去做的当作已做好的:把要求寻找的当作已经找剑的,把必颂求证的当作已经成立的。”
这是矛盾的,假定我们所必须斛决的问题已经解决了,这不是自欺欺人吗?这是含糊不清的,它的意思指什么?如果我们紧密地联系其上下文,并且老老实实地尝试去体会我们自己在解题方面的经验,则对它的意思就很少会疑惑不解了。
我们首先考虑一个“求解题”。让我们称未知数为x,已知数为a,b,c。
“把问题当作已经解决的”这意思是说假定存在一个满足条件的x——即具有条件所规定的关于数据a,b,c的那些关系。我们作出这个假定,不过是为了开始分析,它只是权宜之计,但又无伤大雅。这是因为,如果没有这样的对象,
则不论分析把我们带向何方,它势必把我们引导到最终无解的问题,于是它将表明我们的原问题无解。所以这种假定是有用的。为了检验所给条件,我们必须看出,或在心里描述,或几何地想象出条件在x与a,b,c间所规定的各种关系;如果不把x看成(或描述成,或想象成)是存在的,我们又如何能做到这点呢?
最后,上述假定又是顺乎自然的。这原始人(他的想法和所作所为我们在评注4
中已经讨论过),远在他真正渡河以前,就先想象他自己走在一棵倒下的树上渡过了河;即他先把他的问题看成是“已解决的”。
“求证题”的对象是证明某个定理A。正式作出“假设A成立”的建议导致从定理A导出一系列结果,虽然A尚未得到证明。从一个尚未得到证明的定理推出一系列结果,可能会使具有某种心理性格或某种哲理的人们退避三舍;但这样的人无法开始分析工作。
请与“图形,(2)”中的内容相比较。
(7)在上述意译中两次出现下述重要句子:“假定我们所有的推导可逆。”
这是对原文的篡改;原文并未包台这类内容,而在现时代,已经察觉到缺少这样一个假定并提出了批评,参见“辅助问题,(6)”中可逆化归的概念。
(8)在意译中听阐述的“求证题的分析”在字句上与原文十分不同,但意思没变,无论如何,我们并没有改变它原意的企图。“求解题的分析”在意译中则比原文更具体。原文的目标看来是描述多多少少更一般化的程序,构造一个等价辅助问题链,这在“辅助问题”一节第(7)点中已有叙述。
(9)许多初等几何教科书在分析、综合及“假定问题已解”这方面,只作了少量的说明。人们几乎不怀疑:这种近乎根深蒂固的传统,可追溯到帕扑斯的时代,虽然目前几乎没有一本流行教科书的作者表明他对帕扑斯有任何直接的了解。这个题目很重要,应当在初等教科书中提到,但却很容易误解。单凭它局限于几何教科书这一点来说,就说明普遍地缺乏了解(参见上面第(2)点评注)。如果上面各个评注能促进对这件事的了解,则它们所占篇幅也就无可非议了。
对于其他的例子,不同的观点以及进一步的评论参见“倒着干”一节。
还可与“归谬法与间接证明,(2)”的内容作比较。
40.拘泥与精通
这是对待规则的两种相反的态度。
(1)不论场合是否合适,不折不扣地、刻板地、不加思索地应用某一规则,
这是拘泥迂腐。某些拘泥的迂夫子是可怜的傻瓜;他们自始至终没有理解他们如此不分青红皂白,亦步亦趋地照办的规则。有些迂夫子则很成功;他们理解规则,至少在开始时(在成为迂夫子之前)理解,并且他们作出了适合于许多场合而只偶然失利的良好选择。
应用规则要得心应手,运用自如;并且注意到规则所适用的场合,有选择、
有批判地加以应用;不为规则的词句而迷失本意,或本末倒置或坐失良机,这就是精通。
(2)本书中关于问题与建议的表,对解题者和教师可能都有帮助。但是,
首先必须了解这张表,学习怎样正确地应用这些问题与建议,并且通过试验与错误、成功与失败、通过应用求学习这张表。其次,决不可拘泥迂腐地应用它们。你不应当按照某种刻板的习惯,不加选择地提出问题和作出建议。要准备各式各样的问题和建议并作出判断。你正在做一件艰巨而又令人兴奋的工作;
你下一步将要干什么应当根据对你面前的问题作仔细的、无偏见的考虑后再来决定。你希望帮助一个学生;你对你的学生说些什么,应当从同情并了解学生的困难这点出发。
但如果你喜欢当一名迂夫子而必须依靠某种规则,请学习这一条,
永远要首先开动自己的脑筋
41.实际问题
实际问题在许多方面与纯数学问题不同,但求解的中心思想与程序基本上相同。实际工程问题通常包含数学问题。关于这两类问题的差别、类似之处与联系,我们在下面将稍加讨沦。
(1)一个令人印象深刻的实际问题是在河上筑坝。了解这个问题不需要什么特殊的知识。几乎就在史前时期,远在有科学理论的时期之前,人们就在尼罗河谷以及世界的其他地方建立过某种坝,这些地方的谷物需要灌溉。
让我们想象建筑一个重要的现代堤坝问题。
未知数是什么?这类问题包含许多未知数:坝的确切位置,它的几何形状和大小,建筑用料等等。
条件是什么?我们无法用一个短句子回答这个问题,因为有许多条件。在如此庞大的一个项目中,需要满足许多重要的经济要求,并且要尽量不妨碍别的要求。这坝应提供电力,能灌溉,给某城市供水并有助于控制洪水。另一方面,它尽量不妨碍搬运、或有重要经济价值的鱼类、或美丽的风景等等。当然还应该尽量节约,工程进度应尽量快。
已知数据是什么?所需要的数据数量极大。我们需要关于河流及其支流附近地区的地形地貌数据;对基础的坚固性,可能有的渗漏等重要的地质数据,
以及可用的建筑材料;关于年降雨量和洪水高度方面的气象数据;关于被淹土地的价值、材料和劳力费用等方面的经济数据,如此等等。
此例说明在实际问题中,未知数、已知数据与条件要比数学问题更复杂并且定义得没那么清楚。
(2)为了解决一个问题,我们需要有一定数量的、以前获得的知识。现代工程师具有大量的高度专门化的知识可加以运用,例如材料强度的科学理论、
他本人的经验以及专门技术文献中所贮存的大量工程经验。这里我们不能利用这些专门知识,但我们可以设想一个古代埃及堤坝的建造者在想些什么。
他当然看见过各式各样的坝(或许较小):土造的或砖石砌成的防水堤。他看见过挟带各种泥石的洪水向堤岸压来。他可能协助整治过洪水所产生的侵蚀与破损。他可能看见过在洪水冲击下决口的堤坝。他肯定听说过关于经受几世纪考验的堤坝或由于意外决口酿成大祸的故事。在他的脑海中可能已经描绘了一幅河水压迫堤面和内部应力应变的情景。
然而埃及堤坝建造者并没有关于流体压力或固体内部应力应变方面的精确定量的科学概念。这些概念是现代工程师的知识库中的主要部分。后者还使用了不少尚未完全臻于精确科学水平的知识;他所知道的关于流水侵蚀、淤泥流迂、某些材料的弹塑性以及其他尚未弄清的特性方面的知识,就是这样一种带有经验性的知识。
上述例子表明,在实际问题中所需要的知识和所使用的概念比在数学问题中更复杂,定义得也不那么清楚。
(3)在实际问题中,未知数、已知数据、条件、概念、所需要的预备知识,
每一项都比纯粹数学问题更复杂,更不清楚。这是重要的差别,或许是主要的差别,并且它肯定还包含更多的差别;但对于这两类问题来说,求解的基本中心思想与程序看来却是相同的。
有一种广为流传的意见,即实际问题比数学问题需要更多的经验。这可能如此。但很可能,这种差别只存在于所需要知识的性质,而不是我们对问题的态度。在解决这样那样的问题时,我们必须依赖我们在处理类似问题方面的经验,我们经常问这个问题:你是否见过相同的问题,只是形式上稍有不同?你知道一个与此有关的问题吗?
在解决一个数学问题时,我们经常从非常清楚的概念出发,这些概念在我们心中是井然有序的。在解决一个实际问题时,我们被迫从颇为含糊不清的概念出发;于是把这些概念搞清楚可能成为问题的一个重要部分。这样,今天的医学比巴斯特以前的时代更有能力去检验传染病,那时“传染”这一概念本身还是颇为模糊的。你是否已将问题包含的所有必要的概念都考虑在内?对于各类问题来说,这都是个很好的问题。但它的应用,随着所涉及的概念性质而变化很大。
在一个陈述的数学问题中,所有已知数据与有关条件的所有条款都是必不可少的,从而必须加以考虑。在实际问题中,我们有大量的已知数据与条件;
我们力求尽可能多地加以考虑,但我们却不得不忽略一部分。就拿大坝设计者为例来说吧,他考虑了公众的利益和重大的经济利益,但他不得不舍去次要的要求与损害。严格说来,他问题中的已知数据是数不胜数的。例如,他可能愿意多了解一些地基的地质性质,但他最后毕竟必须停止收集地质数据,虽然不可避免地仍然有一些不清楚的地方。
你是否利用了所有的已知数据?你是否利用了整个条件?当我们处理纯数学问题时,我们不能放过这些问题。但在实际问题中,我们应当改变这些问题的形式:你是否利用了可能对求解有显著作用的所有数据?你是否利用了可能对求解显著影响的全部条件?我们估量一下现成可用的有关资料,如果必要的话,
我们再去收集一些,但最终我们必定要停止收集,我们必会在某处划地为界不再越雷池一步,我们不能不忽略某些东西;正是:“若要航行无险,除非永不下海”。常常存在着大量过剩数据,这些过剩数据对解答的最终形式其实并无明显影响。
(4)古埃及坝的设计者不得不依靠其经验的朴素的解释,他们除此之外,
别无其他可以依据的东西。现代工程师不能单单依靠普通常识,特别当他的方案是一种崭新而大胆的设计时更是如此;他必须计算所设计的堤坝的抗力,定量地预测其内部的应力和应变。为此,他必须应用弹性理论(这种理论颇适用于混凝土结构问题)。为了应用此理论,他需用大量数学;而实际工程问题也引出数学问题。
这种数学问题技术性过强,这里不予讨论;对它我们只能做一般性的评论。
在建立和解决从实际问题引出的数学问题时,我们通常只能满足于近似。我们不得不舍去实际问题中的某些次要的数据和条件。因此在计算中允许有小的误差是合理的,特别当我们失去精确性但获得简单性的时候更是如此。
(5)关于引起普遍兴趣的近似问题还可以谈很多。然而,我们不能假定读者有任何专门化的数学知识,所以我们这里仅限于举出一个直观而富有启发性的例子。
画地图是个重要的实际问题。设计一张地图时,我们常常假定地球是圆的。
但这仅是个近似的假设而并非精确的真理。地球表面完全不是一个数学上所定义的球面,而且我们消楚地知道,地球在两极变成扁平。然而,假定地球是个球,找们可以更容易画出它的地图。我们在简单性方面所获甚多而在精确性方面所失并不太多。事实上,我们设想有个球和地球一模一样,它在赤道处的直径为25英尺,由于地球两极扁平,所以它两极间的距离小于25英尺,但仅仅短一英寸。因此球体是一个很好的实际近似。
42、求解题,求证题
我们将这两类问题进行比较。
(1)“求解题”的目的是找出某个对象,即问题的未知数。
未知数在拉丁文中称为“quaesitum”,或要找的东西,或需求的东西。
“求解题”可以是理论的或实际的,抽象的或具体的,严肃的问题或仅仅是谜语。我们可以寻求各种各样的未知数。我们可以尝试去发现、得到、获取、产生、或者构造所有各种各样想象得到的对象。在侦探小说里,未知数是个凶手;
在下棋问题中,未知数是棋手的下一步棋。在某些初等代数问题中,未知数是一个数。在一个几何作图问题中,未知数是一个图形。
(2)“求证题”的目的是最终说明某个清晰陈述的推断是成立还是不成立。
我们必须回答这问题:这推断是成立还是不成立?并且我们必须通过证明谈推断成立或不成立来作出最终的回答。
一个证人肯定,被告某晚是在家的,法官必须查明此断言究竟是真实的还是不真实的。此外,他还必须尽可能地给出充分的理由。这样,法官就有了一个“求证题”。另一个“求证题”是“证明毕达哥拉斯定理”。我们不说“证明或推翻毕达哥拉斯定理”。从某些方面来说,在问题的陈述中包括推翻的可能性在内可能更好。但因为我们已知推翻此定理的可能性很小,所以可以略去。
(3)“求解题”的主要部分是:未知数、已知数据与条件。
如果我们必须作一个具有边a,b,c的三角形,则未知数是个三角形,已知数为其三边a,b,c,而三角形要满足条件,即其三边具有给定的长度a,b,
c。如果我们必须作一个三角形,它的三个高是a,b,c,则未知数是与上题同一类型的对象,数据也相同,但联系未知数与已知数的条件则不同。
(4)如果一个“求证题”属于通常一类数学问题,则其主要部分是待证或待推翻定理的前提与结论。
“如果四边形的四边相等,则二对角线互相垂直”。在这句话中,从“则”
开始的下半句是结论,而以“如果”开始的上半句是前提。
[并非所有的数学定理都能自然地划分成前提与结论两部分。例如,下列定理就几乎不可能这样划分:“质数有无穷多个”。]
(5)如果你希望解一个“求解题”,你必须知道并且十分准确地知道其主要部分:未知数,已知数与条件。我们的表中有不少关于这些主要部分的问题与建议。
未知数是什么?已如数是什么?条件是什么?
把条件的各个部分分开。
找出已知数与未知数间的联系。
看着未知数!试想出一个具有相同或相似未知数的熟悉的问题。
仅仅保持条件的一部分,而舍去其余部分;这样对于未知数能确定到什么程度?它会怎样变化?你能不能从已知数据导出某些有用的东西?你能不能想出适于确定此未知数的其他已知数?如果需要的话你能不能改变未知数或已知数,
或者二者都改变,使得新未如数和新已知数彼此更接近?
你是否利用了所有的已知数?你是否利用了整个条件?
(6)如果你想解决一个“求证题”,你必须知道而且非常准确地知道其主要部分,即前提与结论。关于这些主要部分,有些有用的问题与建议与我们表中那些特别适合于“求解题”的问题与建议一一相对应。
前提是什么?结论是什么?
把前提的各个部分分开。
找出前提与结论间的联系。
看着结论!试想出一个具有相同或相似结论的熟悉的定理。
仅仅保持前提的一部分,舍去其余部分;这结论是否仍有效?你能不能从前提导出某些有用的东西?你能不能想出容易导出此结论的其他前提?如果需要的话,你能不能改变前提,或改变结论,或者二者都改变,使得新前提和新结论彼此更接近?
你是否利用了整个前提?
(7)在初等数学中,“求解题”更重要,在高等数学中,“求证题”更重要。本书比较着重“求解题”。作者希望在有关本论题的一部更详尽的著作中重新对这两类问题作平衡处理。
43.进展与成就
你有任何进展吗?主要成就是什么?在解题过程中,你可能问自己或者问一个你督促其功课的学生。这样,我们惯于或多或少满怀信心地判断具体情况下的进展与成就。但由这种具体情况进一步得到一个普遍性叙述却很不容易。可是,如果我们想促使我们对探索法的研究进一步完整的话,我们必须进行这一步,而且我们必须力图弄清楚,在普遍意义上,解题中的进展与成就包含哪些内容。
(1)为了解题,我们必须具备本论题方面的知识并且必须对我们现有的,但原来属于休眠状态的知识进行挑选并收集相关内容。我们对该问题的理解在问题结束时总比开始时要丰富得多;增加了些什么呢?从我们的记忆中,我们成功地汲取了什么呢?为了得到解答,我们必须回忆各式各样的基本事实。如果是个数学问题,则我们为了得到解答,必须回忆以前解答过的问题,已知的定理和定义。从我们的记忆中汲取这些有关内容可称之为“动员”。
(2)然而,为了解题,仅收集上述孤立的事实是不够的,我们还必须把上述事实组合起来,并使之比较适用于我们手头的问题。这样,在解一个数学问题时,我们必须将所收集到的材料联成适用的一个整体,从而构造一种引理。
这种加以组合和使之适用的活动可取名为“组织”。
(3)实际上,动员和组织绝不能真正相互分开。当我们全神贯注地解题时,
我们仅仅回想那些与我们目的多多少少有联系的事情,并且除了我们已经收集和动员的资料以外,别无其他内容可加以联系和组织。
动员与组织只不过是同一复杂过程的两个侧面,这种过程还有许多其他侧面。
(4)工作进展的另一侧面是:概念变换的方式。收集了资料并进行加工以后,我们关于问题的概念在结束时比在开始时丰富得多了。由于我们想从初始的概念前进到一个更满足要求的、更适用的概念,我们可以尝试从不同的观点并从各个不同的侧面观察此问题。如果不“变化问题”,我们几乎不能有什么进展。
(5)当我们朝最终目标进展时,我们越来越清楚地看到它,当我们看它看得越清楚时,则我们判断自己离它越来越近。我们还越来越清楚地预见到,对于求解,我们应该做些什么和怎样做。解决数学问题时,我们可能预见到,某个已知定理可能用得着,某个已解决的问题可能有帮助,返回到某个专业术语的定义去可能是必要的。但我们并不能肯定无疑地加以预见,而只不过以某种程度的似乎可信来预见它们而已。然而如果没有这种仅仅是“似乎可信”的和假设的考虑,我们将永远不能找到最终的、肯定的结果。我们需要探索式论证。
(6)趋向解答的进展是什么?进一步动员与组织我们的知识,改进我们对问题的理解,增加我们对构造最终论证的步骤的预见性,这些都是进展。我们可能以难以察觉的小碎步缓缓前进,但时而又飞腾跳跃,突飞猛进。向求解的突然进展称为“好念头”、“妙主意”、“巧想法”、“灵机一动”[在德语中还有一个更技术性的词汇叫“Einfall(点子)”]。什么是好念头?是我们观点上的一次重人突变,我们看问题方式的一个骤然变动,在解题步骤方面的一个刚刚露头的有信心的预感。
(7)上述考虑为我们表中的问题和建议提供了合适的背景。
许多问题和建议的直接目的是动员我们以前得到的知识:你以前见过它吗?
你是否见过相同的问题而形式稍有不同?你是否知道一个与此有关的问题?你是否知道一个可能用得上的定理?看着未知数。试想出一个具有相同未知数或类似未知数的熟悉的问题。
在某些典型情况下,我们认为我们已经收集了合适的材料,于是我们争取把我们已经动员的内容组织得更好些:这里有一个与你有关且早已解决的问题,
你能不能利用它?你能不能和利用它的结果?你能不能利用它的方法?为了能利用它,你是否应该引入某些辅助元素?
还有其它的典型情况,在这些典型情况下,我们认为我们尚未收集到足够的材料。我们怀疑丢掉了什么:你是否利用了所有的已知数据?你是否利用了整个条件?你是否考虑了包含在问题中的所有必要概念?
有些问题直接以变化问题为目的:你能不能重新叙述这个问题?你能不能重新叙述得更不同些?许多问题的目的是用特殊手段来变化问题的,如回到“定义”去,利用“类比”,“普遍化”,“特殊化”,“分解与重新组合”。
还有其它的问题建议我们去尝试预测我们所力图取得的解答具有什么性质:满足条件是否可能?为了确定未知数,条件是否充分?或者它是否不充分?
或者它是多余的?或者它是矛盾的?
我们表中的问题和建议并不直接提到好念头;但实际上,所有的问题和建议都与它有关。了解问题是为好念头的出现作准备;制订计划是试图引发它;
在引发以后,我们实现它;回顾此过程和求解的结果,我们是试图更好地利用它。
44.谜语
根据第3节,我们表中的问题和建议与论题无关,可以适用于所有各类问题。用形形色色的谜语来考验我们这句话是十分有趣的。
例如,下列各字
DRY OXTAIL IN REAR
问题是组字游戏,即,将这些字中的字母重新组合成另一个字。我们很感兴趣地注意到,在解这个谜语时,表中有好几个问题是贴切的,甚至是有促进作用的。
未知数是什么?一个字。
已知数据是什么?四个字:DRY OXTAIL IN REAR。
条件是什么?所求的字有15个字母,即上述四个字中的字母。它可能是个不太常见的英文字。
画张图。标出十五个空格很有用。
………………………………
你能重新叙述这问题吗?我们必须找出一个字,它的字母是
A AEIIOY DLNRRRTX
这说法肯定与前面的问题等价(参见“辅助问题”,第(6)点)。这种重新叙述可能很有好处。把元音和辅音分开(这点很重要,不是按字母顺序排列),
我们会看到问题的另一个侧面。我们看出所找的字有七个音节,除非它有某些双元音。
如果你不能解决所提的问题,尝试先解决一个与此有关的问题。用给定的字母中的一部分来组字就是一个有关的问题。我们肯定能组成这种短的字。然后我们尝试去找越来越长的字。我们所用的字母越多,我们距所找的字可能就越近。
你能不能解决问题的一部分?所找的字既然这样长,它必可清楚地分成几部分。它大概是一个复合字,或者它可通过添加常见的字尾而由某个其他的字派生出来。它可能是哪个常见的字尾呢?
…………………………………………ATION
…………………………………………ELY
仅仅保持条件的一部分,舍去其余部分。我们可能尝试去想一个长的字,
它可能有多至七个字母但却有相对较少的辅音,它有一个X和一个Y。
我们表中的问题与建议并非灵丹妙药。它们不能使我们不费吹灰之力而解答所有可能的谜语。如果读者想找到这个字,他必须继续冥思苦想。表中的问题与建议所起的作用是“开动脑筋”。当我们因不成功而沮丧时,我们倾向于放弃它,而这些问题与建议却可以向我们提示一种新的尝试、新侧面、问题的一个新变化、一种新的促进因素;它们使我们继续思索。
另外一个例子见“分解与重新组合,第(8)点”。
45.归谬法与间接证明
归谬法和间接证明是两种不同的、然而相互关联的过程。
归谬法是利用导出一个明显的谬误来证明假设不成立。归谬法是个数学过程,但它和讽刺家所爱好的做法——反话,却有几分相似。反话,很明显地采纳某个见解,但强调它并且过分强调它,直到产生一个显然的谬误。
间接证明是通过证明相反的假设不成立来证明某个推断成立。因此间接证明与政客用败坏对手声誉的办法来建立自己候选人威信的诡计有几分相似。
归谬法与间接证明都是发明创造的有效工具,这些发现创造是在一个专心致志的头脑中自然涌现出来的。然而有少数哲学家和不少初学者却不喜欢归谬法和间接证明,这是可以理解的;爱挖苦的讽刺家和诡诈的政客并不能投每人所好。我们将先用例子说明这两种过程是有效的,然后再讨论对它们的反对意见。
(1)归谬法。在0,1…,9这10个数字中,每个都要用一次且只允许用一次,
写出几个数使得其和数恰为l00。
在尝试解决这个谜语时,我们会学到一些东西,但谜语的叙述还需要某些说明。
未知数是什么?一组数字;当然,这里数的意思是指普通整数。
给定的巳知数是什么!数100。
条件是什么?条件有两部分。首先,写这组数时,我们必须使用0,1,…9
这十个数字中的每一个,而且每个数字只能用一次。其次,这组数之和必须为
100。
仅仅保持条件的一部分,舍去其余部分。单独第一部分容易满足。例如这组数19,28,37,46,50,每个数字仅出现一次。当然,条件的第二部分并未满足;其和为180,而不是100。但是我们可以做得更好些。“试试,再试试。”
好,
19+28+30+7+6+5+4=99
条件的第一部分满足,第二部分差一点满足;但我们得到的是99,还不是100。
当然,如果我们舍去条件的第一部分,则很容易满足第二部分,
19+28+3l+7+6+5+4=100
但这不满足条件的第一部分:数字1出现了两次,而0根本不出现;其他数字则全都对头。“试试,再试试”。
然而,在几次试验都不成功之后,我们可能产生了怀疑:按照所要求的方式是否不可能得到100?终于涌现了下述问题:证明所提条件的两个部分不可能同时满足。
即使是十分优秀的学生也可能感到这个问题超出了他们力所能及的范围。
但如果我们有正确的态度,并不难回答。我们必须检验一下“能同时满足此条件的两部分”这一假想情况。
我们猜测这种情况实际上不会发生,而我们的猜疑,基于我们屡试不成的经验,是有某些根据的。无论如何,让我们虚心地正视这种所谓条件两部分均能得到满足的假想的、想象的、宣称而未得到证实的情况。我们想象有一组数,
其和为100。它们必定是一些有一位或两位数字的数。我们这里有十个数字0,1,
2,……,9,由于题中规定每个数字正好只出现一次,所以它们彼此不相同。
于是,这10个数字之和为
O+l+2+3+4+5+6+7+8+9=45
这些数字,有的用来表示个位数,有的用来表示十位数。稍具头脑就会想起这个念头,即,表示十位数的数字之和可能有某种重要性。事实上,t表示此数字和,则其余数字表示个位数,其和为45-t。因此,这组数之和必为
10t+(45一t)=100
这是个确定t的方程。它是个一次方程,可求得
t=55/9
现在,肯定有某种错误。所得t值竟不是整数,而t当然应该是整数。我们假定所给条件的两个部分能同时满足,从这一假定出发,我们已经导出了明显的谬误。如何解释这一点呢?我们原先的假定一定是错误的;即条件的两部分不能同时得到满足。于是我们达到了我们的目的:我们已经成功地证明了所提条件的两个部分是不相容的。
上述论证是典型的“归谬法”。
(2)注解。让我们回顾上述论证并了解其普遍趋势。
我们希望证明某个条件不可能得到满足,即,该条件的所有各部分同时得到满足的情况永不会发生。但是,如果我们迄今并未作出任何证明,则我们就不得不正视这种情况有发生的可能性。只有干脆面对这一假设情况,并仔细地加以检验,我们才有希望从中查出某个确定无疑的错误之处。如果我们希望确切地证明这情况不可能发生,那么我们必须从某个确定无疑的错误之处着手。
因此,我们可以看出,在我们例子中已成功的做法是普遍合理的:就是即使条件的所有各部分都得到满足这一假想情况看起来极不可能发生,我们也必须对它加以检验。
更有经验的读者可能从这里看出另外一点:上述做法的主要一步是对t建立一个方程,而我们不必猜测条件中是否有什么错误便可以建立同一个方程。
如果我们希望建立方程,我们必须用数学语言来表达条件的所有各部分都得到满足这一内容,虽然我们迄今尚不知道是否真正可能同时满足所有各部分。
我们的做法是“无偏见的”。我们可以希望找出满足条件的未知数,或者希望证明条件无法满足。就考察问题这方面来说.这点无足轻重,如果进行顺利,就在两种情况下都按同一方式开始,检验满足条件的假想情况,而仅在以后的过程中再表永哪一种希望是合乎道理的。
请与“图形”(2)的内容相比较;还可与“帕扑斯”的内容相比较;如果一个分析的最终结果是推翻所提定理或者证明所提“求解题”无解,则这个分析实际上是一个“归谬法”。
(3)间接证明。质数是数2,3,5,7,11,13,17,19,23,29,31,37…。
它们不能分解成较小的因子,虽然它们大于1(这最后一句已把1除外,1虽显然不能分解成较小的因子,但它具有不同的性质而不应当看成质数)。把整数(大于1的)分解因数时其中每个“最终元素”就是质数。例如
630=2·3·3·5·7
分解为五个质数的乘积。
质数序列是无穷的?还是到某数以后就没有质数了呢?我们自然会猜测质数序列是无穷的。如果它在某处终止,则所有整数能被分解成有限个这种最终元素,于是就可以说世界似乎是“太贫乏”了。这就产生了要证明质数有无穷多的问题。
这个问题和通常的初等数学问题很不相同,并且乍一看来不好下手。但是,
正如我们说过的,极不可能存在一个最后的质数P。为什么这是不可能的呢?
让我们干脆正视这一不大可能的情况,在这情况下假想地、想象地、声称如此而未经证实地存在一个最后的质数P。于是我们可写出整个质数序列2,3,
5,7,11,…,P。为什么这是不可能的呢?它错在哪儿?我们能否指出任何肯定的错误?确实,我们能够。我们可以构造一个数
Q=(2·3·5·7·11…P)+1
这个数Q大于P,由此可断言Q不可能是一个质数。因此Q一定能被质数除尽。但现在摆在我们面前的所有质数已假定是2,3,5,…P,而Q被其中任何一个数除时,都有余数1;所以 Q不能被任何一个上述质数除尽。根据假定,上述质数是全部质数。于是,现在必有某个肯定性的错误存在;Q须是一个质数或者可由某个质数所除尽,二者必居其一。从存在一个最后的质数这一假定出发,我们已经被引导到一个明显的荒谬。我们怎么解释这点呢?我们原来的假定一定是错误的,即并不存在一个最后的质数P。于是我们成功地证明了质数的序列永不终止。
我们的证明是一个典型的间接证明(它也是一个著名的证明,是由欧几里得给出的:参见欧几里得《几何原木》第九册,命题20)。
我们已经建立了我们的定理(即质数序列永不终止)。这是通过反驳其对立面(即质数序列终止于某数)而得到的,即从对立面引出一个明显的谬误。这样,
我们已经把间接证明和“归谬法”合并在一起了,这种合并也是非常典型的。
(4)反对意见。我们正在研究的做法遭受过颇多的非议。人们提出了许多反对意见,它们可能是同一个基本反对意见的小同形式。下面我们讨论反对意见的一种适合本书水平的“实际形式”。
找出一个并非显而易见的证明是一个值得重视的智力成就,但学习这样一个证明,甚或去彻底了解它也要耗费一定的脑汁。不言而喻,我们希望从我们所花费的精力上得到某些好处,而且希望我们保存在记忆中的东西是真实而且正确的,并非虚假或荒诞不经的。
可是,从“妇谬法”得到某些止确的东西看来并非易事。这个做法是从一个虚假的假设出发,导出某些具有同样程度、或更明显的错误结果,直至达到最后一个显而易见的错误结果为止。如果我们不希望在记忆中保存谬误,我们应当尽快地忘记每一件事,但这是行不通的,因为在我们研究证明的过程中,
所有各点都必须鲜明而正确地记在心头。
现在可以非常简略地陈述对间接证明的反对意见了。在倾听这样一种证明时,我们被迫以全部时间注意一个我们应该忘却并且虚假的假没,而并不是注意一个我们应该保存在记忆中的正确的定理。
如果我们希望正确地评判这些反对意见的是非,我们应当区分“归谬法”
作为一个研究手段和作为一种阐述方式这两种用途;对于间接证明,我们也要作同样的处理。
必须承认,“归谬法”作为阐述方式并非纯粹乐事。这样一种归纳,特别当它很冗长时,对读者或听众来说,确实可能成为一桩苦差事。我们连接不断加以审查的所有推导都是正确的,然而我们面对的所有情况却又都是不可能的。
如果措辞象理所当然那样不断强调每一点都根据一个初始的假设,那么甚至连所用措辞都会令人生厌;“假想地”,“假设地”,“未经证实地”,这些词一定反复出现或者必须连续使用某种其他手段。我们希望把所述情况看作不可能的事而加以摒弃与忘却,但我们又必须把它作为下一步的基础而加以保留和审查,于是这种内在的不和谐,久而久之,可能令人不堪忍受。
但是,如果否定“归谬法”作为发明创造的手段则是愚蠢的。如前面例子所表明的那样,当所有其他手段看来已用尽时,这种手段会自然而然地涌现出来并导致对前提作出判断。
需要有些经验才能看出在两种争论之间并无根本的对立。经验表明,把一种间接证明变为直接证明或者把一个用冗长的“归谬法”所得到的证明重新安排成令人更愉快的形式(其中甚至可以完全去掉“归谬法”,或者经过适当的加工后,将“归谬法”压缩成寥寥几句醒目的话),通常几乎并无困难。
总之,如果我们希望充分发挥我们的能力,我们应当既熟悉“归谬法”,
又熟悉间接证明。但当我们利用其中一个方法成功地导出结果时,我们应当回顾这个解,并且问:你能不能用不同的方式导出这个结果?
我们用例子说明如下。
(5)重新安排“归谬法”。我们回顾第(1)点中的论证。“归谬法”是从一个最终成为不可能的情况开始的。但是,让我们把与初始虚假的假设无关并且包含正面信息的一部分论证剖析出来。重新考虑我们所已经做过的分析,我们看出下述一点无疑为真:若一组具有一位或两位数字的数使得O,1,2,…,9
十个数字恰好出现一次,则这组数之和有下列形式
10t+(45-t)=9(t+5)
这样,此和数可彼9所除尽。然而,所提的谜语却要求此和数为100。这是否可能呢?不,不可能,因为100不能被9除尽。
在我们这个新的提法中,原来导致发现论证的“归谬法”消逝了。
顺便捉一提,熟悉“弃9法”的读者现在能够一眼看出整个论证。
(6)改造一个间接证明。现在我们回顾前面第3点提出的论证。仃细地重新考虑我们所做过的分析,我们可能会找到与任何虚假的假设无关的元素,但若对原问题本身的含义重靳加以考虑,则会得到最好的线索。
我们说质数序列永不终止,其含义是什么?显然,就是以下这点:当我们已经确定任何一组有限的质数集合,如2,3,5,7,11,……,P,其中P是迄今所找到的最后一个质数,则总还有另一个质数存在。这样,为了证明质数有无穷多,我们必须做什么?我们必须指出找到一个新质数的方法,这个新质数不同于迄今所找到的所有质数。于是,我们的“求证题”就化成一个“求解题”:
给定质数2,3,5,……,P,求一个新质数 N与所有给定质数不同。
既然已经用这种新形式重新叙述了我们原来的问题,我们就已经跨出了关键的一步。现在就比较容易看出怎样把我们上述论证用于新目的。事实上,数
Q=(2·3·5·7·1l……P)+1
肯定可被一质数除尽。让我们取——这就是主意——Q的任何质数除数(例如最小的一个)为N(当然,如果Q恰为一个质数,则 N=Q)。显然,Q除以质数2,3,5,……
P中的任何一个都将剩下余数1,因此,这些数中的任何一个都不会是 N,即不会是Q的除数。但这就是我们全部所需要的:N是一个质数并且它和我们迄今所找到的质数2,3,5,7,11,……,P不同。
这个证明给出一个无限延长质数序列的确定性过程。其中没有什么是间接的,也不需要考虑不可能的情况。但它基本上与我们前面的间接证明相同,我们已经成功地把间接证明改造了。
46.多余
(见“条件”一:节)。
47.代公式问题
如果以前讲解过和示范过一般二次方程的求解,以致学生在解方程
x2-3x+2=O时除了把普遍解中的两个字母代之以数字-3和2以外无事可干,则解这个方程的问题就叫做“代公式问题”或“例行程序题”。即使二次方程并未曾一般地用“字母”求解过,但却在不久以前求解过五六个具有数字系数的类似的二次方程,那么该问题也应当叫做“代公式问题”。一般说来,如果求解一个问题可以把具体数据代入以前解过的普遍性问题,或者依样画葫芦地按照一个陈旧而明显的例子硬套而不必穷究其源,则这个问题就是一个“代公式问题”。布置一个代公式题,教师就是在学生面前公然对下列问题强行给出一个无需思索而又明确的回答:你知道一个与此有关的问题吗?这样,学生除了在恪守“照章办事”的箴言时稍稍细心与耐心一点之外,别无他求,而学生也就无从发挥他的判断能力或创造才能。
在数学教学中,代公式问题,甚至许多代公式问题是必要的.但苦使学生别无其他类型的题可做,则是不可原谅的。
只教授代公式数学演算这种刻板作业而不教授其他内容,这对以低于菜谱的标准来说倒是不坏,因为厨房调配菜、料毕竟还给炊事员的想象力与判断力留有余地,然而菜谱式的数学却不然。
48.发明创造的规律
发明创造的规律,第一条是动脑筋和运气好,第二条是锲而不舍直到出现一个好念头。
有些壮志是难酬的,我们略为提醒一下这点很有好处。指引人们去解答所有可能的数学问题的这种放之四海皆准的发明创造规律,可能比炼丹术士所痴心妄想梦寐以求的点金石更为人所响往。这类规律验之如神;但事实上并不存在。找到一种可适用于各种各样问题的万灵规律是一个古老的哲学梦想;但它永远也只能是一种梦想罢了。
合理的探索法不能以万灵规律为目标,但它可以努力研究在解题中典型有用的做法(智力活动,策略,步骤)。这种做法是每一个对他的问题很感兴趣的正常人所经验过的。这些人被提示以某些固定格式的问题与建议(聪明的人是自己向自己提,学生则由老师向他提)。收集这类问题与建议并使其具有充分的普遍性并且安排简洁得体,这虽不及点金石那样使人响往,但却切实可行。我们的表就是这样的内容。
49.风格的规律
第一条是要言之有物。第二条是,如果碰巧有两件事要谈,要一件一件地谈,不要两件同时谈。
50.教学的规律
第一条是:要懂得自己打算教的内容。第二条是:懂得的要比打算教的内容多一些。
先谈第一条。本书作者并不认为教师所奉行的所有规律规则都是毫无用处的;否则作者就不敢写一整本关于教师和学生行为的书了。但是绝不可忘记,
数学教师应当懂得一些数学,而希望教给学生正确思考问题的方法的教师应当自己首先掌握它。
51.把条件的各个部分分开
我们的首要责任是了解问题。在把问题作为一个整体了解之后,我们进入细节。我们考虑问题的主要部分,未知数,已知数据,条件,逐个考虑每一部分本身。当我们对这些部分了解得很清楚,但仍想不出特别有帮助的念头时,
我们深入到更进一步的细节。我们考虑各个已知数据,考虑单个已知数据本身。
在把条件作为整体了解以后,我们将其各部分分开,并考虑每个部分本身。
现在我们看一看在本节所讨论的建议有什么作用。它的作用是:当我们正试图弄清问题并且越来越细致地进入细节时,它会使我们想起所必须采取的一个步骤。它就是“分解与重新组合”中的一个步骤。
把条件的各个部分分开。你能写下它们吗?我们在“建立方程”时常常有机会提出这个问题。
52.建立方程
建立方程就象把一种语言翻译成另一种语言[见“符号”一节(1)]。牛顿在他的著作《通用数学》中所打的比方可以帮助我们弄清学生和教师所常常遇到的某些困难的性质。
(1)建立方程的意思是把用文字表达的条件改用数学符号来表示,它是从普通语言到数学公式语言的一种翻译。在建立方程中所可能遇到的困难是翻译的困难。
为了把一个英文句子译成法文,有两件事必须做到。第一,我们必须彻底理解该英文句子。第二,我们必须熟悉法语特有的表达形式。在建立方程时,
情况与此十分类似。第一,我们必须彻底了解条件。第二,我们必须熟悉数学表达形式。
如果该英文句子可一字对一字地翻译,那么它比较容易译成法文。但是有些英文成句并不能一字对一字地译成法文。当我们的句子中包含这种成语时,
翻译起来就比较困难了;这时,我们必须少注意孤立的字,多注意整个含义;
在翻译句子之前,可能需要把它重新排列一下。
建立方程时与此几乎雷同。在容易的情况下,文字叙述几乎自动分成接连的几个部分,每一部分都能直接用数学记号表示出来。在较困难的情况下,条件的某些部分不能直接译成数学记号。倘若如此,我们应少注意文字叙述,多注意含义。在我们写出表达式之前,可能需要把条件重新排列,当我们这样做的时候,我们要同时留意从数学记号宝库中选用什么数学记号。
在所有情况下(不论难或易),我们必须了解条件,把条件的各部分分开,
并且问:你能写出它们吗?在容易的情况下,我们可以立即把条件分成各部分,
并用数学记号写出来;在困难的情况下,条件的适当分法并不那么明显。
在研究过下面例子之后,你应当再读一遍上面的说明。
(2)求两个数,其和为78,其积为1296。
我们把书的一面用直线分成两半。一侧写上已分开成适当部分的文字叙述,另一侧写上代数符号,与文字叙述中的相应部分相对。即原问题在左,符号翻译在右。
问题的叙述(表)
在本例中,文字叙述几乎自动地分成几个相互衔接的部分,每个部分可直接用数学符号表示出来。
(3)求底面为正方形的正棱柱的宽与高,已知其体积为63英寸3,表面积为
102英寸2。
未知数是什么?底的一边,设为x。棱柱的高,设为y。
已知数是什么?体积63,面积102。
条件是什么?底为正方形的棱柱,边长为x,高为y,体积须为63,面积须为
102。
把条件的各部分分开。这里有两部分,一部分与体积有关,另一部分与面积有关。
我们可以几乎毫不犹豫地把整个条件正好分成上述两部分;但我们却不能
“直接”把这两部分写出来。我们需要知道如何计算体积和各部分面积。但是,
如果我们多懂一点儿几何学,我们不准把此条件的两部分加以重新叙述,使得译成方程式一事成为切实可行。下面我们在直线左侧所写下的问题基本上经过重新排列并加了解释以便译成代数语言。
(4)已知一直线方程及一点的座标,求一点与已知点对称于已知直线。
这是一个平面几何问题。
未知是什么?一点,其座标设为p,q。
已知是什么?直线方程,设为y=mx+n;一点,其座标设为a,b。
条件是什么?点(a,b)与(p,q),彼此对称于直线y=mx+n。
我们现在碰上了基本的困难,即把条件分为几部分,而每一部分都可用解析几何的语言表达出来。我们对困难的性质必须很好了解。把条件分成各部分的某种分解法在逻辑上可能是无懈可击的,但却于事无补。我们这里所需要的分解是适合解析表达式的分解。为了找出这样一个分解,我们必须回到对称的定义去,但我们同时要留意解析几何这个宝库。对于一直线对称,这意思是什么?什么样的几何关系可以在解析几何中表达得很简单?我们集中力量在第一个问题上,但我们也不应当忘记第二个。这样,最终我们可能找到一个下面将要叙述的分解。
53.进展的标志
当哥伦布及其伙伴西渡一个未知的大洋时,每当他们看见鸟就欢呼。他们把鸟看成是接近陆地的有利标记。这一点他们屡次失望。他们也注意到了别的标记。他们想,漂浮的海草或低垂的云层可能指示陆地,但他们又失望了。然而,有一天,标志变得越来越多。在1492年10月11日星期四,“他们看见了一种名叫矶鹞(sandpiper)的鸟,和船边的绿色芦苇。帆船‘品塔’(Pinta)号上的乘客看见了一段藤茎和一根圆棍,他们拣起了另一根短棒,上面似乎有铁器处理过的痕迹;还有另一小段藤茎,一种陆地植物,和一小块木板。‘尼娜’
(Nina)号帆船上的船员也看到了陆地的标志,看见一条树枝上面有草莓。每个人都为这些标志容光焕发、欢欣鼓舞”。实际上,他们在第二天就看见了陆地,
“新大陆”的第一个岛屿。
我们的任务可能重要,也可能不重要,任何类型的问题—一当我们正聚精会神地工作时,我们总热切地找寻进展的标志,这就象哥伦布及其同伴找寻接近陆地的标志一样。为了了解什么可以被合理地作为接近解答的标志,我们将讨论几个例子。
(1)例子。有一个国际象棋的棋局。比方说,我必须在两步内将死黑王。
而棋盘上有个白王假定在两步之内。在棋盘上有个白马离开黑王颇远,显然它是多余的。怎么办才好?起初我不得不躲开这个问题。但经过各种试验以后,我发现一个新走法并且发现这种走法将使显然多余的白马起作用。这次观察给了我新的希望。我把它看作有利的标志:即新走法有可能成为正确的走法。为什么呢?
在精心构造的棋局问题中,不会有多余的棋子。所以我们必须考虑棋盘上的所有棋子;我们必须利用所有的已知事项。正确的解答肯定利用所有的棋子,
乃至那只明显多余的白马。在这最后一方面,我所想到的新走法和我所希望要找的正确答案相一致。新走法看来很象正确的一着;它可能就是正确的一着。
在数学问题中考虑一个类似的情况是有趣的。我们的任务足将三角形的面积用其三边a,b,c表示。我已制定了某种计划。我多少有些清楚地知道我必须考虑哪些几何关系,以及我必须进行哪些计算。但我的计划是否行得通,我还不十分有把握。如果现在沿着我计划所描述的路线前进,我看到量
acb?+
进入我即将构造的面积表达式,那么我有充分的理由感到欣慰。为什么呢?
事实上,必须考虑到三角形的任何二边之和大于第三边。这包含了某种限制。已知长度a,b,c不能是十分任意的;例如,b+c必须大于a。这是条件的一个必要部分,我们应该利用整个条件。如果b+c不大于a,则我所找到的公式势必成为荒谬的。如果b十c-a为负——即,如果b+c小于a——则上述方根值为虚数,所以正是在所求表达式必然成为荒谬的情况下,力根不适于代表一个现实中的量。因此,我的含有方根的公式和计算面积的真正的公式具有同样的重要性质。我的公式看起来象个能成立的公式;它可能就是个能成立的公式。
下面还有一个例予。不久前,我想证明一个立体几何定理。不费多大周折,
我便找到第一个看来合适的说明;但接着我就停顿了下来。完成这个证明还缺少什么东西。在我放弃那一天,我比开始时有更清楚的见解,我知道应当怎样看证明,怎样弥补差距;但我还没有能力弥补它。第二天,我在好好地睡了一夜之后,重新考虑这个问题,于是很快想起一个平面几何中的类比定理。在刹那之间,我相信我已抓住了解答并且我想我有充分的理由这样相信。为什么呢?
事实上,类比是一个伟大的引路人。求解立体几何问题往往有赖于平面几何中的类比问题[见“类比”节,(3)-(7)]。这样,在我的例子中,从一开始就有个机会,即,所需的证明将利用迄今出现在我脑海中的某个平面几何定理作为引理。“这个定理看来象是我所需要的引理;它大概就是我所需要的引理”
——这就是我的推理。
如果哥伦布及其同伴曾致力于明确地推理的话,他们也可能用某种与此类似的方式进行推理。他们知道接近海岸时,海岸看起来是什么样子。他们知道近岸要比在公海上更常常看见从陆地来的空中飞鸟以及从海滨漂来的东西。很多人在他们以前远航归来回到故国港口的时候都曾看见过这些东西。在他们看见圣·萨尔瓦多岛这个值得纪念的日子的前一天,因为水中飘浮的东西变得如此频繁,他们就想:“看来象是接近某个陆地了,我们大概就是接近了某个陆地”,于是“每个人都为这些标志容光焕发,欢欣鼓舞”。
(2)进展标志的探索法性质。让我们坚持说明一下可能每个人已经明白的一点;由于很重要,所以我们应该弄得一清二楚。
以上各例所表明的推理类型值得我们注意和加以认真的考虑,虽然它仅产生一个似乎可信的指示而并非一种可靠的肯定。让我们用学究方式、详详细细地、详细到颇不自然的程度来重新叙述这类推理,其中之一是,
若我们接近陆地,我们常常看见鸟。
现在我们看见鸟了。
所以,大概,我们正在接近陆地。
如果没有“大概”这个词,上述结论将是彻头彻尾地荒诞不经。事实上,
哥伦布及其同伴曾多次看见过鸟,但不久就大失所望。只有一次,当他们看见矶鹞这种鸟,第二天,才发现了新大陆。
如果有“大概”这个词,则上述结论就合理而又自然了,但它却绝非一个证明,这不是一个经过证明的结论;它仅仅是个指示,是一个探索法式的建议。
如果忘记这样一种结论只不过是可能成立的而把它当作肯定成立的,那将是莫大的错误。如果你把一个探索法结论当作肯定的结论,你可能上当受骗,大失所望;但若你全然忽视探索法结论,你将毫无进展。最重要的进展标志是探索法的。我们是否应该信任它?我们是否应当随着它前进?应当随着,但要留神。
信任,但要警惕。并且永远不要弃而不用你的判断力。
(3)可清晰表达的标志。我们可以从另外一个观点来看以上各例。
在上述例子之一,我们把成功地利用了一个前此未用过的已知数据(白马)
当作进展的正面标志。我们这样看很对。实际上,求解一个问题主要是找出已知数据与未知之间的联系。除此之外,我们还应该(至少在一个陈述完善的问题中)利用所有的已知数据,把它们每一个都和未知数联系起来。因此,每当多一个已知数据发挥了作用都可十分正确地当作进展,当作向前跨进了一步。
在另一个例子中,我们把公式中适当地考虑了条件的基本部分当作进展的正面标志。我们这样看也很对。事实上,我们应当利用整个条件。所以,多考虑条件的一个部分,也可以合理地当作进展,当作沿正确方向移动了一步。
还有一个例子,其中我们把出现一个较简单的类比问题当作进展的正面标志。这也很合理。确实,类比是发明创造的主要源泉之一。如果其他方法失败,
我们应该尝试去想象一个类比问题。因此,如果有这样一个问题自然而然地涌现,我们当然兴高采烈;因为我们感到我们正在向解答靠拢。
经过上述几个例子之后,我们现在能够轻而易举地抓住普遍性的概念了。
在解题中确实有某些典型有用的智力活动(最有用的这类活动已列在本书的表中)。如果有这样一个典型智力活动获得成功(如果又有一个数据和未知数相联系——条件中又有一个句子被考虑——有一个较简单的类比问题被引入),则其成功就可被认作是进展的标志。在我们明了这个基本点以后,我们就能以某种清晰程度来表达其余进展标志的性质。我们所需做的全部事情,就是通读我们的表,并用新观点来看待各个问题和建议。
这样,清楚地了解未知数的性质意味着进展。把各个已知数据清楚地加以排列,使得我们可以容易回想起任一个,这也意味着进展。把条件做为整体清晰具体地加以想象可能意味着一个重大的进步;而把条件分为适当的各个部分也可能意味着向前跨进了重大一步。当我们找出一个我们很容易想象的图形或一个我们很容易记住的记号时,我们都有理由自信作出了某些进展。回忆起一个与我们的问题有关且早已解决的问题可能是朝着正确方向的决定性的一步。
如此等等,不必赘言。每个可清晰想到的智力活动都对应着某个可清晰表达的标志。我们的表,恰当地理解时,所列出的也是进展的标志。
我们表中的问题与建议是简单的,显而易见的,其实不过是普通常识。这点我们已经不厌其烦地再三说明过了。我们这里讨论的与之有关的进展标志也是如此。读这些标志并不需要神秘的学问,而只需要一点常识,当然,还需要一点经验。
(4)不太能清晰表达的标志。当我们专心一致地工作时,我们清楚地感觉到自己进展的步伐;当进展快时,我们高兴;当进展慢时,我们沮丧。我们虽能清楚地感觉到上述差别,但却不能清楚地指出任何明确无误的标志。情绪、
感觉和情况的总面貌可用来指出我们的进展。但它们不易表达。“它看来良好”,
或者“它不那么好”,朴实无华的人这样说,而比较装腔作势的人则说:“这是一个平衡性良好的计划”,或者“不,缺少某些东西.而这破坏了和谐性”。
然而,在原始的或模棚的表达后面,有一种不会错的感觉,我们自信地跟随它前进并且它常常引导我们到正确的方向。如果这种感觉很强烈并且是突然发生的,我们称之为灵感。人们通常并不怀疑他们的灵感,可是有时却会上当受骗。
事实上,我们对待这些感觉和灵感要象对待前述可清晰表达的进展标志一徉,
信任然而保持警惕。
永远随着你的灵感——但不能全信。
[那些向导性感觉的性质是什么?在诸如“平衡性良好”或“和谐”这样的艺术词汇微妙差别的背后,是否有某种比较不含糊的意义? 这些问题可能比较纯理论性而不那么实际,但是现在的上下文却指出了或许值得一提的答案:既然可清晰表达的进展标志与某个相当明确的智力活动的成效相联系,我们不妨猜测我们表达不那么清晰的向导性感觉可能类似地和其他较模糊的智力活动有联系——或者和那些性质上较多属了。“心理学”,较少属于“逻辑学”的活动有联系]。
(5)标志有什么帮助。我有一个计划。我很清楚地看出我应从哪儿开始,
并且应当首先采取什么步骤。但我还不十分清楚前进道路上的情况,我不那么肯定我的计划是否行得通。于是,在任何一种情况下,我都还有一大段路要走。
因此,我小心地沿着我计划中的方向前进并留心观察进展的标志。如果标志很少或者不清楚,我会变得更为犹豫。如果标志在很长时间里完全不出现,我可能会丧失信心,回过头去试走另一条路。反之,如果在我前进的时候,标志较为频繁地出现,如果它们大批涌现,则我的犹豫云消雾散,情绪高涨,我信心百倍地向前迈进,就象哥伦布及其同伴发现新大陆以前的表现那样。
标志可以指导我们的行动。不出现标志时可以警告我们别陷入死胡同,从而省去我们的光阴和徒劳的努力;出现标志时可以促使我们集中精力于正确的一点。
但是标志也可能是靠不住的。有一次,由于缺乏标志,我放弃了某个道路,
但有一个人在我之后沿着那条路再前进一点,得剑了一个重要发现——这使我极为恼火并悔恨良久。他不仅比我有更大的坚持性而且他还正确地认出了某个标志而我却未曾注意到。还有一种可能,我可能被有利于进展的标志所鼓舞,
而兴高采烈地顺着一条路走,但却碰上一个未曾料到的不可克服的困难。
是的,在任何个别情况下,标志都有可能错误地引导我们,但在大多数情况下,它们是正确地引导我们的。一个猎人可能偶而错误地解释了猎物的踪迹,
但总的说来他必定是正确的。否则他就不可能靠打猎为生了。
正确地解释标志需要经验。哥伦布的某些同伴根据经验肯定知道靠近海滨的海是什么样子,这样,他们才能认出告知他们接近陆地的标志。专家根据经验知道,当解答快到手的时候,情况看起来和感觉起来是怎样的,这样,他们才能认出指示他们已接近解答的标志。专家比无经验的人知道更多的标志,而且知道得更好;他的主要长处可能就在于有这样的知识。一个熟练的猎人能注意到猎物的踪迹,甚至能鉴定这些踪迹的新鲜程度或陈旧程度,而无经验的人却什么也看不出来。
具有特殊天资的人的主要长处可能就在于有一种超乎寻常的智力灵敏度。
根据超乎寻常的灵敏度,他感觉出微妙的进展标志或者注意到它们不出现,而天资平庸的人则感觉不出什么差别。
[(6)探索式的三段论法。在第(2)点,我们碰到一种探索法推理方式,这种推理方式值得作进一步的探讨并且值得给予一个专门术语。我们用下列形式把那段推理重述一遍以作为本节的开始,
如果我们正在接近陆地,我们常常看见鸟。
现在我们看见鸟了。
因此,说我们正在接近陆地较为可信。
横线以上的两个语句可称为前提,横线以下的语句可称为结论。整个推理的模式可称为探索式的三段论法。
这里所叙述的前提和第(2)点中的一样,但结论的措辞经过更仔细的推敲。
更好地强调了一个基本情况。哥伦布及其同伴从一开始就推测他们向西航行最终将发现陆地;并且他们对这种推测一定有某种信任,否则他们根本不会出发。
当他们航行时,他们把每一件事情,无论大小,都和他压倒一切的问题:“我们是否正在接近陆地?”相联系。他们的信心随着事件的发生与否而起伏,而每个人的信心起伏程度又随个人的背景与性恪有所不同。航程中的整个戏剧般的紧张气氛就与这种信心的增减有关。
上面摘录的探索式三段论为置信度的变化提供了合理的基础。有时这种变化在这类准理中起主要作用,这一点,在本节的叙述中表达得要比第(2)点中为好。
我们例子所给出的普遍模式可表示如下,
如我们所知,若A为真,则B也为真。
现在,发生了B为真的情况。
因此,A较为可信。
还可写得更简短些。
若A则B
B真
A较可信
在这个公式化的叙述中,横线代表“所以”这个词,并且表现了前提和结论之间的主要联系。]
(7)似真论证的性质。在这本小册予中,我们现在讨论一个哲学问题。我们讨论这一问题时将尽可能地讲求实际和口语比,并且尽量远离高深的表达方式,但尽管如此,我们的题目仍然是哲学的。它是关于探索式推理的,或拓广地说,它是一种虽很重要,但却非证明性论证的。这类论证,由于没有更好的术语,我们将称之为似真论证。
使发明家相信他的念头是个好念头的标志,在日常事务中指导我们的征兆、律师的情况证据、科学家的归纳根据,在许多形形色色事物中起怍用的统计迹象一一所有这类证据在关键的两点上是一致的。第一点,它们都不具有严格证明的肯定性。第二点,它们在获得那些本质上是新知识这一方面是有用的,
并且对于任何非纯数学、或非逻辑的知识来说,对任何与现实世界有关的知识来说,甚至是不可缺少的。我们可以把构成这类证据基础的推理叫做“探索式论证”或“归纳论证”,或者(如果我们想避免引伸现行术语的含义)叫做“似真论证”。我们采纳最后一个术语。
前面介绍的探索式三段论法可看成是似真论证的一个最简单、最广为流传的模式。它使我们想起证明论点的古典模式,即所谓“前提三段论法模式”。
我们将两种模式并列如下,
证明性的 | 探索式的
若A则B | 若A则B
B伪 | B真
A伪 | A较可信
对这两种模式进行比较是有教益的。这使我们看清楚似真论证(探索式论证、归纳论证)的性质,而在别处不易做到。
两种模式具有相同的第一前提,
若A则B。
所差者在第二前提。这两个语句恰好彼此对立,
B伪 B真
但它们有“相似的逻辑性质”,处于相同的“逻辑水平”上。在前提之后,出现了很大的差别。结论
A伪 A较可信
是在不同的逻辑水平上,并且它们和相应前提的关系具有不同的逻辑性质。
证明性的三段论法,其结论与前提具有相同的逻辑性质。此外,其结论是充分表达的并完全以前提为其支柱。如果我的邻居和我同意接受其前提,则不论我俩的口味或其他认识有何不同,我们在接受结论这一点上不会有什么合理的不一致。
探索式三段论法中的结论和前提的逻辑性质不同;它比较含糊而不如此清晰分明,表达得不那么充分。这个结论可与一个力相比拟,它有方向,有大小。
它把我们推向某个方向:A更可信。这个方向也有某种强度:A可以是甚为可信或稍微可信。这个结论是未充分表达的,并且其前提并不完全支持它。方向已表达了并由前提所蕴涵,但大小并未表达。对于任何有理性的人来说,前提已经涉及到A更可信(肯定不是更不可信)这一点。然而,由于我和我的邻居在秉性、
背景和其他未说明的理由方面有不同,我们在A究竟更可信到什么程度这一点上可以真正不同。
在证明性三段论法中,前提是结论所依据的充分基础。如果两个前提都成立,则结论也成立。如果我们接受某个新信息,它不改变我们对前提的信任,
那么它也不会改变我们对结论的信任。
在探索式三段论中,前提仅仅是结论所依据的一部分基础,即前提是结论基础中所充分表达的“可见”部分;这里还有一个未表达的不可见的部分,由其他什么东西所构成,大概是说不清的感觉,或者是说不出的理由。事实上,
可以发生这样的情况:我们接受某个新信息,它完全不触动我们对两个前提的信任,但却影响我们对A的信任,方式恰与结论中所表达的相反。以我们探索式三段论的前提为基础,去得到A较可信的结论仅仅是合理的。可是,第二天我可能找到理由,完全不影响前提,但却使A减少了可信程度,甚至明确地加以否定。
于是结论可能被动摇,并且甚至由于其基础中不可见部分的动摇而完全颠倒过来,虽然其前提,即可见部分,依然成立,稳定如故。
上述说明似乎会使探索式的、归纳的以及其他各种非证明性的似真论证的性质更容易理解一些,这些论证若从纯粹证明性的逻辑角度看来,似乎是令人大惑不解与不可捉摸的。完成这里所选择的课题研究看来需要有更多的例子,
考虑其他种类的探索式三段论法以及研究概率的概念;参见作者的“数学与似真论证”一文。]
探索式的理由是重要的,虽然它们并不证明任何东西。弄清探索法理由也非常重要,虽然在弄清楚的理由后面还有许多仍然模糊并且也许是更重要的理由。
54.特殊化
特殊化是从考虑一组给定的对象集合过渡到考虑该集合中一个较小的集合,或仅仅一个对象。特殊化在求解问题时常常很有用。
(1)例子。在一个三角形中,令内切圆的半径为r,外接圆的半径为R和最长的高为H。则
r+R≤H
我们必须证明(或推翻)这个定理;这是一个“求证题”。
所捉定理属于不常见的一种。我们几乎想不起任何有关三角形具有相似结论的定理。如果什么也想不起,我们可以试验一下这个陌生推论的某个特殊情况。最熟知的特殊三角形是等边三角形,对于它,
r=H/3,R=2H/3
所以,在这个特殊情况下,上述推论是正确的。
如果想不出其他念头,我们可以试验等腰三角形这一较一般的特例。等腰三角形的形式随顶点处的角度而变化。它有两个极限情况,一个是顶点角为o
°,另一个是180°。在第一个极限情况中,等腰三角形的底消失了,显然有
r=O,R=H/2
于是,推论得到证实。可是,在第二个极限情况中,所有三个高都消失了,而
r=0; R=∞,H=O
上述推论并未得到证实。于是,我们已经证明了所提定理不成立,这就解决了我们的问题。
顺便说一下,对于非平常的等腰三角形(其顶角接近于180°),显然上述推论同样不成立。所以,我们可以“正式”地舍去上述极端情况,考虑这些极端情况不那么“正统”。
(2)“以例外证明规律”。这是一句人所共知的格言,我们必须把它当作一个笑话来嘲笑某类逻辑的不严谨。如果我们严肃认真地对待事物,一个例外当然就足以不容争辩地反驳任何自封的规律或普遍性的命题。否定这类命题最常用、而且从某些方面说来是最好的方法,就是举出一个和它不一致的对象;
有些作者称这类对象,为反例。
未得证实的所谓普遍性命题是和对象的某一集合有关的;为了反驳这一命题,我们利用特殊化,从集合中检出一个与命题不一致的对象。前例(在第1点中)已表明如何这样做。开始时,我们可以审查任何简单的特例,即或多或少是随便选择的、任何容易试验的对象。如果试验结果表明这个特例与普遍性命题不一致,则此命题就得到反驳而我们的工作得以完成。但是,如果所审查的对象和命题一致,我们则有可能从它的审查中得到某种暗示。我们可能得到下述印象,即,该命题最后可能为真;并且得到某个建议,即我们应当沿什么方向去寻找证明。或者,象第1点中的例子那样,我们得到某个建议;即沿什么方向去找一个反例,换言之,我们应当试验哪些其他特例。我们可以修改我们刚才所审查的特例,把它加以变化,研究某个更一般的特例,寻找极端情况,象上述(1)中所示范的那样。
极端情况特别富于启发性。如果我们假定一个普遍性命题适用于所有的哺乳类动物,它必定可适用于乃至鲸这样一种不寻常的哺乳动物。让我们不要忘记鲸这个极端的情况。审查它以后,我们有可能否定这个普遍性命题;这是个很好的机会,因为这种极端情况很容易被普遍性命题的发明者所忽略。但是,
如果我们发现普遍性命题甚至在此极端情况下也被证实,则由这一验证所得出的归纳证据将是强有力的,这正是由于想根据它否定的期望曾经很强烈的缘故。
所以,我们很想把本节开始时的格言修改成:“以期望中的例外检验规律”。
(3)例。已知两船的速度及其在某一时刻的位置;两船各以匀速沿直线航道行驶。求当两船相距最近时,彼此之间的距离。看成质点。
已知数是什么?移动物体的初始位置及速度。这些速度的大小与方向都是固定的。
条件是什么?求出当两船相距最近时(即当两个动点(船)彼此最靠近时)的距离。
画张图。引入合适的符号。在图19中,点A与点B标出了两船所给定的初始位置,有方向的线段(向量)AP及BQ代表给定的速度,即第一只船沿着过点 A及点P的直线方向前进,并在单位时间内走过距离AP。类似地,第二只船沿直线BQ
前进。
图19
未知数是什么? 两船间的最短距离。一船沿AP航行,另一船沿BQ航行。
现在我们已经清楚应该求什么了;
但是如果我们只希望用初等方法,则我们可能仍束手无策。问题不太容易,
但其困难有某些特色,我们不妨表达为:“变化太多。”初始位置A,B和速度
AP,BQ都可用各种不同方式给定;事实上,四点A,B,P,Q是可以任意选择的。
现在,不论已知数据是什么,所求解答必须都能适用,而我们迄今尚未看出怎样才能使同一个解答适用于所有上述可能的情况。出于“变化太多”这样的感觉,下述问题与解答可能终久会涌现出来,
你能不能想出一个更好着手的有关问题?一个更特殊的问题?当然,有极端情况,例如其中有一个速度为0。是的,在B点的船可以抛锚,Q点可以与B点重合。从停泊的船到移动的船之间的最短距离垂直于后者所移动的直线。
(4)如果上述念头出现时,预感到“前面路还远”,但又感到该极端特例(它看来似乎太简单而不合用)会起某些作用——则这个念头确实是一个好念头。
这里有一个与你的问题有关的问题,就是你刚才解决的那个特殊问题。你能不能利用它?你能不能利用它的结果?为了能利用它,你是否应当引入某个辅助元素?应该利用,但怎么利用呢?怎样才能把B停止不动这一情况的结果用到B
向前移动的情况去呢?停止不动是运动的特例。而且运动是相对的——所以,不论B的给定速度是多少,我都可把B看作是停止不动的!下面更清楚地表达了这个念头:如果我们使由两只船所组成的整个系统具有同一个均匀速度。则两船的相对位置不变,相对距离保持不变,特别是问题所求的两船最短相对距离也保持不变。现在,我可以加上一个运动,把其中一个船的速度减为O,这样就把问题的普遍情况简化为刚才解决的特殊情况。让我在BQ与AB上加上一个与BQ方向相反,大小相等的速度。这是个辅助元素,它使我们有可能利用特殊结果。
作出最短距离BS,见图20。
图20
(5)上述【第(3),(4)点】解答用了一个值得我们分析与记住的逻辑模式。
为了解决我们原来的问题【(3)中前几行】,我们先解决另一个问题,该问题我们恰当地称之为辅助问题【(3)中最后几行】。此辅助问题是原问题的一个特例(即极端情况,两船之一停止不动)。原问题是提出来的,而辅助问题是在求解过程中创造出来的。原问题看起来困难,而辅助问题的求解比较直接了当。作为特例的辅助问题实际上比原问题的期望更小。那么,在辅助问题的基础上居然能解决原问题,这是为什么呢?这是因为在把原问题化简成辅助问题时,我们增加了一个重要的补充说明(关于运动的相对性)。
我们成功地解决了我们的原问题归功于两点。第一,我们想出了一个有利的辅助问题。第二,我们发现了一个从辅助问题过渡到原问题的补充说明。我们用两步解决了原问题,就象小河当中正好有块合适的石头可作临时的踏脚石,
我们用两步过河一样。
总之,在解决更困难的、期望大的、具普遍性的原问题时,我们利用了不太困难的、期望小的、特殊的辅助问题作为踏脚石。
(6)特殊化还有许多其他用途,这里无法一一讨论了。我们仅仅提一下,
它在检验解答方面也是有用的【见“你能检验这结果吗?”第(2)点】。
一种多少有点原始的特殊化类型对教师常常很有用。这就是将问题中的抽象数学元素给以某些具体解释。例如,问题中有个长方体,教师可取教室为例(见第8节)。在立体解析几何巾,教室的一角可取为坐标原点,地板和两面墙作为坐标平面,两个水平的棱和一个垂直的棱作为坐标轴。在解释旋转面的概念时,
教师可以用粉笔在门上画一根曲线,然后慢慢开门。这些肯定都是些简单的演示,然而凡是能使学生熟习教学的事,我们都不应当忽视。数学是一门非常抽象的科学,但同时它又应当非常具体地加以阐述。
55.潜意识的工作
有一天傍晚,我想和一位朋友讨论一位作者,但我怎么也想不起他的名字。
我烦恼不堪,因为我对他的故事之一记得很清楚。我还记得一些有关作者本人的故事,很希望谈谈这些故事;事实上,除了名字以外,我什么全都记得。我冥思苦想,仍然没有想起他的名字。可是第二天清晨,我刚一想到头天晚上的烦恼时,竟不费吹灰之力就想出了他的名字。
读者很可能也有这方面的类似经验。如果读者是一个热心的解题者,他在解题方面大概也有某些类似的经验。常常会出现下列情况,你对一个问题束手无策;你奋力工作,然而毫无进展。但当你休息一夜或者中断几天之后,突然出现了一个好念头,问题便迎刃而解了。这与问题本身的性质关系甚微;无沦足一个忘却的词,字谜中的一个难字,一封恼人信件的开头或者数学题的解,
都可能这样想出来。
这类偶然发生的事给人的印象是潜意识工作。事实是:一个问题;在搁置一段时间以后,可能返回到意识中来,而且基本上已澄清,比当初搁置起来时要接近解答得多。是谁使它澄清了呢?是谁使它接近解答的?显然,不是别人,
是解题者本人在潜意识地工作。除此之外,很难作出其他的解答。虽然心理学家已经发现了另一种解答的起始部分,这种解答也许有朝一日会变成令人更为满意的。
不论潜意识工作的理论有没有什么价值,肯定存在着一个限度,超过这个限度我们就不能迫使意识有所反应。有某些时刻,这时我们最好暂时把问题束之高阁。“谋之于枕”是一句古老的忠告。稍事休息之后,我们可能事半而功倍。“今日不成,明朝可能”,这是另一句古谚。然而我们不希望把将来还要回过头来再搞的问题没有什么进展就搁置一旁,当我们搁置这个问题时,问题至少应当解决了一些,在某个方面有所澄清。
只有下述问题重新拿起时才有所进展,即那些我们热切希望求解或者我们曾经为之十分紧张地工作过的问题。看来,为使潜意识工作,主观意识的努力与紧张是必不可少的。不管怎样,如若不然,则问题未免过于轻而易举了,因为我们只要睡上一觉,袖手等待一个好念头便可解决了棘手问题。
自古以来,我们总把突然而来的好念头看成一种灵感,一种上帝的恩赐。
你必须工作或者至少有强烈的愿望,才配得到这种恩赐。
56.对称
对称有两个意义,一个是较为常用的、特定的几何意义,另一个是不太常见的,普遍化的逻辑意义。
在初等立体几何中考虑两类对称:对称于一个平面(称为对称平面)和对称于一个点(称为对称中心)。人体看起来好象很对称,但事实上不然,许多内脏的位置并不对称。一个雕像可以完全对称于一个垂直平面,以致其两半看起来可以完全“互换”。
,对称”这个词的较为普遍接受的一个含意是:一个整体若有可互换的诸部分就称为对称。有各种各样的对称;它们可互换部分的数目不同,可互换部分的功能不同。这样,一个正方体是高度对称的,因为它的六面可互换,8个顶点可互换,12条边也可以互换。表达式
yz+zx+xy
是对称的;因为三个字母x、y、z中任意两个都可以互换而表达式不变。
从一般意义上讲,对称对我们的论题很重要。如果一个问题以某种方式对称,则我们注意其可互换部分常会得到某种好处,而且常常很值得用同样的方式处理有同样作用的部分(见“辅助问题”一节,第3点)。
我们要尝试对称地处理对称的东西,而不要随便破坏任何自然对称性。然而,有时我们不得不用非对称方式处理自然对称的东西。一付手套肯定是对称的;但却没有人完全对称地处理过,没有人是同时戴上两只手套的,总是先戴上一只,再戴上另一只。
在检验结果方面,对称也可能有用。见第14节。
57.新术语和老术语
描述解题活动的新老术语常常语意含混。至于解题活动本身,每个人都很熟悉并且常常加以讨论,但它却象其他智力活动一样,难于描述。由于缺乏系统的研究,没有描述它的专门术语,而某些常用的半专业性术语,因为被不同的作者用于不同的含意,反而增添了混乱。
下面列出的包括若干本书所采用的新术语和避而不用的老术语,也包括一些尽管词意含混但却保留使用的老术语。读者除非将其概念用实例加以巩固,
否则可能被以下术语的讨论弄得晕头转向。
(1)分析。分析一词已由帕扑斯简洁地定义过了,这是个很有用的术语,它描述制定计划的典型方法,即由未知数(或结论)开始,朝着已知数据(或前提)
的方向,倒着干。不幸,这个词已经获得了各种很不相同的意义(例如,数学分析,化学分析,逻辑分析)。因此,本书遗憾地避而不用。
(2)条件。条件把“求解题”的未知数和已知数据联系起来[见“求解题,
求证题”一节,第(3)点]。在这个意义上,它是一种清楚、有用、不可或缺的术语。常常有必要将条件分解为几个部分[在“分解与重新组合”一节第(7)、
(8)点中的例子里,分解成部分(I)与部分(Ⅱ)]。现在,通常把条件的每一部分都称为一个条件。这种有时惹麻烦的含混不清,很容易用引入某个专门术语来代表整个条件中的各个部分这一办法而加以避免。例如,我们不妨把这样一个部分称做一个“子条件”。
(3)前提。前提是较常见的一类数学定理中的基本部分(见“求解题,求证题”一节,第(4)点)。在这个意义上,这个术语是完全清楚并且令人满意的。
困难在于此前提的每一部分也称为一个前提,因而使得前提可能是由好几个前提所组成。补救的方法是:可以把整个前提的各个部分叫做一个“子前提”或其他类似名词(请与前面“条件”一节的说明相比较)。
(4)主要部分。一个问题的主要部分已在“求解题,求证题”一节第(3),
(4)点中定义过。
(5)求解题,求证题。这是一对新术语,我们遗憾地引进它们来代替历史悠久、然而其含义已被流行用法弄混而无法补救的术语。在希腊数学课本的拉丁文本中,两类问题的共同名字是“propositio”;,求解题”称为“problema”,
而“求证题”称为“theorema”。在老式数学语言中,proposition、problem,
theorem仍然保留着这种“欧几里得”式的含义,但在近代数学语言中,已经完全变了样;这就说明我们引进新术语是合理的。
(6)渐近论证。“渐进论证”一词曾经被各种作者用于各种含义,而有些作者则采用古老的含义:“综合”(参见下面第9点)。后一用法语义虽不含混,
但我们这里避而不用这个术语。
(7)回归论证。这一名词曾被某些作者用于古老的含义:“分析”[请与第
(1),(6)点相比较]。此术语虽不含混,但我们这里避而不用。
(8)解。如果就纯数学意义而言,它是一个完全清楚的术语;它表示满足
“求解题”条件的任何对象。这样,方程x^2一3x+2=0的解是它的根:1与2。不幸,这个名词还有其他不是纯数学的含义并且和其数学含义一起被数学家所袭用。“解”一词也可意味着“解题过程”或“解题中所做的工作”;当我们谈到“难解”时,我们就是用的这个意义。“解”也可意味着解题所做的工作结果;当我们说“巧妙的解”时,我们用的是这个含义。于是,有时会出现这种情况,即我们必须在同一句话里谈到满足问题条件的对象、为得到这个对象所做的工作以及此工作结果时,如果我们放任自流地把三类事物全都称为“解”
的话,那么我们的文意就无法十分清楚了。
(9)综合。“综合”一词曾被帕扑斯用于一个已很好定义过的意义上,值得我们加以保留。然而很遗憾,由于和反义语“分析”相同的原因,本书中避而不用。
58.量纲检验
量纲检验是一种众所周知的、快速而有效的检验几何或物理公式的方法。
(1)为了回忆检验怎样进行,我们考虑一个正圆台。令
R为下底半径,
r为上底半径,
h为圆台的高,
S为圆台的测面积如果R,r,h已知,则S显然确定。我们求出表达式
S=π(R+r) 22)( hrR +?
对于这个表达式,我们希望用量纲检验。
一个几何量的量纲是显而易见的。R,r,h是长度,如果我们采用科学单位,用厘米量度,则它们的量纲是厘米。面积S用平方厘米量度,其量纲为厘米
^2。上式中的π=3.14159……仅仅是个数;如果我们想给一个纯数量指定一个量纲,则它必然是厘米0=1。
一个和的各个项必须有相同的量纲,并且该量纲也必须是和的量纲。这样
R,r,和R+r具有相同的量纲,即厘米。(R—r)2和h2这两项也有相同的量纲厘米2(正如它们必须如此那样)。
乘积的量纲是其因子量纲之积,关于幂也有类似的规则。在我们正在检验的公式两边把各个量代之以其量纲,我们得到
厘米2=1·厘米· 2厘米
这显然成立,此检验未检测出公式有何错误。于是公式通过了检验。
其他的例子,参见第14节和“你能检验这结果吗?”一节,第(2)点。
(2)对问题的最终结果或中间结果,我们自己的工作或他人的工作(量纲检验非常适用于检验考卷的错误)都可以应用量纲检验,还可用于我们收集到(想到)的公式,乃至我们所猜测到的公式。
如果你想起球的面积与体积公式是4πr3和 34
3rp
,但没有把握哪个公式是哪个的,那么量纲检验很容易消除疑团。
(3)量纲检验在物理中甚至比在几何中还更重要。
让我们考虑一个“简单的”摆,即一个小小的重物用一根线悬挂起来,线长我们认为不变,而线的重量我们认为可忽略不计。令l代表线长,g代表重力加速度,T代表摆的周期。
根据力学方面的考虑,T仅仅依赖于l和g。但依赖的形式是什么?我们可能记得或猜测到
T=Clmgn
式中c,m,n是数值常数。即,我们假定T正比于l和g的某个幂lm,gn。
我们注意量纲。由于T为时间,它的单位是秒,因此量纲是秒。长度l的量纲是厘米,加速度g的量纲是厘米·秒-2,而数值常数C的量纲是l。量纲检验得出方程
秒=1·(厘米)m·(厘米·秒-2)n

秒=(厘米)m+n·秒-2n
现在,在公式两侧,基本单位厘米与秒必须有相同的幂,于是我们得到
0=m+n
所以
n=1/2 m=-1/2
因此,周期T的公式必定为下列形式,
T=Cl1/2g-1/2=C gl
在本例中,量纲检验的收获很大,然而量纲并不能产生一切。首先,它并不给出有关常数C数值方面的信息(事实上,C是2x)。其次,它不提供公式有效使用范围方面的信息;公式只对摆的小幅度振动有效并且还仅仅是近似公式(对
“无限小的”振动,它才是精确的。)尽管有上述限制,但毫无疑问,考虑量纲使我们得以很快地并且以最初等的方法预见到结果的基本部分,而此结果的彻底讨论要用高深得多的方法。在许多类似情况下都是如此。
59.未来的数学家
未来的数学家应当是一个聪明的解题者,但仅仅是一个聪明的解题者还不够。在适当的时刻,他应当解决重要的数学问题;而他首先应当找出他的天赋特别适合哪一类问题。
对于他,工作中最重要的一部分是回顾已完成的解。在研究他自己的工作过程和解答的最后形式时,他可能发现需要观察研究的事物真是形形色色、无穷无尽。他可以沉思问题的困难所在以及关键性的念头是什么;他可以试图找出究竟是什么阻碍了他,是什么最后帮助了他。他可以寻求简单的直观概念:
你能不能一下子看出它?他可以比较与发展各种方法:你能不能用不同的方法导出这结果?他可以尝试把当前的问题与一个早已解决的问题相比较从而澄清当前的问题;他可以尝试去发明新问题,而这些新问题可以在他刚才所完成的工作基础上加以解决:你能不能把这结果或这方法利用到某个其他问题上?对他所解决的问题尽可能完全地消化吸收、融会贯通,则他可以得到井然有序的知识,
以备随时应用。
未来的数学家,正象其他每个人一样,是通过模仿和实践来学习的。他应当寻求要模仿的正确模型。他应该观察一个激励人心的教师。他应该和一个有能力的朋友竞赛。然后,可能是最重要的,他所阅读的东西应当不仅仅限于流行的教科书而应当阅读优秀作者的著怍,直到他找到一个他天然倾向于模仿其方法的作者为止。他应当享受并探求什么对他看来是简单的,或者是有启发性的,或者是漂亮的。他应当解题,选择符合他思路的问题,冥思苦想其解答并发明新问题。用上述方法以及所有其他方法,他应该力争做出他的第一个重要发现;他应当发现他自己的爱好与厌恶、他的情趣和他自己的擅长。
60.聪明的解题者
聪明的解题者经常问自己那些类似于我们的表中的问题。他大概自己发现过这类问题;或者从别人那里听到过这样一个问题,然后自己发现了其恰当用法。他再三重复同一个刻板定型的问题可能完全出于不自觉。或者这问题是他的一个特殊爱好;他知道该问题是他智力活动的一部分,适合于工作的这样和那样的阶段,于是他通过提问正确的问题,唤起正确的智力活动。
聪明的解题者会发现我们表中的问题与建议很有用。他可能对说明表中某一个问题的阐述与实例理解得十分好,他可能猜到该问题的正确用法;然而他并不能真正的理解,除非他亲自碰到过该问题在他工作中所试图引起的做法,
同时在体验了问题有用以后,亲自发现了其正确用法。
聪明的解题者应当准备好提问表中所有的问题,但是只有当他仔细地考虑过手头的题目并且经过他自己不抱成见地判断之后他才能提问题,否则一个问题也别问。事实上,他必须亲自识别当前情况是否与他过去成功地应用该问题的其他情况相类似。
聪明的解题者首先要力争尽可能彻底而又清楚地了解题目。可是仅仅了解还不够,他必须全神贯注于该题,他必须渴望得到解答。如果他不能激发真正的解题愿望,他最好将该题束之高阁。真正成功的公开秘诀是:把你的全部身心投入到你的题目中去!
61.聪明的读者
一本数学书的聪明读者希望两件事,
第一,看出当前这步论证是正确的。
第二,看出当前这步的目的。
一个数学讲座的聪明的听众具有相同的愿望。如果他不能看出当前这步论证是正确的,甚至怀疑它可能不正确,他可能表示非议并提出问题。如果他不能看出当前这步的任何目的,也猜不出任何理由,他通常甚至不能形成清楚的反对意见,他虽不提出非议但却灰心而厌烦,并且丢失了论证的线索。
聪明的教师和教科:书的聪明作者应当把上述观点牢记心头。正确地写与讲肯定是必要的,但还不够。如果在书中或黑板上的推导很正确,但接连几步的目的难以理解,如果读者或听众不能理解怎样才能在力所能及的范围内找出这样一个论证,如果不能从推导中得到他自己如何找出这样一个论证的启示,
那么这种推导是难于接受和无教益的。
我们表中的问题与建议,在强调论证的目的和动机方面,可能对作者和教师很有用。在这方面特别有用的问题是:“你是否利用了所有的已知数?”作者或教师可以通过这个问题表明有充分理由考虑一个前此未曾用过的已知数据。
读者或听众为了理解作者与教师为什么考虑某个元素也可以提出相同的问题,
同时,读者或听众在提问这个问题时可能会感到仿佛他自己也能发现论证的这一步。
62.传统的数学教授
民间传说中的传统数学教授是心不在焉的。通常他在公众面前出现时,仿佛每只手里都握着一把已丢失的伞。他宁可面向黑板,背对学生。他写a时,口里讲b,意思指c;但实际应当是d。他的某些说法是世代相传的,
,为了求解微分方程,你盯着它瞧,直到你得到一个解答。”
,这个原理太一般化,以致不可能有什么具体应用。”
,几何是在不正确的图形上进行正确论证的艺术。”
,克服困难的方法就是绕过它。”
“方法与手段之间有什么差别?方法就是你用过两次的手段。”
你毕竟还可以从这位传统数学教授那里学到点什么。我们但愿那些教师
(学生从他那里学不到任何东西的)不要变成传统的才好。
63.问题的变化
一个飞虫尝试穿过玻璃窗逃遁,它不断重复同一个毫无希望的动作,不断撞击玻璃,而不去试试那扇开着的、它原先从那儿进来的窗子。老鼠则比飞虫聪明,它被捕在鼠笼中,它在两根笼柱之间试试,想钻出去,然后又试试旁边两根,后来又试试别的柱子;它变化它的试验,它探索各种可能。人能够,而且也应当能够更聪明地变化他的试验,以更深入的理解来探索各种可能,通过自己的错误与缺点来学习。“试试,再试试”是个通用的忠告。这是个好忠告。
飞虫、老鼠和人都照这个忠告办事;但若其中一个比其他的办得更成功,这是因为他更聪明地变化其问题的缘故。
(1)在工作结束时,我们得到了解答,我们对问题的概念比开始时更完整、
更充分。由于我们期望从问题的初始概念进展到更充分、更适当的概念,我们试验各种立足点,从各个方面观察该问题。
解题中的成功有赖于选择正确的方面,有赖于从好接近的一侧攻击堡垒。
为了找出哪个方面是正确的方面,哪一侧是好接近的一侧,我们从各个方面、
各个侧边去试验,我们变化问题。
(2)问题的变化是必要的。这一事实可用各种方法加以解释。这样,从某种观点看来,解题中的进展被看作是对早先获得的知识进行了动员与组织的结果。我们必须从记忆中汲取某些元素并且放进当前的问题中。变化当前的问题,
有利于我们去汲取这样的元素。怎么做呢?
我们记忆事物是通过一类被称为“内心联想”的“接触活动”来进行的;
我们现在脑中的事物倾向于使我们回想过去曾和这事物接触过的东西(这里因篇幅所限,同时也无必要去对联想的理论作较简略介绍或者讨论其限度)。变化问题使我们引进了新的内容,从而产生了新的接触,产生了和我们问题有关的元素接触的新可能性。
(3)没有专心致志,我们不能希望解决任何有价值的问题。但若把我们的注意力专心致志地集中于同一点上,我们很容易疲劳。为了保持我们的注意力而又生气勃勃,我们必须不停地变换我们的对象。
如果我们的工作有进展,有事情可做,有新内容要审查,我们的注意力就会集中,兴趣盎然。但如果工作没有进展,我们的注意力就会松懈,兴趣索然。
一旦我们对问题感到疲劳,我们思想开小差,就会有完全丢失该问题的危险。
为了避免这种危险,我们必须给自己提一个有关这个问题的新问题。
新问题展现了接触我们以前知识的新可能性,它使我们作出有用接触的希望死而复苏。通过变化问题,显露它的某个新方面,新问题将重新使我们的兴趣油然而生。
(4)例子。已知棱台的底面是正方形。下底的边为a,上底的边为b,棱台的高为h,求棱台的体积。
这问题可向熟悉棱柱与棱锥体积公式的班级提出。如果学生不能用他们自己的想法前进,教师可以从变化问题的已知数开始。我们现在从a>b的棱台开始。
当b增加到与a相等时会出现什么情况?棱台变成一个棱柱,而体积成为 a2h。当
b减小到等于0时会出现什么情况?棱台将成为一个棱锥,而其体积成为a2h/3。
这种已知数的变化,首先可以引起对问题的兴趣。其次,它可以提示我们去以这种或那种方式利用上面所提到的关于棱柱与棱锥的结果。无论如何,我们已发现了我们最后结果的明确性质;最终的公式必须是这样,当b=a时,它化简成a2h,当b=0时,化简成a2h/3。预见到我们所打算求的结果具有什么性质这是一大长处。 这种性质可以提出有价值的建议,而在任何情况下,当我们已经找到最后公式时,我们可借助这种性质来检验公式。这样,我们就预先回答了下述问题:“你能检验这结果吗?”[参见“你能检验这结果吗?”一节第(2)
点]。
(5)例子。已知梯形的四个边a,b,c,d。作出此梯形的图形。
我们令a为下底,c为下底;a与c平行但不相等,而b与d不平行。如果没有别的念头出现,我们可以从变化已知数据开始。
我们从一个a>c的梯形开始。当c减小到变为0时发生什么情况?梯形退化成三角形。三角形是一个熟悉而又简单的图形,我们能用各种数据作图;把这个三角形引进我们的图中可能会有某些好处。我们只引入一根辅助线,梯形的一根对角线,便引进了这个三角形(图21)。但我们审查这个三角形时,但我们发现它几乎没有什么用处;我们已知其两边a与d,但我们应该有三个数据才能作图。
图21
让我们试试别的。当c增加到等于a,会发生什么情况?梯形变成一个平行四边形。我们能利用它吗?稍许加以审视(图22),我们就会注意到在画平行四边形时,在原梯形图上所附加的那个三角形。这个三角形很容易作图,因为我们知道三个数据:其三边为b,d和a—c。
图22
变更原问题(作梯形)后,我们得到一个更好接近的辅助问题(作三角形图)。利用辅助问题的结果,我们容易地解决了我们原来的问题(但必须完成平行四边形)。
我们的例子是典型的。我们第一次尝试失败了,这也是典型的。但回顾这点,我们可以看出第一次尝试并非如此无用。在其中含有某些念头;特别是,
它给我们一个机会去想起用三角形作为达到目的的手段。实际上,我们是通过修改第一次不成功的试验才达到第二次成功的试验的。我们变更c:先试验把它减少,然后试验把它增加。
(6)如上例所述,我们经常需要试验对问题作各种修改。我们必须一再地变化它,重新叙述它,变换它,直到最后成功地找到某些有用的东西为止。我们可以从失败中学习;在不成功的试验中可能存在有好念头,并且我们通过修改一个不成功的试验可以达到一个较成功的试验。在各种试验以后,我们经常得到一个更好下手的辅助问题(如上例所示)。
(7)存在着某些变化问题的模式,它们是典型有用的,例如“回到定义去”,
“分解与重新组合”,“引入辅助元素”,“普遍化”,“特殊化”,以及利用“类比”。
(8)在前面第(3)点谈到可引起我们兴趣的新问题,所谈内容对于正确使用我们的表很重要。
教师可以利用这张表去帮助他的学生。如果学生有进展,无需帮助,教师就不应当问他任何问题而应任其独自工作,这对他独立工作的能力显然有益。
但当学生停滞不前时,教师当然应该尝试找一个适当的问题或建议去帮助他。
这是因为担心学生对问题感到疲倦而扔下它,或者失去兴趣从而由于纯粹的漠不关心而铸成大错。
我们在解决自己的问题时也可以利用这张表。为了正确地使用它,我们照上面例子那样进行。当我们进展顺利时,当新的标记自动涌现时,如果拿一些多余的问题妨碍我们的自然进展,简直是愚蠢。但当我们进展不顺利,想不起什么念头,我们则有对问题感到厌倦的危险。这时正是去思索某个有帮助的一般性念头、或思索表中哪个问题与建议可能合适的好时机。而任何一个可能指明问题新方面的问题,都值得欢迎;因为它可以引起我们的兴趣,可以使我们继续工作、继续思索。
64.未知数是什么?
所求的是什么?找什么?你所设想寻求的是什么?
已知数据是什么?已给定的是什么?
条件是什么? 未知数是由什么条件与已知数据相联系的?
教师可用上述问题去检查学生对题目的了解程度;学生应能清楚地回答上述问题。此外,上述问题还把学生的注意力引向“求解题”的主要部分:未知数,已知数据和条件。由于一再需要考虑这些部分,所以在求解过程中的后面几个阶段里还可能经常重复提出上述问题[参见第8、10、18、20节中的例子;
“建立方程”一节中的第(3)、(4)点;“实际问题”一节中的第(1)点;“谜语”
一节及其他]。
这些问题对解题者至关重要。他检验自己对问题的了解,他集中注意力于题目的这个或那个主要部分。解答,本质上就是把未知数和已知数据联系起来。
所以,解题者必须再三集中注意于上述元素,并询问:未知数是什么?已知数据是什么?
题目中可能有许多未知数,或者是条件可能具有必须分开考虑的各个部分,或者我们希望考虑某个已知数据本身。因此,我们可以利用上述问题的各个变型,如未知数是些什么?第一个已知数据是什么?第二个已知数据是什么?
条件的各部分是些什么?条件中的第一个子条件是什么?
“求证题”的主要部分是前提与结论,其相应问题是:前提是什么?结论是什么?我们可能需要变更用词和句式,或者修改上述经常使用的问题,例如:
你假设了什么?你的假定中的各部分是些什么?(参见第19节中的例子)。
65.为什么要证明?
关丁牛顿有个传说:当他是个青年学生的时候,他开始学习几何,阅读欧几里得的《几何原本》,这在他那个时代是很普通的。他阅读定理,认为它们成立,就略而不读其证明。他奇怪为什么别人要煞费苦心地证明如此明显的东西。但许多年以后,他改变了他的意见,并且对欧几里得称赞备至。
且不管这故事是真是假,但这个问题确实存在:为什么我们要学习证明或教授证明呢?何者更可取?是完全不证明还是每点都证明?还是有的证,有的不证?
如果只是有的证,有的不证,应证明哪些?
(1)完全证明。对某类逻辑学家来说,只有完全证明才算证明。想成为一个证明,必须不留空隙,没有破绽,也没有任何不肯定性,否则它就不是证明。
在我们日常生活中,或在法律程序中,或在物理科学中,能不能根据这样一个高标准找到完全证明呢?几乎找不到。所以,很难理解,我们是怎样得到这样一个关于严格的“完全证明”的概念的。
我们不防带点夸张地说:人类是从一个人和一本书(即欧几里得和他所写的《几何原本》)学到这个概念的。无论如何,学习平面几何原理提供了得到严格证明这一概念的迄今最好的机会。
我们把下列定理的证明作为一例:在任何一个三角形中,三角之和等于两个直角。图23是我们大多数人已有的知识,只须稍加解释。过顶点A作一直线平行于边BC。由于内错角相等,三角形中在B角与C角等于图上所指出的在 A点的某一个角。三角形的三个角与具有一公共点A的三个角相等,后者形成一平角或两个直角。于是定理得证。
图23
如果一个学生学过数学课,而并不真正懂得几个象上面那样的证明,他有权向学校和教师提出尖锐批评。事实上,我们应当分清什么是较重要和什么是不那么重要的。如果学生不熟悉某个具体的几何事实,他的损失并不大;因为在他今后的生活中很少用到这些事实。然而如果他未能熟悉几何证明,他损失的却是“真实证据”的最佳与最简单的例子,并且他也损失了获得严格论证概念的良机。而没有这种概念,他就缺少一种真正的标准来比较现代生活中针对他的各式各样的所谓证据。
总之,如果普通教育打算给学生以直观证明与逻辑论证的概念,那么就必须重视几何证明。
(2)逻辑系统。几何学,正如欧几里得《几何原本》所表明的那样,并非仅仅是事实的一种汇集而是一个逻辑系统。公理、定义和命题不是按随便(随机)
的序列排列的,而是按一种完美的顺序安排的。每一命题都这样安排,使得它能以前面的定理、定义与命题为基础。我们可以把命题的这种安排看作是欧几里得的主要成就,并把它们所组成的逻辑系统看作是《几何原本》一书的主要优点。
欧几里得的几何学还不仅是一个逻辑系统,它还是这类系统的第一个和最伟大的范例,其他的科学已经并且将继续尝试去模仿它。其他科学——特别是那些离几何特别远的学科,如心理学、法学——是否应该模仿欧几里得的严格逻辑呢?这是个可辩论的问题;但是,一个人若不熟悉这种欧几里得系统,他便没有能力参加这种辩论。
现在,几何体系是通过证明的“粘合”而成的。每个命题与其前面的公理、
定义以及命题借助于一个证明而联系起来。不了解这种证明,就无法了解这体系的本质。
总之,如果普通教育打算给学生以逻辑体系的概念,它必额重视几何证明。
(3)助记体系。作者并不认为直观证明、严格论证和逻辑体系对任何人是多余的。但是也可能有这样的情况:由于时间不够或其他原因,学习这些概念并不是绝对必要的。不过,即使在这种情况下,我们也希望学习证明。
证明给出证据;在这样做时,它们组合成逻辑系统;它们帮助我们记住组合起来的各项内容。以上面图23为例,该图使三角形三个角之和等于180°这一事实变得更加清楚了。该图并使上述事实与内错角相等这一事实联系了起来。
这种相互联系的事实,无论如何是比较有趣的,因之也比孤立的事实记得牢固。
所以,图形使我们牢记两个互相联系的几何命题并且最后很可能使该图和该命题成为我们不可剥夺的知识财富。
现在我们讨论那些我们认为不必获得普遍性概念而只希望知道某些事实的情况。即使在这种情况下,事实也必须在某种联系、某种体系中提出来,因为孤立的内容不容易学到手而且容易忘却。这里,我们欢迎把事实简单地、自然地、合适地结合起来的任何一种联系。这种体系并不需要建立在逻辑基础上,
它只需设计成能有效地帮助记忆即可;即,它必须是所谓“助记体系”。然而,
即使从纯助记体系的观点来看,证明也可能有用,尤其是简单的证明。例如,
学生必须学习关于三角形三角之和这一事实以及关于内错角的另一事实。难道有任何方法能比图23更简单、更自然或更有效地帮助我们记住这些事实吗?
总之,即使一般化的逻辑概念没有什么特殊重要性,证明作为助记方法也可能是有用的。
(4)菜谱体系。我们已经讨论了证明的好处,但是我们肯定并未鼓吹所有的证明都应“详尽”地给出。相反,在有的情况下几乎不能这样做;其中一种重要的情况就是对工程专业的学生教授微积分。
如果根据现代严格标准讲授微积分,则其证明要求有一定程度的难度而且技巧精微(“e—证明”)。可是工程专业的学生学习微积分乃着眼于应用,而且他们既无充分的时间,也无充分的训练或兴趣来和冗长的证明打交道或赞赏其技巧精微之处。所以授课者不免强烈地倾向于把所有的证明都割爱。但这样做,
就把微积分降到菜谱的水平了。
菜谱对于原料及程序描述得很详细,但对其规定并无证明,对其配方不说明理由;点心的证明就在于品尝。菜谱可以完全为其目的服务。实际上,它并不需要有任何逻辑体系或助记体系,因为其配方是印刷文字而并非保留在记忆之中。
但是,如果一本微积分教材的作者或者一位大学教师过分遵循菜谱体系,
则它几乎不能为其目的服务。如果他只教方法,不教证明,则那些无吸引力的方法不能被人理解。若他只给出规则而不讲理由,则干巴巴的规则会很快会被遗忘。数学不能用品尝点心一模一样的方式来尝试;如果所有论证都被拒之于课堂之外,则微积分课程很容易成为一种无法消化的知识大杂烩。
(5)不完全证明。在过分证明与菜谱水平之间权衡折衷的最好方法可能是合理使用不完全证明。
对于一个严格的逻辑学家来说,一个不完全的证明根本不算证明。除此之外,我们肯定还应当把不完全证明和完全证明仔细加以区分;把二者混为一谈很不好,张冠李戴则更糟。当一本教科书的作者含糊不清地提出一个不完全证明,在羞愧与夸耀自己的证明是完全证明之间显然进退维谷时,这是痛苦的。
但是应用不完全证明的场合是合适的并且应用得体,却可能很有用。它们的用处不是去代替完全证明(这点它们永远办不到),而是赋予所提问题或出版物以趣味与和谐。
例1.一个n阶代数方程恰有n个根。这个命题被高斯称为“代数的基本定理”。
我们必须经常向那些预备知识不多尚不足以理解其完全证明的学生提出这个命题。可是这些学生知道一阶方程有一个根,两阶方程有两个根。此外,这个困难的命题还有一个很容易证明的部分:没有任何n阶方程会具有多于n个的不同的根。上述这些事实是否构成这基本定理的一个完全证明呢?绝不。但是它们足以使这个基本定理有趣与可信——并且使学生牢记这个基本定理,而这是主要的。
例2.由三面角的棱所形成的平面角中,任意二个面角之和大于第三个面角。显然,这个定理等于肯定:在一个球面三角形中,任意两边之和大于第三边。看到这点,我们自然想起球面三角形与直线三角形之间的类比。上述说明是否构成一个证明呢?绝不;但它们帮助我们了解与记住所提定理。
上述第一个例子有历史价值。大约有250年之久,数学家一直相信这个基本定理而并无完全证明——实际上没有比上述更多的基础。上述第二个例子指出“类比”是推测的重要源泉。在数学中,就象在自然科学与物理科学中一样,
发现常常从观察、类比和归纳开始。这些方法,用在构成一个似真的探索法论证时很得体,特别投物理学家与工程师之所好[参见“归纳与数学归纳法”一节,
第(1),(2),(3)点]。
不完全证明的作用与趣味,在某种程度上可由我们所研究的解题过程加以说明。在解题中的某些经验表明:证明的最初念头经常是不完全的。其中可能有最基本的说明、主要联系和萌芽状态的证明,但其细节尚待以后补充,并且常常很麻烦。有些作者(但不很多)具有以最简单形式提出萌芽状态的证明和主要念头并指出其余细节性质的天赋。这样一个证明虽不完全,但却可能比提出一个带有完全细节的证明更富于启发性。
总之,当我们的目标是所提问题或出版物具有相当的和谐一致性而非严格的逻辑一致性时,不完全证明可用作一种助记方法(当然不可代替完全证明)。
鼓吹不完全证明是非常危险的。但是我们可以用下面几条规则使可能出现的误用不超过一定限度:第一,如果一个证明不完全,就必须在某处以某种方式指出来。第二,除非作者或教师本人对完全证明知道得很清楚,否则他无权对定理提出一个不完全的证明。
此外,人们可能承认:十分得体地提出一个不完全证明毕竟也并不容易。
66.谚语的智慧
解决问题是一项基本的人类行为。事实上,我们大部分有意识的思维都和问题有关。当我们并未沉溺于娱乐或白日作梦时,我们的思想是有方向和有目的的:我们寻找方法,我们试图解决一个问题。
在达到目的和解决问题方面,人们的成败有所不同。这种成败的差别受到注意、探讨与评论,而某些谚语保存了这些评论的精华。无论如何,有大量谚语惊人地描述了解题中典型过程的特点、与它有关的常识、常用策略和常见错误。在谚语中不乏机智的和有点微妙的评述,然而谚语中并没有不矛盾、不含糊的科学体系。正相反,有许多谚语和另一个谚语配合后恰好给出相反的忠告,
并且可以有各式各样的解释。把谚语当作放之四海皆准的智慧权威之源是愚蠢的,然而忽略谚语所提供的探索法过程的生动描述却是可悲的。
把关于制定计划、探求方法、选择行动路线的谚语,简言之,把解题的谚语收集起来并加以分类是个趣味盎然的任务。完成这样一个任务需要大量篇幅,
这里只能列出其中很小一部分;我们所能实行的最好办法是摘引少量谚语以表明在我们表中和第6—14节等处所强调的解题主要阶段。下面,凡属于摘引的谚语我们均加上着重点。
(1)我们为解题所必须做的第一件事是理解题意,
知己知彼,百战不殆。
我们必须对我们所要达到的目的一清二楚,
凡事预则立,不预则废。
这是句老生常谈。不幸,并非每个人都重视这句良言,于是人们常常在没有很好了解他们的工作目的之前冒然开始推测、谈话、甚至忙乱地行动起来,
愚者鲁莽从事,智者深谋远虑。
如果我们目的不明,我们很容易误入歧途,
智者三思而行,愚者轻举妄动。
但是了解问题还不够,我们还必须有求解的愿望。没有解题的强烈愿望,我们就没有解决难题的可能;有了这样的愿望,我们才有解决的可能,
有志者事竟成。
(2)制定一个计划,想出一个适合行动的念头,这是求解中的主要成就。
一个好主意往往是一个好运气、一个灵感,我们必须受之无愧,
天才来自勤奋。
坚持就是胜利。
滴水穿石,功到自然成。
初败不馁,再接再厉。
然而,仅仅不断努力还不够,我们必须尝试以不同的方法变化我们的试验,
千方百计,不厌其烦。
条条大路通罗马。我们还必须使我们的试验适应环境,
看风使舵。
量体裁衣。
因势利导,不可强求。
勿蹈前辙。
智者随机应变,愚者固执己见。
我们甚至应该从一开始便作好方案失败的准备,并且以另一方案作后备,
狡兔三窟。
当然,我们也有可能因变换方案次数过多以致造成时间上的损失。这时,我们可能听到冷嘲热讽,
折腾短,折腾长,有的是时光。
如果我们不忘我们的目的,我们就会少犯错误,
钓鱼的目的在于鱼而不在于钓。
我们力争从我们的记忆中汲取有益的东西,但当一个可能有益的念头涌现时,我们却常常因为它不显眼而未意识到它。专家的念头也许并不比无经验的初学者多,可是他对已经涌现的念头能较明确地意识到并运用自如,
英雄造时势。
智者审时度势,不失良机。
另一方面,专家的长处也可能在于:他密切注意着机会,
莫失良机。
(3)我们应当在正确的时刻开始实现计划,要在它成熟的时候,而不要提前。
我们切不可鲁莽从事,
出门观天,远行问路。
三思而行。
但另一方面,我们又不可磋砣过久,
不入虎穴,焉得虎子。
做最可能的事,抱最好的希望。
想方设法,天助人愿。
我们必须依靠判断,来确定正确的时刻。下面及时提醒我们最常见的判断错误是,
轻信所求。
我们的计划通常只给出一个一般性大纲。我们必须使自己相信其细节是适应计划大纲的,为此,我们必须逐个逐个地仔细审查各个细节,
步步登天,滴滴穿石。
饭要一口一口地吃。
在实现计划时,我们的步骤必须依照恰当的顺序,它常与发明时的次序相反,
愚者最后所行,乃智者最初所为。
(4)回顾所完成的解是工作中一个重要而有启发性的阶段,
温故而知新。
再思则明。
重新审查解答后,我们对所得结果可能格外坚信。我们必须向初学者指出:这种格外坚信是有价值的,两个证明总比一个证明强,
有备无患。
(5)我们这里并没有举尽所有的关于解题的谚语。尚有许多谚语也可以摘引,但它们几乎没有什么新内容而只不过是以上谚语的改头换面。至于解题过程的某些更系统,更复杂的方面则难以用“谚语的智慧”加以概括。
作者常常模仿谚语的特殊措辞(这并不容易)尝试描述解题过程的更系统的方面。下面是几条自拟的“综合”谚语,它们描述稍为复杂一点的情况,
方法取决于目的。
你的五个最好的朋友是:什么,为什么,哪里,何时与怎样。当你需要忠告时,你去问它们而不要问别人。
不要相信一切,也不要怀疑一切。
当你找到第一个蘑菇(或作出第一个发现)后,要环顾四周,因为它们总是成堆生长的。
67.倒着干
如果我们希望理解人类的行为,我们应当把它与动物的行为相比较。动物也“有问题”,也“解决问题”。最近十年来,实验心理学在探索各种动物“解决问题”的行为方面已经作出了重要的进展。我们这里不能讨论这些研究,但我们只大致描述一个简单而又有启发性的实验,并且我们的描述将作为一种分析方法——“倒着干”方法的说明。顺便提一下,这种方法在本书的其他地方也讨论过,其重要描述要归功于帕扑斯(见“帕扑斯”一节)。
(1)让我们试找出下列数学游戏题的答案:有两个容器:小桶的容量是4夸脱,大桶的容量是9夸脱,怎样才能从河中恰好打上6夸脱水呢?
让我们弄消楚所给定的工具,我们必须用这两个容器来工作(已知的是什么?)。我们设想两个圆柱形容器,其底相同,其高为9与4,见图24。如果桶的侧面有刻度,它们是些等间隔的水平横线,用它们可读出水平面的高度,则这个问题容易解答。热而这里并没有这样的刻度,所以我们离解答还远。
图24
目前我们还不知道怎样去量出恰好6夸脱;但我们是否能测量其他某个东西?(如果你不能解决所提问题,首先去解决某个与此有关的问题。你能从已知数据导出来些有用的东西吗?)让我们做点事,让我们稍微干点什么试试,我们能够把大桶装满,然后倒掉小桶那么多;这样,我们就能够得到5夸脱水。我们能不能也得到6夸脱水呢?这里还是两个空桶,我们也能……。
当碰到这个难题时,我们就象绝大多数人所做的那样。我们从两个空桶开始,试试这个,试试那个,我们倒空又装满,而当我们不成功时,我们重新开始,试试别的做法。我们在向前干,即,从给定的初始情况到所期望的最终情况,从已知数据到未知数。经过多次试验,我们偶而也会成功。
(2)但是有卓越才能的人,或者从数学课中所学到的不仅汉是套公式演算的人,不会在这种试验里浪费太多的时间,而是回过头去开始“倒着干”。
要求我们干的是什么?(未知数是什么?)让我们尽可能清楚地想象一下我们所要达到的最后解答是怎样的。让我们设想,在我们面前,大桶中正好有6
夸脱水,而小桶空空如也,如图25(让我们“从所要求的开始,并设所求的已求得”,帕扑斯语)。
从前面什么情况我们能够得到图25所期望的最终情况呢(让我们“研究从什么前项可导出所求结果”,帕扑斯语)?当然,我们能够灌满大桶,即灌到9夸脱。
但是,接着我们应该能够正好倒掉3夸脱。而为了做到这点……我们必须在小桶
中正好有一夸脱水!这就是念头。见图26。
图25
图26
(我们刚才所完成的一步全然并非易事。很少人能够毫不犹豫地想到它。
事实上,认识到这一步的意义,我们就可预见到下面求解的一个梗概。)
可是我们怎么达到刚才图26的情况呢?让我们“再进一步研究,那个前项的前项是什么?”因为河水量是无限的(就我们的目的而言),所以图26的情况等于图27。又等于图28。
图27
图28
容易承认,如果得到图26、27、28中任何一种情况,则可同样得到另外两种情况。但是除非我们以前见过图28,在我们起始作各种试验时偶然遇见过图
28的情况,我们不容易想到它用这两个桶倒来倒去。我们可能已经遇见过类似的情况,现在,刚好,想起图28可由图29产生:即我们把大桶装满水,然后倒出4夸脱给小桶,再由小桶倒入河中,这样接连干两次。我们“最后终于碰到某个已知的东西”(帕扑斯语)并且随着“倒着干”的分析方法,找到了适当的操作序列。
图29
确实,我们已经找到了合适的序列,其次序是逆行的,于是剩下要做的事是把“这个过程反过来”并“从此分析最后所达到的一点开始”(帕扑斯语)。
第一,我们进行图29所建议的操作,得图28;然后我们过渡到图27,接着到图
26,最后到图25。“根据我们的步骤原路反回,我们终于成功地导出所要求的东西”。
(3)根据希腊传说,分析方法的发现乃归功于柏拉图。这传说可能并不十分可靠,但无论如何,如果此法并非柏拉图所发明,也必定有某个希腊学者认为必须将其发明归诸一位哲学天才。
毫无疑问,这个方法有某些发人深思之处。不照直走一条通往目标的道路,
而是从目标走开,转过头来倒着干,这一方法有某种心理障碍。当我们发现合适的操作步骤后,我们必须在心里恰恰按照相反的次序去进行。对于这种反顺序,有某种心理上的嫌恶,如果我们不是小心翼翼提出来的话,它可能阻止一个很能干的学生去了解这种方法。
但是用倒着干的方法去解决一个具体问题不需要天才;任何有点常识的人都能做到。我们集中注意力于所求目标,我们设想我们所乐意达到的最后位置。
前面是哪个位置,我们才能达到那里呢?问这个问题是很自然的,而这样提问时,
我们就是倒着干。十分原始的问题可以很自然地引导我们倒着干(见“帕扑斯”
一节,第4点)。
倒着于是一种属于常识范围的做法,每个人都办得到,而且我们几乎不怀疑,在柏拉图之前的数学家或非数学家也曾使用过这种方法。曾被某个希腊学者看成是堪与柏拉图天才相媲美的成就的是:用一般术语叙述这种做法并且声称它是在解决数学和非数学题时典型有用的一种操作。
(4)现在,我们转到心理实验上——如果,话题从柏拉图转到狗、鸡和黑猩猩上并不太唐突的话。如图30所示,一个篱笆围成正方形的三边而第四边是敞开的。我们把一只狗放在篱笆的一边的点D上,而把某种食物放在另一边的点
F上。对狗来说,问题颇为容易。它可能首先冲过去,好象要直接扑到食物上,
但是,接着它便很快地转过身来,匆忙地绕过篱笆的端点,沿一条圆滑曲线毫不犹豫地跑到食物那里。然而,有时,特别当点D和点F相距甚近时,求解就不那么顺当;这狗要先花点时间向篱笆叫呀,抓呀,扑呀,然后才“想到”转过身去这一个“好念头”(我们暂且姑妄言之)。
图30
把各种各样的动物放在狗的位置上,然后比较它们的行为是饶有趣味的。
这个问题对于一个黑猩猩或者一个四岁儿童来说很容易(对于儿童,放上食物不如放上玩具更有吸引力)。可是对于一只鸡来说,这个问题则惊人地困难。鸡会激动地在篱笆的这一边转来转去,如果它终于会走到食物那里的话,也会在这之前花费相当多的时间。在许多奔跑之后,它偶然可能成功。
(5)关于这个实验我们只不过大致报告一下。我们不应当只根据一个简单实验构造出一大套理论。但是,假定我们准备重复检验和重新评价在这里所出现的明显类比的话,我们注意到这些类比是没有什么坏处的。
绕过障碍是我们在解决任何一类问题时所常做的事;这个试验的价值具有象征性。鸡的行为就象某些人在解题时钻入了牛角尖,试了又试,最后碰巧成功而未深究其成功原因。狗在绕过篱笆去解决它的问题之前又扑、又抓、又叫,
就象我们摆弄那两只桶一样。开头,我们想象在我们的桶上有表明水平面的刻度,这就是一种几乎毫无用处的“抓”,它只不过显示出我们所寻求的东西在表面下隐藏得比较深。我们又象狗那样,首先试图向前干,然后才冒出转过身倒着干的念头。狗在短暂的观察形势之后,转过身去并且匆忙地到达其目的地。
这给人的印象是:(不论正确与否)它具有较高的洞察力。
不,我们甚至也不应当对鸡的愚笨有所非难。在转过身去、从目标走开、
不盯住目标行动、不按照直接通往目的地的路线走,这些方面确实存在着某种困难。鸡的困难同我们的困难显然是可以类比的。
第四部分 问题,提示,解答
这最后一部分提供读者附加的练习机会。
它不需要读者具有高中课程以外的预备知识。但题目并不太容易而且也不是代公式或者有章可循的题目*。其中有的需要独创性和别出心裁。
——————————
*:除题1外,所有题目均选自斯坦佛大学数学竞赛题。有些题目以前在《美国数学月刊》,和加利福尼亚数学会议报告上公布过。在后一种期刊中,有些解答是作者做出的(题1是人所共知的一个题目,它很有趣,故采用之)。
提示给出引导到解答的说明,大多数是从表中引用一个适当的句子;对于一个用心的、准备接受建议的读者,这些提示可能揭示出解答的关键性念头。
解答不仅给了答案,而且也给了引导到答案的过程,当然,读者必须补充某些细节。有些解答最后用了几个字来进一步开拓眼界。
极希望尝试解题的读者,从这些提示及解答中能得到受益非浅的最好机会。如果他用自己的方法得到结果,他也可以把他的方法与本书中的方法进行比较。如果经过认真的努力,他倾向于放弃自己的方法,则这提示可以给予他所寻找的念头。如果这些提示对他都没有帮助,他可以看看解答,尝试把关键的念头从解答中分离出来,然后把书放到一边,自己去试试能否把题做出来。
1.问题
(1)一只熊,从点P开始,向正南走一里,然后改变方向,向正东走一里。
接着,它再向左转,向正北走一里,恰好到达它所出发的P点。问这熊的颜色是什么?
(2)包勃想要一块真正平坦的土地,它有四条边界线,两条边界线恰好南北走向,另两条恰好是东西走向,并且每条边界线恰好是100尺长。问包勃在美国能买到这样的土地吗?
(3)包勃有10个口袋和44块银元。他想把他的银元放进这十个口袋,同时要求每个口袋中的银元数目不同。他做得到吗?
(4)为了给一本厚书的各页标上页码,印刷工人用了2989个数字,问这本书有多少页?
(5)在祖父的文件中发现了一个帐单,
72只火腿$_67.9_这里显然表示火腿总价的数目。其第一位与最后一位数字在这里用一横线代替,因为它们已褪色看不清了。
问这两个看不清的数字是什么?每只火鸡的价格是多少?
(6)已知一个正六边形及其平面中的一点,请过此已知点画一条直线,将所给六边形分成面积相等的两部分。
(7)已知一个正方形。从某一点观测到这个正方形时,(a)张角为90°,(b)
张角为45°,分别求这种点的轨迹(令P为此正方形外的一点,但在同一平面上。
以P点为顶点,包含此正方形的最小角称为此正方形在此张角下从P点所观测到的角)。试清楚地画出P点的两个轨迹,并给出完整的说明。
(8)把下述直线叫做一个立方体的“轴”,此直线连接这立方体表面上的两点,并且使得这立体环绕此线旋转一个角度(此角度大于O°,小于360°)时,
该立方体与它本身重合。
求一个立方体的各个轴。清楚地描述轴的位置,求与每个轴相联系的旋转角。假定立方体的边长为1个单位长度,计算这些轴长的算术平均值。
(9)在一个四面体中(它不必是正四面体),两个相对的棱具有相同的长度
a,并且它们彼此垂直。此外它们每个都垂直于连接它们的中点且长度为b的一线段。用a、b表示四面体的体积,并证明你的答案。
(10)与棱锥底面相对的顶点称为棱锥的顶点。(a)如果一个棱锥体的顶点与底面所有各顶点距离相等,则我们称它为“等腰的”。采用这个定义,证明一个等腰棱锥的底面可内接一个圆,其圆心为棱锥的高线的垂足。
(b)如果一个棱锥的顶点和底面所有边的距离相等,则我们称此棱锥是“等腰的”。 采用这个定义(与前一定义不同),证明此等腰棱锥的底面可外接于一个圆,其圆心是这棱锥的高线的垂足。
(11)求满足下列四个方程的方程组的x,y,u与v,
x+7y+3v+5u=16
8x+4y+6v+2u=-16
2x+6y+4v+8u=16
5x+3y+7v+u=-16
(此方程组显得长而繁,试找出一个窍门来)。
(12)包勃、彼得和保尔一起旅行。彼得和保尔是徒步好手,每人每小时走p
里。包勃有只脚有毛病,他开了部小汽车,能坐两个人,但不能坐三个人;这车每小时走c里。这三个朋友采用下列方案:他们一起出发,保尔与包勃坐车,
彼得步行。过了一会儿,包勃扔下保尔让他步行;包勃回去接彼得,接着包勃与彼得坐车直到他们赶上保尔。在这里,保尔换上去坐车,彼得又步行,就象他们开始时那样。就这样,整个旅程重复了所需要的那么多次数。问,
(a)他们三个人每小时行进多少里?
(b)旅行过程中有几分之几的时间恰有一人乘车?
(c)检验p=0与p=c的极端情况。
(13)三个数成算术级数,另有三个数成几何级数,将此两级数的对应项相加,分别得到:85,76与84;同时把算术级数的三项相加,得126。求此两级数的各个项。
(14)已知x的方程
x4-(3m+2)x2+m2=0
的四个实根成算术级数,求m。
(15)直角三角形的周长为60英寸,垂直于斜边的高的长度为12英寸。求此三角形的各边。
(16)你从山顶看平地上的两点A与B。指向这两点的视线的夹角为γ。第一条视线对水平面的倾角为α,第二条视线的倾角为β。已知点A与B在同一水平线上,两点间的距离为C。
用角α,β,γ与距离C表达山顶在A与B公共水平面上的高度x。
(17)当n为1,2,3时,观察出
)!1(!4
3
!3
2
!2
1
++……+++ n
n
的值分别为1/2,5/6,23/24。请推测其值的一般规律(如果需要,可以再观察一些值),并证明你的推测。
(18)考虑下表
1=1
3+5=8
7+9+11=27
13+15+17+19=64
2l+23+25+27+29=125
推测由这些例子所提示的一般规律,用适当的数学记号表达并加以证明。
(19)正六边形的边长为n(n是一个整数)。用平行于其边的等距平行线把此六边形分成T个等边三角形,每个边长为1个单位长。令V表示在此划分中所出现的顶点数,而L为长度是1的边界线数目(一根边界线属于一个或两个三角形,一个顶点属于丽个或更多的三角形)。当n=1,这是最简单的情况,T=6,V=7,L=12。
考虑普遍情况并用n表达T,V与L(如果能推测出结果就很好了,如果能证明则更佳)。
(20)你能用多少种方式兑换一个美元?(如果已知凑整一美元所需每一种钱币——分币、五分币、一角币,二角五分币、五角币——各有多少个,就确定一种兑换方式)。
2.提示
(1)未知数是什么?熊的颜色。——但是,从数学数据怎么能得出一个熊的颜色呢?
已知的是什么?一个地理位置。——但它似乎是自相矛盾的:这熊以所述方式走了三英里后,怎能回到它的出发点呢?
(2)你知道一个与此有关的问题吗?
(3)如果包勃有大量美元,那么把每个口袋中装上不同数目的美元显然并无困难。
你能重新叙述这个问题吗?能放入10个口袋并且每个口袋中的数目各不相同的美元最小数目是多少?
(4)这里有个问题与你的问题有关:如果该书恰有9页被标上数,印刷者用了多少个数码字?(当然是9个)
这里还有另一个问题与你的问题有关:如果该书恰有99页被标上数,印刷者要用多少个数码字?
(5)你能重新叙述这个问题吗?如果整个价格用分来表示,可用72除尽,则这两个褪色的数字可能是什么?
(6)你能设想一个更好着手的有关问题吗?一个更普遍的问题?一个类比的问题?【见“一般化”一节,第(2)点】
(7)你知道一个与此有关的问题吗?以已知张角观测一条已知直线段的点的轨迹是两个圆弧,它们终止于线段的端点,并关于此线段彼此对称。
(8)假定读者熟悉这正方体的形状,并且只根据观察已经发现了某些轴
——但它们是所有的轴吗?你能证明你所列出的轴的表已经举尽了吗?你的表是否有清楚的分类原则?
(9)看着未知数!未知数是一个四面体的体积——是的,我知道,当已知底面与高时,任何棱锥的体积都能计算(等于两者的乘积,除以3),但在本例中,
底面与高并未给出。你能设想一个更好着手的有关问题吗?(你有没有看到一个更好下手的四面体,它是所给定四面体的等分部分?)
(10)你知道一个与此有关的定理吗?你知道一个有关的……更简单的……
类比的定理吗?是的,我知道在一个等腰棱锥中,高线的垂足是底的中点。这里有一个定理,与你的问题有关,且早已证明。你能利用……它的方法吗?关于等腰三角形的定理是从全等直角三角形来证明的,它们的高是一个公共边。
(11)假定读者关于线性方程组有一定的了解。解决这样一个方程组,我们必须用某种方式合并其方程——找出方程组的关系,它能指出一个特别有利的合并方法。
(12)把条件的各个部分分开,你能写出它们吗?在出发地点与这三个朋友重新碰头的地点之间有三个不同的阶段,
(a)包勃与保尔乘车,
(b)包勃一人乘车,
(c)包勃与彼得乘车。
令这三个阶段的时间长度分别为t1,t2与t3。你如何把条件分成几个适当的部分?
(13)把条件的各个部分分开。你能写出它们吗?令
a-d,a,a+d
为算术级数的各个项,以及
bg-1,b,bg
为几何级数的各个项。
(14)条件是什么?这四个根必须成算术级数。但这方程有一个特点:它仅包含未知数x的偶次幂。因此,如果a是一个根,则一a也是一个根。
(15)把条件的各个部分分开。你能写出它们吗?我们可以把条件分为关于,
(a)周长
(b)直角三角形
(c)到弦的高的三个部分。
(16)把条件的各个部分分开。你能写出它们吗?令a与b表示(未知的)视线的长度,α、β分别表示它们对于水平平面的倾角。我们可以把条件分为关于,
(a)a的倾角
(b)b的倾角
(c)其边为a,b与c的三角形的三个部分。
(17)你识别出分母2,6,24是什么吗?你知道一个与此有关的问题吗?你知道一个类比的问题吗?(“归纳与数学归纳法”一节)
(18)靠归纳发现规律需要观察。请观察右边,左边起初几项和最后几项。
请问其一般规律是什么?
(19)画张图。它可以帮助你归纳地发现这规律,或者它可以指引你找到T、
V、L与n间的关系。
(20)未知数是什么?我们需要寻求的是什么?连这一问题的目标是什么都可能需要弄清楚。你能不能设想一个更好着手的有关问题?一个更普遍性的问题?
一个类比的问题?这里有个非常简单的类比问题:你付一分钱可能有多少方式?(只有一种)。这里有一个更普遍的问题:使用分币、五分币、一角币、两角五分币和五角币这样五种钱币,你付 n分可能有多少种方式?我们尤其关心特例:n=100。
在最简单的特例中,对于小的n值,我们只用实验或观察而不必用伤脑筋的方法就能得出这解答。下面有张简表(读者应加以检验)
n 4 5 9 10 14 15 19 20 24 25
En 1 2 2 4 4 6 6 9 9 13
第一行列出应付款数,一般称为n。第二行列出相应的付法。的数目,一般称为
En(我为什么选择这个记号是我的一个秘密,在现阶段,我还不愿公开)。
我们特别关心E100,但是如果没有一种明确的方法,我们不大可能计算E100。
事实上,本题对读者的要求比前面各题的要求更高;读者应当创造出一点理论来。
我们的问题是普遍性的(对于一个一般的n值计算En),但它被“分离”出来了。你能不能设想一个更好着手的有关问题?一个类比的问题?这里有个非常简单的类比问题:仅使用一分币,求付n分的方法An有多少种?(An=1)
3.解答
(1)你想过这熊是白的而点p是在北极吗?你能证明这点是正确的吗?由于我们对这一点多多少少有所了解,我们把这个问题加以理想化。我们把地球看成正好是个球,而把熊看成是个移动的质点。这个点,向正南或正北移动时,走的是经线的一段弧,而当它向正东移动时,走的是纬线(平行于赤道)的一段弧。
我们必须区分两种情况,
(a)如果这熊沿着经线回到点p(这根经线和它离开p点时所沿的经线不同),
则p点必须是北极。事实上,地球上两根经线相交的仅有的另一点是南极,但是这熊只有向北移动才能离开南极。
(b)如果当熊向正东走一里时,它正好在一个纬圈上定了n次(这里,n可以是1,2,3…),那么,这熊沿着它离开 p点时的同一条经线能够回到p点。在这种情况下,p点不是北极而是十分靠近南极的一个纬圈上的一点(这个纬圈的周长用英里表示时,稍小于2x+1英里)。
(2)我们把地球表示成题1解答中那样的球。包勃所要的土地由两根经线与两个纬线所围成。设想有两根固定的经线以及一个沿离开赤道的方向移动的纬圈:在移动的纬圈中被两根固定的经线所截取的圆弧将越来越短。包勃所要的地的中心应当在赤道上;他在美国不能得到。
(3)在一个口袋中最小的数目显然是O。下一个较大的数目至少是1,再下一个较大的数目至少是2…,而在最后一个(第十个)口袋里的数目至少是9。因此,
所需要的美元总数至少为
O+1+2+…+9=45
包勃做不到这一点:他只有44美元。
(4)一本999页的书需要
9+2×90+3×900=2889
个数字。如果问题中的原书有x页,则
2889+4(x-999)=2989
x=1024
这个问题教导我们:对于未知数有个预先的估计可能是有用的(或者甚至是必要的,如本例)。
(5)如果_679_能被72整除,则它同时能被8和9整除。如果它能被8整除,则数79_必须能被8整除(因为1000能被8整除,于是79_必须是792:即最后一个褪色的数字是2。如果_6792能被9整除,则其各位数字的和必须能被9整除(关于弃
9法的法则),于是这第一个褪色的数字必须为3。
一只火鸡的价格是(在祖父的时代)
$367.92÷72=$5.11
(6)“已知一点和一个具有对称中心的图形(在同一平面上)之位置,求过此点并二等分已知图形面积的直线”。所求的直线当然必经过对称中心(见“发明家的矛盾”一节)。
(7)在任何位置,角的两边必须经过这正方形的两个顶点。只要它们经过这同一对顶点,则角的顶点在圆的同一个弧上移动(根据作为提示基础的定理)。
因此,所求的两个轨迹之中的每一个都包括几段圆弧:在情况(a)中包括4个半圆,在情况(b)中包括8个1/4圆,见图31。
图31
(8)袖在某点穿通正方体的表面,该点或者是正方体的顶点,或者在一条棱上,或者在一个面内。如果这轴经过棱上的一点(但不经过它的端点之一),
则此点必须是中点,否则在旋转后此棱不能与其本身重合。类似的,一个轴穿过一个面的内部,则必经过这面的中心。当然,任何轴都必须经过正方体的中心。因而有三类轴,
(a)4个轴,每个都经过两个相对的顶点:角120°,角240°;
(b)6个轴,每个都经过两个相对的棱的中点;角180°;
(c)3个轴,每个都经过两个相对的面的中心;角90°,180°,270°。
第一类的一个轴长见第12节;其余的轴长更好计算。所求平均值为
1416.113 32634 =++
(此问题对准备研究晶体学的读者可能有用。念过不少微积分的读者可以看出所计算的平均值是正方体“平均宽度”的一个很好的近似,事实上,平均宽度是
3/2=1.5)。
(9)经过长为a的一条棱和长为b的垂直线的平面把这四面体分成了两个更好下手的全等四面体,每个的底面都是 ab/2,高为a/2。因此所求体积为
2223
12 2baaab =×××
(10)棱锥的底面是一个具有n边的多边形。在情况(a),棱锥的n个侧棱相等;在情况(b),其n个侧面的高(从顶点作出的)相等。如果我们画出棱锥的高,
并在情况(a),把高线的垂足与底面的n个顶点连结起来,但在情况(b)把高线的垂足与n个侧面的高线的垂足相连结,则在这两种情况下,我们都得到n个直角三角形,它们的高(也是棱锥的高)是一公共边:我们说这n个直角三角形是全等的。事实上,每个直角三角形的斜边(在情况(a)是一个侧棱,在情况(b)是一个斜高)的长度相等,根据所提问题给出的定义;我们刚才已提到,所有三角形另一边是公共边(棱锥的高),而且还都有一个角是直角。在这n个全等三角形中,
第三边也必须相等;它们是从同一点(高的足)作出的,在同一平面内(底面):
它们构成一个圆的n个半径,在情况(a),这圆外接于棱锥的底面,而在情况(b),
这圆内切于棱锥的底面。(但在情况(b),上述 n个半径垂直于底的各边,这点尚有待证明;这可由解析几何中关于投影的一个著名定理得出)。
十分值得注意的是:一个平面图形(等腰三角形)可以在立体几何中有两个不同的类比。
(11)我们观察到,第一方程对第四方程的关系和第二方程对第三方程的关系相似:其左边的系数相同而次序相反;右边的数的符号相反。将第一方程与第四方程相加,第二方程与第三方程相加,得
6(x+u)+10(y+v)=O
10(x+u)+10(y+v)=0
这可以看成是含两个新未知数(x+u)和(y+v)的线性方程组,由此易得
x+u=0 y+v=0
以-x代替u,-y代替v,代入原方程组的前两个方程中,我们有
-4x+4y=16
6x一2y=-16
由此解得
x=-2,y=2,u=2,v=-2
(12)在出发点与会合点之间,每个朋友旅行了同样的距离(记住,距离=速度×时间)。我们区分条件的两个部分,
包勃与保尔的行程一样多,
ct1-ct2+ct3=ct1+pt2+pt3
保尔与彼得的行程一样多,
ct1-pt2+pt3=pt1+pt2+ct3
从第二方程得出,
(c-p)t1=(c-p)t3
当然,我们假定,汽车跑的比步行要快,即c>p。于是有
t1=t3
即彼得步行时间与保尔一样多。在第一方程中将t3代以t1,我们得
t1/t2=(c+p)/(c-p)
这当然也是t3/t2的值。所以我们得到解,
(a) pc pccttt tttc ++=++ ++ 3 )3()(
321
321
(b) pc pcttt t +?=++ 3
321
2
(c)事实上,o<p<c,这里有两个极端情况:如果p=0,(a)得出c/3而(b)得出1/3;
如果p=c,(a)得出 c而(b)得出0。不必计算也很容易看出这些结果。
(13)这条件很容易分成用下列四个方程所表示的四个部分,
a一d+bg-1=85
a+b=76
a+d+bg=84
3a=126
由最后一个方程得出n=42,于是由第二式得出b=34。将其余两方程相加(以消去
d),我们得
2a+b(g-1+g)=169
因为a与b已知,这是g的二次方程,解得
g=2,d=-26,

g=1/2,d=25
这级数是
68,42,16 17,42,67
17,34,68 或 68,34,17
(14)如果a与一a是具有最小绝对值的根,则在级数中,它们将处在级数中彼此相邻的位置,因此将是下述形式,
-3a,-a,a,3a
因此,所提方程的左侧必有以下形式,
(x2-a2)(x2-9a2)
相乘后,比较同次幂的系数,得到
lOa2=3m十2
9a4=m2
消去a,得到
19m2一108m一36=0
故m=6或-6/19。
(15)令a、b表示两直角边,c表示斜边。条件的三个部分表示如下,
a+b+c=60
a2+b2=c2
ab=12c
注意
(a+b)2=a2+b2+2ab
我们得到
(60—c)2=c2+24c
所以,c=25,a=15,b=20或a=20,b=15,c=25(对三角形来说,这两个答案是一致的)。
(16)条件的三部分可表示为
sinα=x/a
sinβ=x/b
c2=a2+b2-2abcosγ
消去a与b后,得
gbaba
ba
CosSinSinSinSin
SinSincx
222
222
2
+=
(17)我们推测
)!1(
11
)!1(!3
2
!2
1
+?=++……++ nn
n
服从“归纳与数学归纳法”的模式、我们问:当我们从n过渡到下一个值n+1时,
所推测的公式是否仍然成立?根据上式,我们应当有
)!2(
11
)!2(
1
)!1(!3
2
!2
1
+?=+
++
++……++ nn
n
n
n
将上二式相减来检验这点,我们得
)!1(
1
)!2(
1
)!2(
1
+++?=+
+
nnn
n
化简得
)!1(
1
)!2(
2
+=+
+
nn
n
当n=1,2,3,…时,显然此式成立。于是根据上述所提到的模式,我们可以证明我们的推测。
(18)在第n行,右侧看起来是n3,而左侧是n项之和。和的最后一项是第m个奇数,即2m-1,式中
m=l+2+3+…+n= 2 )1( +nn
见“归纳法与数学归纳法”一节,4。因此,左侧和的最后一项应该是
2m-1=n2+n-1
于是我们可以用两种方法导出所考虑的和的首项;从末项倒退n-1步,我们得
(n2+n-1)-2(n-1)=n2-n+1
而从前面一行的最末项往前进一步,我们有
[(n-1)2+(n-1)-1]+2
把它化简后,得到同一结果。好!我们已经知道是
(n2-n+1)+(n2-n+3)+……+(n2+n-1)=n3
这里左侧表示一个算术级数的n个连续项的和,其差为2。如果读者知道这样一个级数的和的规律(首项与末项的算术平均值乘以项数),他可以证明
(n2-n+1)(n2+n-1)n/2=n3
于是证明了推论。(所引公式用一张与图18稍有不同的图很容易加以证明。)
(19)边长为n的正六边形的周长为6n。所以这周长包括6n个长度为1的边界线,并包含6n个顶点。因此,从 n-1转到n,V增加6n个单位,于是
V=1+6(1+2+3+……+n)=3n2+3n+1
见“归纳与数学归纳法”一节,(4)。这六边形被通过其中心的三根对角线分成
6个等边三角形。检查其中一个
T=6(1+3+5+……+2n-1)=6n2
(算术级数和的规则,见第18题的解)。这T个三角形合起来共有3T个边。在这总数3T中,长度为1的内划分线都被计算了两次,但沿六边形周长的6n根线只算了一次。因此,
2L=3T+6n
L=9n2+3。
(致程度较高的读者:根据欧拉关于多面体的定理:T+V=L+1可得此结果。请证明此关系式!)
(20)下面有排列整齐的一系列类比问题:计算An,Bn,Cn,Dn与En。其中每一个都是付n分钱的支付方式的数目;其差别只在于所用的钱币不同,
An,只用一分币
Bn,只用一分币与五分币
Cn,一分币、五分币、一角币
Dn,一分币、五分币、一角币、二角五分币
En,一分币、五分币、一角币、二角五分币、五角币前面已用过记号En (现在理由清楚了吧?!) E n列举出用五种钱币付n分钱的所有方式的数目。但是我们可以区分两种可能性:第一,未用五角币。根据定义,这种付法的数目是Dn;
第二,用五角币(可能性更多)。当第一个五角币放在收款计数器上时,剩下n-50
分要支付,这恰有En-50种方式来支付。我们推出
En=Dn+ En-50
同理
Dn=Cn+Dn-25
Cn=Bn+Cn-10
Bn=An+Bn-5
稍稍留意便可证明:如果我们令
A0=B0=C0=D0=E0=1
(这显然有意义),并且把这些量An,Bn,…,En中任何一个看作等于0,则当其下标为负时,上述公式仍然有效。例如,可立即看出,E25=D25,而这与我们的第一个公式一致,因为 E25-50=E-25=0。
我们的公式使我们能够递推地计算所考虑的量(递推,即通过回到n的较低数值或回到字母表中位置较前的字母的方法)。例如,如果C20与B30已知,我们可以通过简单加法求 C30。在下表中第一行开头标以An;第一列开头标以0,只包含等于1的数(为什么?)。从这些起始的数出发,我们可以用
O 5 10 l5 20 25 30 35 40 45 50
An
Bn
Cn
Dn
En
1 1 1 1 1 1 1 1 1 1 1
1 2 3 4 5 6 7 8 9 lO 11
1 2 4 6 9 12 16 20 25 30 36
1 2 4 6 9 13 18 24 31 39 49
1 2 4 6 9 13 18 24 31 39 50
简单的加法递推地计算其他项:表中任何其他数等于它上面一数或者等于它上面一数与另一个在其左边适当距离的数。例如
C30=B30+C20=7+9=16
一直这样计算下去直到E50=50;即,你可以用恰好50种方式支付50分钱。再进行下去,你可以得到E100=292,即,你可以用292种不同的方式兑换一美元。